Patho Exam 1 (PrepU Study Guide)

¡Supera tus tareas y exámenes ahora con Quizwiz!

The pathophysiology student is studying the general adaptation syndrome (GAS). Which statement by the student demonstrates understanding?

"Epinephrine is released in the alarm stage." Explanation: The general adaptation syndrome involves three stages. The alarm stage is characterized by general stimulation of the sympathetic nervous system, resulting in the release of catecholamines (epinephrine). In the resistance stage, cortisol levels drop as they are no longer needed. In the exhaustion stage, resources are depleted and the signs of "wear and tear, or systemic damage, appear.

A client underwent an open cholecystectomy 2 days ago, and the incision is now in the proliferative phase of healing. What is the dominant cellular process that characterizes this phase of the client's healing?

Collagen secretion by fibroblasts Explanation: The proliferative phase of wound healing is characterized by the action of fibroblasts. Hemostasis, vasoconstriction, and phagocytosis are characteristic of the inflammatory phase, whereas keloid formation is an abnormality in the remodeling phase.

A client with a history of posttraumatic stress syndrome reports frequently reliving the traumatic event. The nurse documents that the client is experiencing:

Intrusion Explanation: The nurse documents that the client is experiencing intrusion, defined as the occurrence of flashbacks or nightmares in which the traumatic event is relived in vivid detail.

A child has a congenital condition in which the thymus gland is absent. What should the nurse include in the education of care of this client to the parents?

Observe and report signs and symptoms of infection. Explanation: The thymus is essential to the development of the immune system because it is responsible for the production of mature, immunocompetent T lymphocytes.

Which body tissue exhibits the highest rate of turnover and renewal?

The epithelial cells of the skin Cells making up the epithelial tissues generally undergo rapid proliferation for replacing injured cells, which is related to their location and function. Renewal of connective and muscle tissue takes place at a much slower pace, whereas nervous tissue is incapable of postnatal regeneration

The nurse is selecting a dressing for a vascular wound that has a dry wound surface. The most appropriate dressing for this wound is one that:

adds moisture to wound bed. Explanation: The most appropriate dressing for this wound is one that adds moisture to the wound bed, as epithelial cell migration requires a moist vascular wound bed.

The entrance of a microbe into an individual's vascular space has initiated opsonization. How will the health care provider explain this process that is critical in stopping the infiltration of the microbe through opsonization? Opsonization:

coats a microbe to activate phagocytosis recognition. Explanation: The coating of particles, such as microbes, is called opsonization, and the coating materials are called opsonins. The opsonin bound to the microbe then activates the phagocyte after attachment to a complementary receptor on the phagocyte. Opsonization does not involve stimulation by T cells, stimulation of the bone marrow, or the lysis of microbes by cytotoxic T cells.

The most critical time for teratogenic influence in embryonic development is:

from day 15 to day 60 after conception. Explanation: The embryo's development is most easily disturbed during the period when differentiation and development of the organs are taking place. This time interval, which is often referred to as the period of organogenesis, extends from day 15 to day 60 after conception.

A client is diagnosed with leukemia. The student nurse working with this client correctly identifies this type of cancer to be:

hematologic cancer. Explanation: Hematologic cancers involve cells normally found in the blood and lymph, thereby making them disseminated diseases from the beginning.

While reviewing the basic information related to hemodialysis, the professor explains that water molecules move through adjacent phospholipid molecules in the cell membrane by:

osmosis. Explanation: Water molecules move through adjacent phospholipid molecules in the cell membrane by osmosis without actually dissolving in the region occupied by the fatty acid side of the chains. Osmosis is regulated by the concentration of nondiffusible particles on either side of the membrane, with water moving from the side with the lower concentration of particles to the side with the higher concentration.

While explaining the individual differences in physical traits in the family group, the health care provider states this is usually a result of:

small DNA sequence variation. Explanation: It is the small DNA sequence variation (1 in every 1,000 base pairs) that is thought to account for the individual differences in physical traits, behaviors, and disease susceptibility. Enzyme interference, chemical exposure, and ischemia are not believed to be responsible for these differences.

A teenager has experienced repeated bacterial infections throughout her life and has recently been diagnosed with leukocyte adhesion deficiency. The nurse explains that this is caused by the teenager's inability to:

synthesize appropriate integrin molecules. Explanation: Persons affected with leukocyte adhesion deficiency are unable to synthesize appropriate integrin molecules. As a result, they experience repeated bacterial infections because their white blood cells are not able to transmigrate through vessel walls.

In major athletic competition, athletes are required to submit to liquid chromatography testing looking for:

use of performance-enhancing agents to increase the chances of winning. Explanation: For some steroid or peptide hormones, mass spectrometry is becoming increasingly useful and can be combined with other analytical techniques, such as liquid chromatography. These approaches provide definitive identification of the relevant hormone or compound according to its chemical or physical characteristics (e.g., unequivocal detection of performance-enhancing agents in sports).

The majority of the cell's protoplasm is:

water. Explanation: Water makes up 70% to 85% of the cell's protoplasm. The second most abundant constituents (10% to 20%) of the protoplasm are the cell proteins.

The nurse is caring for an ex-soldier who has been diagnosed with posttraumatic stress disorder (PTSD). The nurse should prioritize which action?

Assessing the client for depression and risk for suicide Explanation: PTSD creates a significant risk of depression and suicide; the nurse's assessments should prioritize these threats to safety. Having the client talk about his trauma may or may not be appropriate, and may be beyond the nurse's scope of practice. Education may be beneficial, but safety is a priority.

A client has been admitted to the hospital for the treatment of HIV infection, which has recently progressed to overt AIDS. Which nursing action should the nurse prioritize when providing care for this client?

Astute infection control and respiratory assessments Explanation: Although all of the cited assessments and interventions may be of some value, infection control and the early identification of potential respiratory infections are paramount in the care of clients with AIDS.

The release of insulin from the pancreatic beta cells can inhibit further release of insulin from the same cells. This is an example of which type of hormone action?

Autocrine Explanation: Hormones can exert autocrine action on the cells from which they were produced. Retinoids are compounds with hormone-like actions. Juxtacrine action involves a chemical messenger imbedded in a plasma membrane that interacts with a specific receptor on a juxtaposed cell. Arachidonic acid is a precursor for eicosanoid compounds (similar to retinoids).

When hormones act locally rather than being secreted into the bloodstream, their actions are termed:

Autocrine and paracrine Explanation: When hormones act locally on cells other than those that produced the hormone, the action is called paracrine. Hormones also can exert an autocrine action on the cells from which they were produced.

Select the type of lymphocyte that matures in the bone marrow.

B lymphocytes Explanation: B lymphocytes mature in the bone marrow and are essential for humoral or antibody-mediated immunity. The T lymphocytes mature in the thymus. Macrophages are part of the monocytic phagocyte system, are in almost all tissues, and are the mature form of monocytes. Cytotoxic T cells are involved in cell-mediated immunity.

A client tells the health care provider he has heard several myths about viruses. The most appropriate information for the nurse to provide would be:

"Viruses can produce symptoms of disease months to years later." Explanation: Viruses can produce symptoms of disease months to years later. Some viruses, such as hepatitis C, can remain latent for long periods of time without causing disease until months to years later. Viruses are incapable of replicating outside of a living cell and are incapable of replication unless in a host cell. Not every virus causes lysis and death of the host cell during replication.

A client with Graves disease has opthalmopathy and asks the nurse if the eyes will stay like this forever. What is the best response by the nurse?

"With treatment of the hyperthyroid state, the opthalmopathy usually tends to stabilize." Explanation: The ophlalmopathy of Graves disease can cause severe eye problems, including tethering of the extraocular muscles resulting in diplopia; involvement of the optic nerve, with some visual loss; and corneal ulceration because the lids do not close over the protruding eyeball. The opthalmopathy usually tends to stabilize after treatment of the hyperthyroidism.

Which client scenario best describes an example of infection originating with a fomite?

32-year-old man who contracts hepatitis B by sharing a syringe Explanation: An object that carries an infectious organism, such as a dirty syringe, is known as a fomite. Airborne contact with TB does not utilize a fomite, and infection by way of contact with an infected arthropod constitutes zoonosis. Consuming ova in meat would constitute infection by ingestion.

Which core body temperature is within normal range?

37.3°C (99.1°F) Explanation: Core temperature is normally maintained within a range of 36.0ºC to 37.5ºC (96.8ºF to 99.5ºF).

A newborn has been diagnosed with phenylketonuria (PKU). The nurse teaches the mother about a special diet to restrict phenylalanine intake. When does this diet need to be initiated?

7 to 10 days of age Explanation: Dietary treatment of PKU must be started early in neonatal life to prevent brain damage. Infants with elevated phenylalanine levels should begin treatment by 7-10 days of age.

Which statement is not true concerning chromosomes?

Chromosomes do not retain integrity between cell divisions.

The nurse is planning to collect a 24-hour urine sample for hormone assay. In which situation does the nurse collaborate with the health care provider to find an alternate type of testing?

Client has anuria. Explanation: The advantages of a urine test include the relative ease of obtaining urine samples and the fact that blood sampling is not required. The disadvantage is that reliably timed urine collections often are difficult to obtain and rely on adequate renal function. Anuria refers to the absence of urine output.

Which of the following relates to bacterial shape?

Cocci Explanation: The structure and synthesis of the cell wall determine the microscopic shape of the bacterium (e.g., spherical [cocci], helical [spirilla], or elongate [bacilli]).

Which regimen serves as the rationale for AIDS treatment with antiretroviral therapy?

Combination therapy incorporating the five classes of antiretroviral drugs Explanation: Because different drugs act on various stages of the replication cycle, optimal treatment includes a combination of drugs, including reverse transcriptase inhibitors, protease inhibitors, fusion/entry inhibitors, and integrase inhibitors, and multidrug combination products. Each type of agent attempts to interrupt viral replication at a different point. Reference:

Which best describes the structure of a virus?

Consists of a capsid that surrounds a nucleic acid core Explanation: Viruses consist of a protein capsid that surrounds a nucleic acid core (either RNA or DNA). Bacteria have both a cell wall and a cell membrane. Spirochetes have a helical cell wall, and mycoplasma are bacteria that lack a cell wall.

A previously healthy 42-year-old client was recently diagnosed with hypertension. Select the response that best describes a positive reaction to the acute stress.

Decreasing dietary sodium and fat intake Explanation: In an acute stress, a healthy or positive response to a diagnosis such as hypertension would be to redirect attention to promoting health (e.g., decreased sodium and fat intake). The other options are not positive responses.

A pregnant woman's ultrasound reveals intrauterine growth retardation (IUGR) and a TORCH agent is thought to be the cause. Which nursing action is appropriate?

Determine whether the client has been immunized against rubella. Explanation: TORCH stands for toxoplasmosis, other, rubella (i.e., German measles), cytomegalovirus, and herpes. Hepatitis, tetanus, and cholera are not included among lists of TORCH agents.

Which type of imaging is preferred to evaluate the bone density of a client with hyperparathyroidism?

Dual energy x-ray absorptiometry (DXA) scan Explanation: Preferred types of scans for endocrine tissues include MRI of the pituitary gland and hypothalamus, CT scan of the adrenal glands, DXA scan of bone density, and ultrasound of possible nodules of the thyroid gland. Bone density may be decreased in a client with hyperparathyroidism.

A client's lab report returns and a nurse is explaining to the client the significance of the changes. The nurse states that the finding is implicated as a precursor of cancer. Which finding was most likely on the lab report?

Dysplasia Explanation: Dysplasia is characterized by deranged cell growth of a specific tissue that results in cells that vary in size, shape, and organization. It is strongly implicated as a precursor of cancer.

A nurse is conducting a community education class on genetics. The nurse sees that the class understands the information when they say that the Human Genome Project involved sequencing of which of the following?

Entire human genetic complement

Which pathophysiologic phenomeon may result in a diagnosis of Cushing disease?

Excess ACTH production by a pituitary tumor Explanation: Three important forms of Cushing syndrome result from excess glucocorticoid production by the body. One is a pituitary form, which results from excessive production of ACTH by a tumor of the pituitary gland. Hypopituitarism and destruction of the adrenal cortex are associated with Addison disease. Disruption of the HPA system is not implicated in the etiology of Cushing disease.

A client who is in the intensive care unit after experiencing a massive heart attack begins to go into renal failure. The nurse plans the client's care knowing that the client is in which phase of Selye's general adpatation syndrome?

Exhaustion Explanation: The nurse plans care knowing that this client is in Selye's exhaustion stage. In this stage, the body's ability to defend itself is overwhelmed and resources are depleted; signs of systemic damage, such as renal failure, begin to appear.

A 9-month-old infant has been diagnosed with botulism after he was fed honey. The child's mother was prompted to seek care because of this child's sudden onset of neuromuscular deficits, which were later attributed to the release of substances by Clostridium botulinum. Which virulence factor contributed to this child's illness?

Exotoxins Explanation: Exotoxins are proteins released from the bacterial cell during growth, as in the case of botulism poisoning. Adhesion factors, evasive factors, and endotoxins are not evident in this release of botulinum toxin.

Therapeutic radiation can be delivered by which methods? Select all that apply.

External beam Brachytherapy Systemic therapy Explanation: Therapeutic radiation can be delivered in one of three ways: external beam, where beams generated by a linear accelerator at a distance and aimed at the client's tumor; brachytherapy, where a sealed radioactive source is placed close to or directly in the tumor site; and systemic therapy, in which radioisotopes with a short half-life are given by mouth or injected into the tumor site. Light therapy is not a form a radiation therapy.

A client with type 1 diabetes mellitus is experiencing hyperglycemia. Consequently, the client's cells lack intracellular glucose, resulting in glucose accumulating in the blood. Which process would best allow glucose to cross into the cell membranes?

Facilitated diffusion Explanation: Transport molecules perform facilitated diffusion, in which one substance carries another substance across a cell membrane. Simple diffusion does not require a transport molecule. Glucose does not cross the cell membrane by secondary active transport or endocytosis.

Most physiologic control systems function under positive feedback mechanisms.

False Explanation: The reason most physiologic control systems function under negative rather than positive feedback mechanisms is that a postive feedback mechanism interjects instability rather than stability into a system.

For the most part, the stress response is meant to be acute and ongoing.

False Explanation: The stress response is meant to be acute and time limited.

Which body response to an acute inflammation will the nurse assess if the client is experiencing a systemic response?

Fever and tachycardia Explanation: The acute-phase system response includes fever, hypotension, and increased heart rate [tachycardia], anorexia, release of neutrophils into the circulation, and increased levels of corticosteroid hormones.

Inflammation can be either local or systemic. What are the most prominent systemic manifestations of inflammation?

Fever, leukocytosis or leukopenia, and the acute phase response Explanation: The most prominent systemic manifestations of inflammation include the acute phase response, alterations in white blood cell count (leukocytosis or leukopenia), and fever. A widening pulse pressure is not indicative of systemic inflammation, and thrombocytopenia is a hematologic disorder, not an indication of systemic inflammation.

Which risk factor increases the chances of the development of a neural tube defect during embryonic development?

Folate deficiency Explanation: Folate deficiency has been closely linked as a risk factor for the development of neural tube defects because folate is needed for the synthesis of amino acids and DNA. It is now recommended that women take folic acid prior to conception to prevent neural tube defects.

The nurse is caring for a client whose temperature is increasing. Which other vital sign/body response will the nurse monitor for an increase?

Heart rate Explanation: Critical to the analysis of a fever pattern is the relation of heart rate to the level of temperature elevation. The features of a systemic response includes fever, hypotension, increased heart rate, anorexia, release of neutrophils into circulation, and increased levels of corticosteroid hormones. The remaining options are not as closely related to fever as heart rate.

A client is suffering from the effects of the opportunistic infection, Cryptosporidium parvum. An important nursing intervention would be to encourage which action?

Hydration Explanation: Crytosporidium parvum has clinical features ranging from mild diarrhea to severe, watery diarrhea with a loss of up to several liters per day. Hydration is an important consideration.

A nurse is caring for an adolescent with posttraumatic stress syndrome. The client reports having difficulty concentrating and has an increased startle reflex. The nurse's documentation includes the presence of:

Hyperarousal Explanation: The nurse should document the presence of hyperarousal, defined as the presence of increased irritability, difficulty concentrating, exaggerated startle reflex, increased vigilance, and/or concern over safety in a client with posttraumatic stress syndrome.

Which immunoglobulin is primarily found in secretions and has a primary function of providing local immunity on mucosal surfaces?

IgA Explanation: IgA is found in saliva and tears, and is a primary defense against infections in mucosal tissues. IgM is the first immunoglobulin to appear in response to antigen invasion. IgG is the most abundant immunoglobulin and can cross the placenta. IgD is an antigen receptor on B-cell membranes.

What is the term for parasitic relationships between microorganisms and the human body in which the human body is harmed?

Infectious disease Explanation: A parasitic relationship is one in which only the infecting organism benefits from the relationship and the host either gains nothing from the relationship or sustains injury from the interaction. If the host sustains injury or pathologic damage in response to a parasitic infection, the process is called an infectious disease. Mutual and commensal relationships do not harm the human body. Communicable diseases can be passed from one human to another; they are not parasitic.

A client has begun taking acyclovir, an antiviral medication, to control herpes simplex outbreaks. What is this drug's mechanism of action?

Interference with viral replication processes Explanation: Acyclovir, like most antiviral drugs, interferes with normal viral replication. Antiviral drugs do not normally affect adhesion, and viruses are not associated with the production of exotoxins.

Which statement is true concerning mitochondrial DNA?

It is inherited from the mother by her offspring. Explanation: Part of the DNA of a cell resides in the mitochondria. Mitochondrial DNA is inherited from the mother by her offspring.

Which cancer is the leading cause of cancer death?

Lung Explanation: Lung cancer is the leading cause of cancer deaths in men and women in the United States.

Which gland is often referred to as the master gland because it secretes many hormones?

Pituitary Explanation: The pituitary gland has been called the master gland because its hormones control the functions of many target glands and cells. That is not a term used to refer to the other options.

What is the main effect of HIV infection?

Poor helper T-cell function Explanation: Helper T cells are also known as CD4+ T cells. These CD4+ T cells are necessary for normal immune function and are the main target of HIV.

The nurse is educating a client with a newly diagnosed thyroid cancer. Which testing procedure should be reviewed because it allows good examination of the tissue structure and provides information about the tissue function as well?

Positron emission tomography/CT (PET/CT) Explanation: PET/CT imaging is advantageous in that the CT component allows a good examination of the tissue structure, whereas the PET component provides information about tissue function. PET/CT has been demonstrated to be useful in managing thyroid cancers. DEXA scan is routinely used for the diagnosis and monitoring of osteoporosis and metabolic bone imagining. Ultrasonography scanning provides good structural imaging and is used frequently to aid in visualization of a lesion for biopsy. MRI provides information about structural changes within solid tissue.

Triplet codes of three bases are the genetic codes used in transmitting genetic information necessary for which one of these?

Protein synthesis Explanation: Codons, triplet codes of bases, form the genetic code necessary for protein synthesis. Chromatin is a complex composed of DNA, RNA, and several types of protein. Polymerase enzymes synthesize RNA in the nucleus. DNA is composed of nucleotides, which are DNA strands bound together by hydrogen bonds.

The route considered the most accurate to measure a core body temperature is:

Rectal Explanation: The rectal temperature is used as a measure of core temperature and is least invasive of all of these options.

Which health problem may be identified by a TORCH screening test?

Rubella and herpes Explanation: The acronym TORCH stands for toxoplasmosis, other, rubella (i.e., German measles), cytomegalovirus, and herpes, which are the agents most frequently implicated in fetal anomalies. The TORCH screening test examines the infant's serum for the presence of antibodies to these agents.

When caring for a client with a wound that is healing by primary intention, the nurse recognizes which characterization best describes this type of wound?

Surgical incision Explanation: A sutured surgical incision is an example of healing by primary intention. Larger wounds (e.g., burns and large surface wounds) or wounds purposely left open due to infection heal by secondary intention.

The nurse understands that a positive ELISA test reported in a baby born to a woman who is HIV positive indicates:

The baby has antibodies from the mother and it is uncertain whether the baby is infected with HIV. Explanation: The diagnosis of HIV infection in children born to HIV-infected mothers is complicated by the fact that infants have the maternal anti-HIV IgG antibody for approximately 6 months. Consequently, infants born to HIV-infected women can be HIV-antibody-positive by ELISA for up to 18 months of age even though they are not infected with HIV.

The health care provider is assessing the functional integrity of all spinal nerves utilizing a pinpoint pressed against the skin. A normal response would be interpreted as:

The withdrawal reflex is activated. Explanation: Observation of a normal withdrawal reflex rules out peripheral nerve disease, disorders of the dorsal root and ganglion, disease of the myoneural junction, and severe muscle diseases. Having no response is abnormal and may identify neurologic damage. A verbal response is not a reflex response.

Which role does adipose tissue play in the body?

Thermal insulation Explanation: Adipose tissue stores triglycerides, which act as energy reserves. It also insulates the body from heat loss, and supports organs and structures. Collagen is made by connective tissue called fibroblasts. Energy is produced in the mitochondria. Electrolyte balance is regulated by the renal system and the cell ion channels.

A nurse sees that a client's X chromosome is metacentric. What does the nurse interpret this to mean?

This a normal occurrence. Explanation: The nurse should interpret a metacentric X chromosome to mean that the arms of the X chromosomes are approximately the same length. This is a normal occurrence so it would not result in severe intellectual disability, blindness, or physical deformity.

Microscopic examination of tissue samples from a deceased client's liver reveal that the hepatocytes contain pathologic vacuoles of fat. The nurse should understand what significance of this finding?

This phenomenon may have been reversible if the client had undertaken lifestyle changes. Explanation: Fatty changes are considered to be reversible. This phenomenon is not known to have a genetic predisposition and it is not the result of metastasis or free radicals.

The pregnant client asks if there are any precautions she should take in caring for the family cat. The nurse explains that she should limit contact with the cat's litter box. The litter box may contain which organism that can cause fetal malformations?

Toxoplasmosis Explanation: The domestic cat can carry the organism Toxoplasma gondii, excreting the protozoa in its feces. It has been suggested that pregnant women should avoid contact with excrement from the family cat. The other options are infectious teratogenic agents, but are not carried by cats.

Levels A, B, and C are levels assigned to potential agents of bioterrorism. What are these categorical assignments based on?

Transmissibility Explanation: Potential agents of bioterrorism have been categorized into three levels (A, B, and C) based on risk of use, transmissibility, invasiveness, and mortality rate.

A client reports chronic pain. Assessment indicates it is located at the T6 dermatome. How will this information assist the nurse to develop an effective plan of care?

Treatment can be tailored to the specific location. Explanation: A dermatome is the region of the body wall that is supplied by a single pair of dorsal root ganglia. Isolating the location of the pain allows for focused treatment. Chronic pain is best handled by a multidisciplinary team that includes specialists in areas such as anesthesiology, nursing, physical therapy, social services, and surgery. Management may reduce referred pain but may not alleviate it because the visceral and somatic neurons share the same dorsal horn projections. Clients with chronic pain benefit most from interventions that include physical, emotional, financial, and spiritual components of care. Using biologic treatments alone is less effective than the combination of all modalities.

Which condition is an example of physiologic hyperplasia?

Uterine enlargement in pregnancy Explanation: Two common types of physiologic hyperplasia are hormonal and compensatory. Breast and uterine enlargements during pregnancy are examples of a physiologic hyperplasia. The other examples are nonphysiologic hyperplasia.

What is the most common mode of transmission of HIV?

Vaginal and anal intercourse Explanation: The most common method of transmission of HIV is from vaginal and anal sexual intercourse. Only 25% of newly diagnosed HIV infections are in those individuals who share needles. Transmission from mother to infant is the most common way children are affected but not the most common overall. Occupation exposure and blood transfusion are the least common of the choices provided.

A recovery room nurse monitoring a client for adverse effects of cold cardiplegia assesses for:

Ventricular dysrhythmia Explanation: The nurse should assess for ventricular dysrhythmia, decreased cerebral blood flow, and postoperative myocardial depression.

The nurse is caring for a client with a chronic wound. The most important intervention for the nurse to include in the plan of care would be:

Vitamin C and zinc supplements Explanation: Vitamins play an essential role in the healing process. Vitamin C is needed for collagen synthesis. Zinc has been found to aid in re-epithelialization. Vitamin C and zinc supplements are often ordered for clients with chronic wounds to promote wound healing. Steroid therapy would decrease wound healing. A diet high in proteins and carbohydrates would be ordered to promote wound healing.

A client asks the nurse how a broad-spectrum antibiotic works. The best response would be that they are active against:

a wide variety of Gram-positive and Gram-negative bacteria. Explanation: Broad-spectrum antibiotics, such as the newer cephalosporins, are active against a wide variety of Gram-positive and Gram-negative bacteria. Antibiotics are not used against viruses.

When educating a client about glargine, the nurse should explain that this medication:

has a prolonged absorption rate and provides a relatively constant concentration for 12-24 hours. Explanation: Glargine is long-acting insulin that has a slow, prolonged absorption rate and provides a relatively constant concentration over 12-24 hours. Rapid acting insulin has a rapid onset and peaks in about 5 minutes after injection. Short acting insulin will have a peak effect within 30 minutes, and thus it can be taken after a meal. Intermediate-acting insulin is a combination with short acting insulin so it is safe to take anytime throughout the day.

A client is said to be in the chill stage of the fever process when the nurse:

observes piloerection on the skin. Explanation: During the second stage or chill, there is the uncomfortable sensation of being chilled and the onset of generalized shaking (rigors). Vasoconstriction and piloerection usually precede the onset of shivering. At this point, the skin is pale and covered with goose flesh. Sweating is an indication of the third stage or flushing. The other options are not descriptive of stages but rather interventions.

As a nurse prepares to change a client's dressing, the client states that she is afraid that it will be painful. Noting that the client's heart rate and respiratory rate have increased, the nurse adjusts the plan of care to reflect care for a client in which phase of Selye's general adaptation syndrome?

Alarm Explanation: The nurse plans care for a client in the alarm stage of Selye's general adaptation syndrome. During this stage, the sympathetic nervous system is stimulated, which can result in increases in heart and respiratory rate.

Which type of a hypersensitivity reaction involves failure of the development of self-tolerance?

Autoimmune reaction Explanation: A key feature of the immune system is its ability to differentiate between foreign antigens and self-antigens. The capacity of the immune system to differentiate self from non-self is called self-tolerance. The development of self-tolerance relies on two coordinated processes: central and peripheral.

A hospital client with a diagnosis of type 1 diabetes has been administered a scheduled dose of regular insulin. Which effect will result from the action of insulin?

Promotion of glucose uptake by target cells Explanation: The actions of insulin are threefold: (1) it promotes glucose uptake by target cells and provides for glucose storage as glycogen; (2) it prevents fat and glycogen breakdown; and (3) it inhibits gluconeogenesis and increases protein synthesis. Glucagon, not insulin, promotes glycogenolysis.

A client who is a performer expends a tremendous amount of energy while on stage, but his body is not harmed by the added stress. Protection against the harmful effects of stress is primarily a result of:

physiologic reserve. Explanation: The ability of body systems to increase their function in response to the need to adapt is the physiologic reserve. Anatomic reserve is attributed to organ structure; immune system compensation may be a component of physiologic reserve.

A nurse administering a synthetic insulin to a client knows that production of synthetic insulin was made possible by:

recombinant DNA technology.

Which symptom indicates the next stage of a fever after a prodrome?

A chill Explanation: The physiologic behaviors that occur during the development of fever can be divided into four successive stages: prodrome, chill, flush, and defervescence. The stages are successive.

Which client has the highest risk of contracting an opportunistic infection?

A client who has had HIV for 3 years and has a CD4+ count of 50 cells/μL Explanation: The lower the CD4+ T-cell count is, the higher the likelihood of contracting an opportunistic infection. In a client who is newly infected with HIV, the CD4+ T-cell count would he high, even though the viral load is high. Platelets are not an important factor when considering the opportunity for an infection.

A health care provider suspects a client has developed posttraumatic stress disorder (PTSD). Select the necessary criterion to validate the diagnosis.

At least 1 month of clinically significant distress affecting a person after witnessing or experiencing a traumatic event that affects other parts of his or her life Explanation: The triad of symptoms of intrusion, avoidance, and hyperarousal that characterize PTSD must be present together for at least 1 month, and the disorder must have caused clinically significant distress or impairment in social, occupational, and other areas of functioning.

Multifactorial inheritance is similar to polygenic inheritance because both involve which of these?

Multiple alleles at different loci Explanation: Both multifactorial and polygenic inheritance involves multiple genes at different loci. Multifactorial is different because it also involves the effects of the environment on genes. Unlike Mendel's law, multifactorial inheritance and polygenic inheritance are unpredictable. Homozygous pairing of alleles is characteristic of recessive traits.

In order to understand gene therapy, which statement about genes in cells is accurate? Select all that apply.

Operating genes are ones necessary for normal function of a cell. Genes determine differentiating characteristics of a cell type. Explanation: Genes expressed in cells fall into two categories: operating genes that are necessary for normal function of a cell and genes that determine the differentiating characteristics of a particular cell type. In many adaptive cellular responses, expression of differentiation genes is altered, whereas that of the operating genes remain unaffected. Thus, a cell is able to change size or form without compromising its normal function.

A client with severe hypoglycemia is unconscious. Which method of providing glucose should be avoided?

Orange juice orally Explanation: When clients are unconscious it is not safe to attempt to have them swallow liquids. Alternate routes that reduce the risk of choking such as buccal absorption, intramuscular or intravenous injections are preferred.

Select the statement that best describes the difference between ribonucleic acid (RNA) and deoxyribonucleic acid (DNA).

RNA is a single-stranded molecule. Explanation: RNA is a single-stranded molecule, while DNA is a double-stranded molecule. The sugar in each nucleotide of RNA is ribose instead of deoxyribose. The pyrimidine base thymine in DNA is replaced by uracil in RNA. RNA, like DNA, is a large molecule made up of a long string of nucleotides.

A nurse working in a genetic clinic understands that the process of transcription is initiated by which of the following?

RNA polymerase Explanation: The nurse knows that the process of transcription is initiated by RNA polymerase.

A client in the acute stage of inflammation will experience vasodilation of the arterioles and congestion in the capillary beds. The nurse would assess the client's skin for:

Redness Explanation: Vasodilation of the arterioles and congestion of the capillary beds result in an increased pooling of blood, leading to redness. The site would also have increased painful sensation and be warmer to touch. It would not result in an increase in bacterial load.

A client has suffered an electrical injury to the hand. Which condition will the nurse expect to find?

Tissue damage at the skin site where the current entered the body Explanation: With an electrical injury, the nurse would expect the most severe injury at the site where the current entered the body. The skin could be blackened, reddened, and several layers of the skin could be destroyed depending on the severity of the injury. There is still injury if the client has dry, thick skin, and there is no correlation between indoor versus outdoor injury.

A client goes to radiology for a series of x-rays. As the lead apron is placed over the client, he asks why it is needed. The client should be told that the vest is to protect:

against chromosomal breakage. Explanation: X-rays have been shown to cause chromosomal breakage and lead to rearrangement and mutation; they do not directly affect the cytoplasm. Proteins and lipids may be altered by X-rays, but they are quickly degraded and removed and therefore have no effect.

The nurse is reviewing the client's medical record for the results of a Western blot test with the understanding that:

the Western blot is used to confirm a diagnosis of HIV infection. Explanation: The Western blot test is more specific than the ELISA, and in the case of false-positive ELISA, the Western blot test can identify the person as uninfected. ELISA tests have high false-positive rates.

A student asks, "What does cell-mediated immunity mean to the client?" The instructor responds, "This means:

the body is trying to defend itself against intracellular microbe invasion by engulfing and destroying the microbe." Explanation: Cell-mediated immunity, which defends against intracellular microbes such as viruses, is provided by cells called T lymphocytes. Some T lymphocytes activate phagocytes to destroy microbes that have been engulfed, whereas others kill any type of host cell that is harboring microbes.

A client with a diagnosis of sepsis has received intravenous immune globulin (IVIg) as partial treatment. The nurse knows that which client response would best suggest an accurate understanding of IVIg treatment?

"A big part of my IVIg treatment is actually stimulating and supplementing my immune system to do the work itself." Explanation: IVIg involves infusion of pooled antibodies which supplement and stimulate the client's immune system to respond above and beyond its own capacities. It is not collected from individuals who have had similar infections. The primary effect of IVIg is not white blood cell production. Stimulation of fever, inflammation, and tissue repair are more closely associated with cytokine therapy.

A nurse who works in the office of an endocrinologist is orienting a new staff member. Which teaching point should the nurse include in the orientation?

"A single hormone can act on not only one process or organ but often on several different locations or processes." Explanation: A single hormone can exert various effects in different tissues, or conversely, a single function can be regulated by several different hormones. Hormones act both distantly from their source and more locally, as in the case of autocrine and paracrine actions. Hormones are normally present at all times.

The nurse is preparing to administer a unit of blood to a client. The client says that he is not sure he wants to give consent to receive the blood transfusion because he does not want to get AIDS from the blood. Select the best response from the nurse.

"All blood is now tested for the AIDS virus, so the risk of getting AIDS from a blood transfusion is extremely low." Explanation: Some transfusions of whole blood, plasma, platelets, or blood cells before 1985 resulted in the transmission of HIV. Since 1985, all blood donations in the United States have been screened for HIV, so this is no longer a transmission risk; however, infection can occur from transfused blood that was screened for HIV antibody and found negative because the donor was recently infected and still in the window period.

When a client who has been newly diagnosed with HIV asks, "What are the chances that I can be cured?," what is the nurse's most therapeutic response?

"Although there is no current treatment that provides a possible cure, there are ones that have successfully managed the infection." Explanation: There is no cure for HIV infection. The medications that are currently available to treat HIV infection decrease the amount of virus in the body, but they do not eradicate HIV. After HIV infection is confirmed, a baseline evaluation should be done. This evaluation should include a complete history and physical examination and baseline laboratory tests including a complete blood count with differential. Routine follow-up care of a stable, asymptomatic person infected with HIV should include a history and physical examination along with CD4+ cell count and viral load testing every 3 to 4 months. People who are symptomatic may need to be seen more frequently. Therapeutic interventions are determined by the level of disease activity based on the viral load, the degree of immunodeficiency based on the CD4+ cell count, and the appearance of specific opportunistic infections.

Following surgery for appendicitis, a teenaged client notes four small "stab" wounds on the abdomen. The client is obviously worried about body appearance. The nurse explains, "Your body will heal quickly and tissue repair will allow for regeneration of any cells needed." The client asks, "What does regeneration mean?" The nurse responds that tissue repair by regeneration means:

"Any injured cells are replaced with cells of the same type. Therefore, after healing, the wound will look like your surrounding skin." Explanation: Tissue repair can take the form of regeneration, in which injured cells are replaced with cells of the same type, sometimes leaving no residual trace of previous injury. Replacement by connective (fibrous) tissue will lead to scar formation.

The nurse working at a community clinic is caring for a young male client. The client asks the nurse if he could get AIDS from being sprayed in the face with breast milk. Select the best response by the nurse.

"Breast milk can contain the virus that causes AIDS, but to be infected, the milk needs to be in contact with your mucous membranes or an open sore." Explanation: HIV can be transmitted through breast-feeding; however, transmission requires contact with mucous membranes or an open sore.

A client who is diagnosed with breast cancer asks the nurse if cancer cells ever die. Which statement is the nurse's best response?

"Cancer cells differ from normal cells by being immortal and have an unlimited life span." Explanation: The best response for the nurse to make is to tell the client the truth: that cancer cells are immortal and can divide an infinite number of times, hence achieving immorality. The other responses are not therapeutic.

A pregnant client's α-fetoprotein (AFP) returns elevated. The couple ask the health care worker to explain what this means. Which is the best response?

"Elevated AFP means your baby is at risk of having a neural tube defect like spina bifida (an opening in the spine)." Explanation: Maternal and amniotic fluid levels of AFP are elevated in pregnancies where the fetus has a neural tube defect (i.e., anencephaly and open spina bifida) or certain other malformations such as an anterior abdominal wall defect in which the fetal integument is not intact. Screening of maternal blood samples usually is done between weeks 16 and 18 of gestation. Although neural tube defects have been associated with elevated levels of AFP, decreased levels have been associated with Down syndrome.

Which statement from a group of young adults demonstrates the need for further teaching related to HIV and prevention of the spread of HIV?

"Having oral sex is one way I can prevent passing on HIV to my partner." Explanation: Sexual contact is the most frequent mode of HIV transmission. There is a risk of transmitting HIV when semen or vaginal fluids come in contact with a part of the body that lets them enter the bloodstream. This can include the vaginal mucosa, anal mucosa, and wounds or sores on the skin. Condoms are highly effective in preventing the transmission of HIV. Unprotected sex between men is still the main mode of transmission. During the window period, a person's HIV antibody test result will be negative, but he or she can still transmit the virus.

The nurse is providing education to the parent of a male diagnosed with Marfan syndrome. The nurse knows that her teaching has been effective when the parent states which of the following?

"His participation in sports may need to be limited." Explanation: Marfan syndrome affects several organ systems, including the eyes, cardiovascular, and skeletal systems. The risks associated with participation in sports depend on which organ system is involved. There is no cure for Marfan syndrome and the most life-threatening aspects of the disorder are the cardiovascular defects.

A nurse is teaching parents of a child with scarlet fever. Which statement indicates that the parents understand the treatment for the child?

"I will give the antibiotic for the full 10 days." Explanation: Streptococcus pyogenes, the agent of scarlet fever, is a gram-positive streptococcal organism. It is spherical, grows in chains, and stains purple by Gram stain. Not all antibiotics are effective against all pathogenic bacteria. Some agents are effective only against gram-negative bacteria, and others are specific for gram-positive organisms. The so-called broad-spectrum antibiotics, such as the newest class of cephalosporins, are active against a wide variety of gram-positive and gram-negative bacteria.

The nurse is teaching a group of college students about reducing the risk of HIV transmission during sexual relations. The nurse makes which appropriate teaching point?

"If a person has a sexually transmitted disease (STD), there is an increased risk for HIV infection." Explanation: Sexual contact is the most frequent mode of HIV transmission. This includes vaginal, oral, and anal sex. The HIV-infected person is infectious even when no symptoms are present and before seroconversion. People with other sexually transmitted diseases (STDs) are at increased risk for HIV. Natural or lambskin condoms do not provide the same protection from HIV as latex because of the larger pores in the material.

When providing nutrition education to the client with diabetes, the nurse should include which statement regarding fat intake?

"If you have diabetes, avoiding saturated fats is important." Explanation: Because diabetes is a risk factor for cardiovascular, cerebrovascular, and peripheral vascular disease, it is recommended that less than 7% of daily calories should be obtained from saturated fat and that dietary cholesterol be limited to 200 mg or less, and intake of trans fats minimized.

When working with a client who has end-stage renal disease (ESRD) and is receiving peritoneal dialysis, the concept of diffusion can be explained by which statement?

"If your potassium level is high, then K+ particles will move from your peritoneal cavity into the dialysis solution, where the concentration of K+ is lower." Diffusion refers to the passive process by which molecules and other particles in a solution become widely dispersed and reach a uniform concentration because of energy created by their spontaneous kinetic movements. In the process of reaching a uniform concentration, these molecules and particles move "downhill" from an area of higher to an area of lower concentration. Lipid-soluble molecules, such as oxygen, carbon dioxide, alcohol, and fatty acids (not K+) become dissolved in the lipid matrix of the cell membrane and diffuse through the membrane in the same manner that diffusion occurs in water. Diuretics are not very effective if a person has ESRD. The rate of diffusion depends on how many particles are available for diffusion, the kinetic movement of the particles, and the number and size of the openings in the membrane through which the molecules or ions can move. The environmental temperature does not play a role in this.

The mother of a 2-year-old newly diagnosed with type 1 diabetes asks why insulin has to be given by injection. The best response by the nurse is:

"Insulin is destroyed by the stomach contents and has to be administered by injection." Explanation: Insulin is destroyed by the gastrointestinal tract and needs to be administered via injection or inhalation. Type 1 diabetes is not treated with oral medications at this time. Insulin is administered subcutaneously rather than in the vein. The statement about the mother not having to give injections once the child is older does not address the mother's concerns.

A client describes himself as being "devastated" after hearing that his HIV test has come back positive. Which response by the nurse is most appropriate?

"It's entirely normal to feel that way. We are going to try our best to help you get all the support you need." Explanation: The nurse's best response is to validate the client's statement and to affirm a commitment to support him. It is presumptuous to conclude that by saying he is "devastated" he means that he is ashamed or afraid. Similarly, it is presumptuous for the nurse to claim that he or she knows how the client is feeling.

A diabetic client presents to a clinic for routine visit. Blood work reveals a HbA1C of 11.0% (high). Which response by the client may account for this abnormal lab result?

"My meter broke so I have not been checking my blood glucose levels for a while." Explanation: Glycosylated hemoglobin is hemoglobin into which glucose has been irreversibly incorporated. Because glucose entry into the red blood cell is not insulin dependent, the rate at which glucose becomes attached to the hemoglobin molecule depends on blood glucose; the level is an index of blood glucose levels over the previous 6 to 12 weeks. If the diabetic client is not monitoring his or her blood glucose, he or she could be having more periods of hyperglycemia and just is not aware of the need for insulin coverage.

A client who was exposed to hepatitis A at a local restaurant has recovered from the disease. At her annual physical, the client asks the health care provider if she should go to her health department and get the hepatitis A "shot." The best response by the health care provider, based on the concepts of adaptive immunity, would be:

"No, since having an active case, you have already developed antigens against hepatitis A." Explanation: The adaptive immune system consists of two groups of lymphocytes and their products, including antibodies. Whereas the cells of the innate immune system recognize structures shared by classes of microorganisms, the cells of the adaptive immune system are capable of recognizing numerous microbial and noninfectious substances and developing a unique specific immune response for each substance. Substances that elicit adaptive immune responses are called antigens. A memory of the substance is also developed so that a repeat exposure to the same microbe or agent produces a quicker and more vigorous response. The hepatitis A virus does not change from year to year. The vaccine does not damage the liver; however, the active hepatitis A disease can.

A client asks if pain threshold and pain tolerance are the same. The best response by the health care provider would be:

"Pain threshold is the point at which a stimulus is perceived as painful." Explanation: Pain threshold is closely associated with the point at which a nociceptive stimulus is perceived as painful. Pain tolerance relates more to the total pain experience; it is defined as the maximum intensity or duration of pain that a person is willing to endure before the he or she wants something done about the pain. Psychological, familial, cultural, and environmental factors significantly influence the amount of pain a person is willing to tolerate. The threshold for pain is fairly uniform from one person to another, whereas pain tolerance is extremely variable.

A client is exposed to second-hand smoke on a daily basis. The client asks if he is at risk for the development of cancer. The best response would be:

"Second-hand exposure does put you at risk." Explanation: Exposure to many carcinogens, such as those contained in cigarette smoke, is associated with a lifestyle risk for development of cancer. It is directly associated with lung and laryngeal cancer and has been linked with cancers of the mouth, nasal cavities, pharynx, esophagus, pancreas, liver, kidney, uterus, cervix, and bladder and myeloid leukemia. Not only is the smoker at risk but others passively exposed to cigarette smoke are at risk. Chewing tobacco increases the risk of cancers of the oral cavity and esophagus.

A client overheard the provider discussing the case and mentioning a "mutation in the TP53 gene." The client asks the nurse, "What does that mean?" Which response is the most appropriate for this client?

"The TP53 gene is a group of tumor suppressor genes associated with lung, breast and colon cancer." Explanation: Multiple tumor suppressor genes have been found that connect with various types of cancer. Of particular interest in this group is the TP53 gene, which is on the short arm of chromosome 17 and codes for the p53 protein. Mutations in the TP53 gene have been associated with lung, breast, and colon cancer. The TP53 gene also appears to initiate apoptosis in radiation- and chemotherapy-damaged tumor cells. The statement does not mean the gene was damaged in embryo and exposed to a toxin. Not all cancers result from an abnormality in some gene or chromosome. A monoclonal antibody is an antibody produced by a single clone of cells or cell line and consisting of identical antibody molecules. It is different from a mutation in a certain gene pool.

A college student has been called into the student health office because she tested positive for HIV on the enzyme-linked immunosorbent assay (ELISA). The student asks, "What is this Western blot assay going to tell you?" The best response by the health care provider is:

"The Western blot is a more sensitive assay that looks for the presence of antibodies to specific viral antigens." Explanation: The Western blot is a more sensitive assay than the EIA that looks for the presence of antibodies to specific viral antigens. In the case of a false-positive EIA result, the Western blot test can identify the person as uninfected. Technologic advances have led to new forms of testing, such as the oral test, home testing kits, and the new rapid blood test. Oral fluids contain antibodies to HIV. In the late 1990s, the FDA approved the OraSure test. The OraSure uses a cotton swab, which is inserted into the mouth for 2 minutes, placed in a transport container with preservative, and then sent to a laboratory for EIA and Western blot testing.

A client's primary care provider has ordered an oral glucose tolerance test (OGTT) as a screening measure for diabetes. Which instruction should the client be given?

"The lab tech will give you a sugar solution and then measure your blood sugar levels at specified intervals." Explanation: The OGTT measures the plasma glucose response to 75 g of concentrated glucose solution at selected intervals, usually 1 and 2 hours. A fasting blood glucose test requires 8 hours without food, and A1C measures glucose binding to hemoglobin. A casual blood glucose test is administered without regard for time or last meal.

A client was recently diagnosed with acquired immunodeficiency syndrome (AIDS) and started on antiviral therapy. The nurse evaluates the client's understanding of the antiviral drug when the client correctly makes which statement regarding the pharmacologic mechanism of action?

"The primary target of most antiviral compounds is viral RNA or DNA synthesis." Explanation: The primary target of antiviral compounds is viral RNA or DNA synthesis. Agents such as acyclovir, ganciclovir, vidarabine, and ribavirin mimic the nucleoside building blocks of RNA and DNA. During active viral replication, the nucleoside analogs inhibit the viral DNA polymerase, preventing duplication of the viral genome and spread of infectious viral progeny to other susceptible host cells.

A nurse educator is describing the way that cells involved in the inflammatory response find their way to the site of injury. Which description best reflects this physiologic mechanism?

"The process of chemotaxis is the process where cells wander through the tissue guided by secreted chemoattractants." Explanation: Chemotaxis is the dynamic and energy-directed process of directed cell migration. Once leukocytes exit the capillary, they wander through the tissue guided by a gradient of secreted chemoattractant, bacterial and cellular debris, and protein fragments generated from activation of the complement system. Phagocytosis is the engulfment of foreign or infective material. Margination is the first step of the cellular stage and is a separate process from chemotaxis, which is the third step in the cellular response. Tissues do not store inflammatory cells.

A client was unaware that intestinal flora are beneficial, stating, "I thought all bacteria were bad." Which is the nurse's most accurate response?

"The term 'mutualism' is applied to an interaction in which the microorganism and the host both derive benefits from the interaction." Explanation: The relationship between host (human) and bacteria that colonize the gut is a mutualistic relationship. The bacteria digest and synthesize nutrients that we cannot digest. With commensalism there is no net benefit or loss, such as is seen in a parasitic relationship; the host is separate from the bacteria.

The parents of a 2-year-old child ask the nurse why their toddler's wounds do not seem to heal as quickly as their older children. Which response by the nurse is most accurate?

"The younger child has an immature immune system with no experience with organisms." Explanation: Infants and young children have decreased immunologic function and are more commonly infected with virulent organisms. The younger the child, the more likely the immune system is not fully developed. The remaining options are not viable risk factors.

While studying for a pathophysiology exam on cells and tissue repair, a student asks, "What happens if, in the process of going through the cell cycle, a cell is damaged?" Which is the best response?

"There are built-in checkpoints in the cell cycle to allow for defects to be edited and repaired." Explanation: Within the cell cycle are checkpoints where pauses or arrests can be made if the specific events in the phases of the cell cycle have not been completed. There are also opportunities for ensuring the accuracy of DNA replication. These DNA damage checkpoints allow for any defects to be edited and repaired, thereby ensuring that each daughter cell receives a full complement of genetic information, identical to that of the parent cell.

A sixth-grade science teacher asks the students to explain the role of cilia in the lower respiratory tract. Which student response is best?

"These little hairs move germs trapped in mucus toward the throat so the body can cough them out." Explanation: Also in the lower respiratory tract, hair-like structures called cilia protrude through the epithelial cells. The synchronous action of the cilia moves many microbes trapped in the mucus toward the throat. The physiologic responses of coughing and sneezing further aid in their removal from the body. Chemical defenses against trapped microbes include the following: lysozyme, which is a hydrolytic enzyme capable of cleaving the walls of bacterial cells; complement, which binds and aggregates bacteria to increase their susceptibility to phagocytosis or disrupt their lipid membrane; and members of the collectin family of surfactant proteins in the respiratory tract. The best-defined function of the surfactants is their ability to opsonize pathogens, including bacteria and viruses, and to facilitate phagocytosis by innate immune cells such as macrophages.

A young child develops type 1A diabetes. The parents ask, "They tell us this is genetic. Does that mean our other children will get diabetes?" The best response by the health care provider would be:

"This autoimmune disorder causes destruction of the beta cells, placing your children at high risk of developing diabetes." Explanation: Type 1 diabetes is subdivided into two types: type 1A, immune-mediated diabetes, and type 1B, idiopathic diabetes. Type 1A diabetes is characterized by autoimmune destruction of beta cells. The other choices are not absolutely correct. The fact that type 1 diabetes is thought to result from an interaction between genetic and environmental factors led to research into methods directed at prevention and early control of the disease. These methods include the identification of genetically susceptible persons and early intervention in newly diagnosed persons with type 1 diabetes.

A client with cancer has been receiving amphotericin B intravenously. The client asks, "How will this help with my fungal disease?" The health care provider responds:

"This drug binds to ergosterol and forms holes in the cell membrane, killing the fungus." Explanation: The polyene family of antifungal compounds (e.g., amphotericin B, nystatin) preferentially binds to ergosterol and forms holes in the cell membrane, causing leakage of the fungal cell contents and, eventually, lysis of the cell. The imidazole class of drugs (e.g., fluconazole, itraconazole) inhibits the synthesis of ergosterol, thereby damaging the integrity of the fungal cytoplasmic membrane. Zidovudine, as well as non-nucleoside inhibitors, impairs the synthesis of the HIV-specific enzyme reverse transcriptase. This key enzyme is essential for viral replication.

Which statement would a nurse tell a client that best describes a lab finding of metaplasia?

"This is a change in cell form as it adapts to increased work demands or threats to survival." Explanation: Metaplasia is a change in the cell's form as it adapts to increased work demands or threats to survival. Hyperplasia is the change in the number of cells, and atrophy and hypertrophy are changes in a cell's size. Hypoxia or hypoxemia are decreases in oxygen.

A daughter of an older adult client tells the nurse that her mother is not able to handle stress like she used to. The best response by the nurse would be:

"This is an expected response." Explanation: The capacity to adapt to stress is influenced by age. Changes in an older adult's adaptation to stress result from age-related changes in renal function. There is a reported decline in ANS and cardiovascular responsiveness to stress associated with advancing age. Aging is also associated with impaired activation and proliferation of T and B lymphocytes, as well as poorer natural killer cell response to stimulatory cytokines.

The nurse educates a client with a suspected myocardial infarction (MI) about the blood test to evaluate troponin levels. The nurse knows the education has been successful when the client says:

"This test is specific to components found in my heart muscle." Explanation: Normally, during the noncontracted state of a cardiac or skeletal muscle cell, troponin covers the tropomyosin-binding sites on the actin filament and prevents formation of cross-bridges. During electrical stimulus, calcium binds with troponin to open the binding sites and allow actin-myosin cross-bridges to form. The presence of troponin I or troponin T (specialized troponins found in the heart muscle) in the blood indicates that cardiac muscle damage released troponin into the bloodstream. The damage could arise from MI or other conditions such as ischemia, sepsis, or renal failure. Troponin levels remain elevated for at least a week after cardiac injury. Lifestyle modification will not lower troponin levels.

A client has been diagnosed with bladder cancer and is meeting with the interdisciplinary team to determine a course of treatment. The client's disease is believed to be treatable with targeted cancer therapies. What should the client be taught about this form of cancer treatment?

"This treatment has far less of an effect on healthy cells than chemotherapy or radiotherapy." Explanation: Targeted cancer therapy uses drugs that selectively attack malignant cells while leaving normal cells unharmed. This stands in sharp contrast to radiation therapy and chemotherapy, which have profound effects on healthy body cells. Targeted therapies do not guarantee quick results and there is no promise of future prevention. Targeted therapies address the activity of cancer cells directly rather than influencing the immune system.

A client develops an immunodeficiency disorder after receiving chemotherapy for the treatment of lung cancer. The client asks the nurse if he was born with this deficiency. What is the nurse's best response?

"You have developed a secondary immunodeficiency disorder as a result of your chemotherapy." Explanation: Secondary immunodeficiency disorders develop later in life as a result of other pathophysiologic states such as malnutrition, disseminated cancers; infection of the cells of the immune system, most notably with human immunodeficiency virus, and treatment with immunosuppressant drugs, such as chemotherapeutic agents, corticosteroids, or transplant rejection medications.

Shortly after being diagnosed with HIV, a client has begun highly active antiretroviral therapy (HAART). The client asks, "My doctor tells me that my viral load is going down. What does that mean?" The nurse's best response is:

"Your HAART medications are working to slow the progression of the disease." Explanation: Antiviral therapies such as HAART are prescribed to slow the progression of AIDS and improve the overall quality of life and survival time of persons with HIV infection. Extension, not limitation, of the latent period is a goal. Minimizing transmission and preventing seroconversion are not normally achievable goals through drug therapy alone. The final 5% to 15% are long-term nonprogressors, who remain asymptomatic for 10 years or more after seroconversion, with stable CD4+ T-cell counts and low plasma HIV RNA levels.

A client who has been exercising says to the nurse, "My skin is so hot!" What is the nurse's best response?

"Your body is trying to lower your temperature." Explanation: After exercise, the body dilates blood vessels and shunts blood to the skin to help to decrease the temperature. Skin typically feels hot and sweaty. Evaporation of sweat from the skin surface contributes to loss of heat. As the heat is lost through the skin, the skin typically feels hot. The nurse should calm the client and explain that this is a normal response to exercise.

The parents of a newborn are relieved that their baby was born healthy, with the exception of a cleft lip that will be surgically corrected in 10 to 12 weeks. Which statement by the nurse to the parent's best conveys the probable cause of the newborn's cleft lip?

"Your newborn's cleft lip likely results from the interplay between environment and genes." Explanation: A cleft lip is considered to be a multifactorial disorder, in which both environment and genes contribute to the condition. It does not depend solely on Mendelian patterns of inheritance. A cleft lip is not known to result from teratogenic drugs.

A clinic health care provider has noticed an increase in the number of clients getting tested for Lyme disease. In addition to blood testing, what other manifestations will clients diagnosed with Lyme disease display? Select all that apply.

"bull's eye" red rash headaches joint pain and swelling Explanation: Many clients have no memory of being bitten by a tick, because the tick is less than 2 mm in size. About half (52%) of clients do not experience the classic bull's eye or erythemia migrans rash associated with Lyme disease. These clients have central nervous system and musculoskeletal manifestations, including headahces, fatigue, and joint pain and swelling. Diarrhea and nosebleeds do not occur in the client with Lyme disease.

Place the following in the sequence of the progression of HIV/AIDS on a cellular level.1)The HIV virus is uncoded.2)HIV attaches to the receptors on the CD4+ cell.3)DNA synthesis occurs by reverse transcription.4)Transcription of the DNA to form a single-strand messenger RNA.

2,1,3,4 Explanation: The first step involves the binding of the virus to the CD4+ T cell. The second step allows for the internalization of the virus. After attachment, the virus envelope peptides fuse to the CD4+ t-cell membrane. Fusion results in uncoding of the virus. The third step consists of DNA synthesis by reverse transcription. The fourth step is integration. The fifth step involves transcription of the double-stranded viral DNA to form a single-stranded messenger RNA.

How long is the half-life of the hormone aldosterone, which is only 15% protein bound?

25 minutes Explanation: Aldosterone, which is only 15% protein-bound, has a half-life of only 25 minutes. The higher the percentage of protein binding, the longer the half-life will be.

A client has experienced an acute inflammatory response with an elevation of white blood cells. The nurse is reviewing the client's most recent lab results to determine if the counts have returned to a normal range. Select the result that suggests the client is now within normal range.

4000 to 10,000 cells/μL Explanation: A normal value of white blood cells would be 4000 to 10,000 cells/μL. In acute inflammatory conditions, the white blood cell count commonly increases from 15,000 to 20,000 cells/μL. The other results are abnormal.

A nurse working in a cancer facility identifies which clients to be at high risk for developing cancer? Select all that apply.

45-year-old smoker of two packs per day 30-year-old who chews tobacco daily 50-year-old who drinks a six-pack five times a week 55-year-old who grills with charcoal three times a week Explanation: Exposure to many chemicals can increase a person's risk for cancer and this exposure is associated with lifestyle risk factors such as smoking, alcohol consumption, and dietary factors.

The physician suspects a client may have Klinefelter syndrome. To confirm the diagnosis, the chromosome pattern would identify:

47, XXY Explanation: Most males with Klinefelter syndrome have one extra X chromosome (47, XXY), and in rare cases, there may be one additional extra X chromosome (48, XXXY).

A mother who has one mutant allele on the X chromosome and one normal allele asks the nurse what the percentage is of passing it on to a daughter to be a carrier. The best response would be:

50% Explanation: The common pattern of inheritance is one in which an unaffected mother carries one normal and one mutant allele on the X chromosome. This means that she has a 50% chance of transmitting the defective gene to her sons, and her daughters have a 50% chance of being carrier of the mutant gene.

The psychologist is leading a group session with clients who have recently undergone a variety of stressful events. Which client will have the best adaptation?

50-year-old female who sees her new life as a widow as a challenge rather than a threat Explanation: Physiologic reserve, time, genetics, age, health status, nutrition, sleep-wake cycles, hardiness, and psychosocial factors influence a person's appraisal of a stressor and the coping mechanisms used to adapt to the new situation. The capacity to adapt is decreased at the extremes of age (89 years old). Gender-based differences in activation of the stress response may partially explain differences in susceptibility to diseases (females have lower activation of the sympathetic nervous system). Sleep disorders, especially insomnia and sleep deprivation, have been shown to alter immune function, and physical and psychological functioning. The ability to conceptualize stressors as a challenge rather than a threat is a characteristic of hardiness, which is correlated with positive health outcomes. Severe emotional stress (single parent) determines physiologic and psychological reserves and is a strong determinant of the ability to adapt.

A client states, "I heard that my healed wound tissue is stronger than my normal tissue. Is that true?" The nurse responds that roughly 3 months after a wound, the wound tensile strength is approximately what percentage of normal?

70% of normal Explanation: Tensile strength increases rapidly over the weeks and then slows, reaching a plateau of approximately 70% to 80% of the tensile strength of unwounded skin at the end of 3 months.

The nurse is reviewing assessment data and determines which client is at highest risk for developing type 2 diabetes?

A 45-year-old obese female with a sedentary lifestyle Explanation: The person most at risk for developing type 2 diabetes is the 45-year-old obese female with a sedentary lifestyle. Other risk factors include family history, over age 40, and history of gestational diabetes. The 60-year-old would have developed it before age 60, if there were additional risk factors. Diabetes and metabolic syndrome are due to pancreatic problems, not liver problems.

Which statement is true about recessive gene inheritance?

A carrier is a nonaffected person with a single copy of a recessive gene. Explanation: Autosomal recessive disorders are manifested only when both members of the gene pair are affected. Therefore, a carrier is a nonaffected person with a single copy of a recessive gene. Dominant genes have greater expression than recessive ones. Because of the presence of a normal X gene, female heterozygotes rarely experience the effects of a defective gene, whereas all males who receive the gene are typically affected. Variability of gene expression is called variable expressivity.

Which client is most likely to experience impaired (slow) wound healing?

A client with a diagnosis of type 1 diabetes and a history of poor blood sugar control Explanation: Specific disorders that slow wound healing include diabetes mellitus, peripheral artery disease, venous insufficiency, and nutritional disorders. Diabetes mellitus is a significant barrier to wound healing. Serial surgeries, chronic obstructive pulmonary disease, hypertension, and related medications are not directly linked to impaired wound healing.

The nurse is caring for several clients on the acute care floor. Which clients are at risk for the development of allergic responses to latex? Select all that apply.

A client with a history of atopic reactions A client with a history of food allergies A client with a history of delayed hypersensitivity reactions Explanation: While exposure to natural rubber latex is necessary for sensitization, other factors play a key role in the development of latex allergy. People with a history of atopic reactions, food allergies, and delayed hypersensitivity reactions are more likely to develop an allergic response to latex. A client with a history of otitis media or respiratory infections is no more likely to develop an allergy to latex than the general population.

The nurse is orienting a student to the oncology setting. Which characteristic does the student identify as one that is representative of a benign tumor?

A fibrous capsule Explanation: Because benign tumors have slow cell replication and growth, they develop a surrounding rim of compressed connective tissue called a fibrous capsule. Benign tumors encapsulate and are incapable of distant metastasis. Benign tumors have mature, differentiated cells but they have lost the ability to suppress cell proliferation.

Which outpatient is at a greater risk for developing Treponema pallidum, the cause of syphilis? Select all that apply.

A homeless adolescent female performing oral sex for money A male who frequents clubs catering to exotic dancers and sexual favors An older adult female living in a condominium who regularly has sex with three to four different men per week Explanation: Treponema pallidum is a sexually transmitted infection that is spread by direct physical contact. The Borrelia type of spirochete is spread from animals to humans through lice or tick bites. Leptospira spirochetes spread from animals to humans through contact with infected animal urine. Spirochetes are anaerobic; therefore, they would not invade the host through oxygen-filled aerobic lungs.

A biopsy of a client's liver has been taken because there is suspicion that his lung cancer may have metastasized. The results confirm that there are cancerous cells in the client's liver and the oncologist has estimated a high growth fraction in the sample. The nurse should draw what implication from this finding?

A large proportion of the cells in the sample are actively dividing. Explanation: The ratio of dividing cells to resting cells in a tissue mass is called the growth fraction. A high growth fraction represents a large proportion of growing cells, but does not necessarily mean that every cell is dividing. Stem cells would not be present in a sample from this location. Growth fraction does not imply a particular size of the cells.

A client is admitted for ketoacidosis. The client's breath is fruity and others state that the client has been acting "different." Initial blood glucose level is 642 mg/dL (35.63 mmol/L). Which rationale for prescribing an insulin drip (IV infusion) is most accurate?

A lower pH of the body fluids (ketoacidosis) reduces insulin binding. Explanation: The affinity of receptors for binding hormones is also affected by a number of conditions. For example, the pH of the body fluids plays an important role in the affinity of insulin receptors. In ketoacidosis, a lower pH reduces insulin binding. While lowering the blood glucose level, potassium can be shifted from the extracellular to intracellular compartment, resulting in hypokalemia. Neurologic status does not cause vasoconstriction of vessels or poor absorption of subcutaneous injections.

Which client may be experiencing the effects of neuropathic pain?

A man with pain secondary to his poorly controlled diabetes Explanation: Conditions that can lead to pain by causing damage to peripheral nerves in a wide area include diabetes mellitus, alcohol consumption, hypothyroidism, rash, and trauma. Fractures, wounds, and cancer pain do not typically have an etiology that is rooted in the neurologic system.

Which client would be an example of an individual experiencing cellular atrophy?

A middle-aged female experiencing menopause due to loss of estrogen stimulation Explanation: In women, the loss of estrogen stimulation during menopause results in atrophic changes in the reproductive organs. Compensatory hypertrophy is the enlargement of a remaining organ or tissue after a portion has been surgically removed or rendered inactive. For instance, if one kidney is removed, the remaining kidney enlarges to compensate for the loss. In hypertension, for example, the increased workload required to pump blood against an elevated arterial pressure results in a progressive increase in left ventricular muscle mass and need for coronary blood flow. The pregnant uterus undergoes both hypertrophy and hyperplasia as a result of estrogen stimulation.

The nurse is teaching a client infected with the flu about viruses. Which explanation would the nurse use to describe a viral infection?

A parasitic relationship between an organism and host is present. Explanation: Viruses are the smallest obligate intracellular pathogens. They have no organized cellular structures. Some viruses are enclosed within a lipoprotein envelope derived from the cytoplasmic membrane of the parasitized host cell. Enveloped viruses include members of the herpes virus group and paramyxoviruses (e.g., influenza and poxviruses).

Which client is experiencing the effects of a positive feedback mechanism?

A pregnant client whose oxytocin levels are rising during the second stage of labor. Explanation: The release of hormones during labor increases rather than mitigates a physiologic system. Specifically, uterine contraction stimulates the production of other relevant hormones that cause temporary instability, which culminates in birth. ADH counters the potential instability of insufficient hydration or blood pressure, much as shivering is an attempt to counter low temperature. WBCs are a response to the potential homeostatic instability of an infectious process.

A client has been diagnosed with a fever of unknown cause. The nurse recognizes this as:

A prolonged fever that does not have an identified source Explanation: A prolonged fever for which the cause is difficult to ascertain is often referred to as fever of unknown origin (FUO). FUO is defined as a temperature elevation of 38.3°C (101°F) or higher that is present for 3 weeks or longer. An FUO can occur in children or adults.

A client presents with an oral temperature of 101.7°F (38.7°C) and painful, swollen cervical lymph nodes. Laboratory results indicate neutrophilia with a shift to the left. Which diagnosis is most likely?

A severe bacterial infection Explanation: Fever and painful, palpable lymph nodes are nonspecific inflammatory conditions; leukocytosis is also common but is a particular hallmark of bacterial infection. Neutrophilia also indicates a bacterial infection, whereas increased levels of other leukocytes would indicate other etiologies. The shift to the left—the presence of many immature neutrophils—indicates that the infection is severe, because the demand for neutrophils exceeds the supply of mature cells.

Which of the following is the best definition of a haplotype?

A small variation in gene sequence Explanation: The human genome sequence is almost exactly the same in all people. There is a small variation of about 0.01% in gene sequence that is termed a haplotype; this is thought to account for the individual differences in physical traits, behaviors, and disease susceptibility.

A client with small cell carcinoma lung cancer may secrete an excess of which hormone, causing an ectopic form of Cushing syndrome due to a nonpituitary tumor?

ACTH Explanation: The third form (of Cushing syndrome) is ectopic Cushing syndrome, caused by a nonpituitary ACTH-secreting tumor. Certain extra pituitary malignant tumors such as small cell carcinoma of the lung may secrete ACTH or, rarely, CRH that can produce Cushing syndrome. The adrenal sex hormone dehydroepiandrosterone (DHEA) contributes to the pubertal growth of body hair, particularly pubic and axillary hair in women. Thyroid-stimulating hormone (TSH) levels are used to differentiate between primary and secondary thyroid disorders. Although secretion of growth hormone (GH) has diurnal variations over a 24-hour period, with nocturnal sleep bursts occurring 1 to 4 hours after onset of sleep, it is unrelated to ACTH and/or CRH secretion.

The health care provider is reviewing diurnal variation pattern in adrenocorticotropic (ACTH) levels. Select the typical diurnal variation pattern in adrenocorticotropic (ACTH) levels.

ACTH peaks in the morning and declines throughout the day. Explanation: ACTH levels have diurnal variation in which they reach their peak in the early morning (around 6 to 8 AM) and decline as the day progresses related to rhythmic activity of the CNS. The diurnal pattern is reversed in people who work during the night and sleep during the day. The rhythm also may be changed by physical and psychological stresses, endogenous depression, and liver disease or other conditions that affect cortisol metabolism.

The school nurse is preparing a lecture on HIV/AIDS for a health class of high school students. The nurse would know to include what information about the transmission of AIDS in her lecture? Select all that apply.

AIDS is transmitted through sexual contact. AIDS is transmitted through blood-to-blood contact. AIDS is transmitted from the mother to her unborn baby. Explanation: HIV is transmitted from one person to another through sexual contact, blood-to-blood contact, or perinatally. It is not transmitted through casual contact. Several studies involving more than 1000 uninfected, nonsexual household contacts with persons with HIV infection (including siblings, parents, and children) have shown no evidence of casual transmission. HIV is not spread by mosquitoes or other insect vectors.

The Krebs cycle provides a common pathway for the metabolism of nutrients by the body. The Krebs cycle forms two pyruvate molecules. Each of the two pyruvate molecules formed in the cytoplasm from one molecule of glucose yields another molecule of:

ATP Explanation: Each of the two pyruvate molecules formed in the cytoplasm from one molecule of glucose yields another molecule of ATP, which is a special carrier for cellular energy. FAD, or flavin adenine dinucleotide, is a coenzyme of protein metabolism that accepts electrons and is reduced. NADH + H+ is an end product of glycolysis. The electron transport chain oxidizes NADH + H+ and FADH2 and donates the electrons to oxygen, which is reduced to water.

A nurse who is testing a client's response to passive movement of the fingers with the client's eyes closed notes that the client cannot accurately identify on which side the movement occurred or in what position the finger was placed. What is an appropriate interpretation of this result?

Abnormal discrimination pathway function Explanation: The discriminative dorsal column-medial lemniscus pathway is able to sense fine touch and discriminate between two points as close as 5 mm. This pathway also mediates sense of position. Loss of this pathway means only that the anterolateral pathway is functioning and is unable to make fine distinctions in touch proprioception and points. The test does not test reflexes or temperature.

Cyclic adenosine monophosphate (cAMP) performs which role in the functioning of the endocrine system?

Acting as a second messenger to mediate hormone action on target cells Explanation: cAMP is one of the most common second messengers, whose role is to generate an intracellular signal in response to cell surface receptor activation by a hormone. cAMP does not mediate hormone synthesis, act as a receptor itself, or inactivate hormones.

The nurse is assessing a client with thyrotoxicosis and the nurse is explaining how the thyroid gland is stimulated to release thyroid hormones. The nurse should describe what process?

Action of releasing hormones from hypothalamus Explanation: The synthesis and release of anterior pituitary hormones are largely regulated by the action of releasing or inhibiting hormones from the hypothalamus, which is the coordinating center of the brain for endocrine activity such as thyroid activity. There are no direct innervations for hormone release from the thyroid gland, and homeostatic receptors do not exist. Steady-state release of hormones does not occur.

The nurse is administering a measles, mumps, and rubella (MMR) vaccination to a pediatric client. When the nurse explains immunity to the mother, which type of immunity will she explain that this vaccination provides?

Active artificial immunity Explanation: Active immunity is acquired when the host mounts an immune response to an antigen either through the process of vaccination (artificial means) or from environmental exposure (natural means). It is called active immunity because it requires the host's own immune system to develop an immunological response. The process by which active immunity is acquired through the administration of a vaccine is termed immunization. An acquired immune response can improve on repeated exposures to an injected antigen (booster vaccines) or a natural infection.

The client is hospitalized for 4 days with an acute myocardial infarction. After a coronary artery stent is placed, the client is discharged to home in stable condition. This would be an example of which pattern of stress?

Acute time limited Explanation: Stressors can assume a number of patterns in relation to time. They may be classified as acute time-limited, chronic intermittent, or chronic sustained. An acute time-limited stressor is one that occurs over a short time and does not recur. A chronic intermittent stressor is one to which a person is chronically exposed intermittently; chronic sustained is continuous. PTSD is characterized by a severe stress response secondary to experiencing previous trauma.

A client taking oral corticosteroids is exposed to varicella. The client has never had the disease and has not received the varicella vaccine. What intervention should the nurse anticipate in order to prevent the client from developing varicella?

Administer varicella zoster immune globulin (VZIG). Explanation: Immunotherapy involves supplementing or stimulating the host's immune response so that the spread of a pathogen is limited or reversed. Several products are available for this purpose, including intravenous immunoglobulin (IVIG). IVIG is a pooled preparation of antibodies obtained from normal, healthy immune human donors that is infused as an intravenous solution. Hyperimmune immunoglobulin preparations, which are also commercially available, contain high titers of antibodies against specific pathogens, including hepatitis B virus, cytomegalovirus, rabies, and varicella-zoster virus.

Hormones exert their action by interacting with which type of receptors?

Affinity Explanation: Target cell response varies with the number and affinity of the relevant receptors. The structure variations allow target cells to respond to one hormone and not to others. Cell membrane permeability can affect transmission; ion transport allows hormones to enter the cell. Hormone receptors are complex molecular structures (usually proteins not fats like cholestrol) that are located either on the cell surface or inside target cells. Antibodies may destroy or block the receptor proteins.

A nurse is teaching a group of clients about causes/risk factors for cancer, which include which factors? Select all that apply.

Age Environment Genetics Heredity Explanation: The cause or causes of cancer can be viewed from two perspectives: (1) the genetic and molecular mechanisms that characterize the transformation of normal cells into cancer cells and (2) the external and more contextual factors such as age, heredity, and environmental agents that contribute to its development and progression. Financial status does not influence risk factors.

In reference to infectious disease, a client cannot be a/an:

Agent Explanation: Viruses are incapable of replication outside of a living cell. They must penetrate a susceptible living cell and use the biosynthetic structure of the cell. A virus is an agent. Not every viral agent causes lysis and death of the host cell during the course of replication. Some viruses enter the host cell and insert their genome into the host cell chromosome, which acts as a reservoir; it remains there in a latent, nonreplicating state for long periods without causing disease.

The mother is heterozygous for blue eyes, a recessive trait. The father is homozygous for brown eyes, a dominant trait. What color eyes will their four children have?

All will have brown eyes. Explanation: A heterozygote with a dominant and a recessive allele will have the dominant phenotype. In Mary's case this will manifest itself in her brown eyes. A homozygote with two dominant alleles will have the dominant phenotype, so John also has brown eyes. To have blue eyes, the children would have to inherit two alleles for blue eyes. Because they will inherit, at most, one recessive allele for blue eyes, the children's eyes will be brown.

A client has been diagnosed with a genetic disorder that affects his voltage-gated sodium channels, increasing his risk of sudden cardiac death. In a healthy individual, what role do ion channels perform in the body?

Allow for membrane potentials to be established in excitable cells for rapid ion movement Explanation: Membrane potentials occur at the level of the cell membrane; ion channels allow for the passage of charged particles in and out of the excitable cell. Membrane potentials do result from the binding of sodium and potassium ions, both of which have a positive charge. Gated channels open and close in response to specific stimuil. Blood cells do not participate directly in the establishment of membrane potential in general, or the role of ion channels in particular. Ion channels do not respond to lipophilic or hydrophilic substances.

The nurse is performing an assessment for a client with Cushing syndrome and observes a "buffalo hump" on the back, a moon face, and a protruding abdomen. What does the nurse understand contributes to the distribution of fat in these areas?

Altered fat metabolism Explanation: The major manifestations of Cushing syndrome represent an exaggeration of the many actions of cortisol. Altered fat metabolism causes a peculiar deposition of fat characterized by a protruding abdomen, subclaviclar fat pads or "buffalo hump" on the back, and a round, plethoric "moon face."

Select the category of hormones that include norepinephrine and epinephrine.

Amines and amino acids Explanation: Hormones can be divided into three categories: (1) amines and amino acids; (2) peptides, polypeptides, proteins, and glycoproteins; and (3) steroids. The amines include norepinephrine and epinephrine. The second category, the peptides, includes polypeptides, proteins, and glycoproteins. The third category consists of the steroid hormones, which are derivatives of cholesterol.

A sonogram on a woman in the first trimester of her pregnancy shows abnormal fetal findings. The nurse prepares to provide the woman with information concerning which diagnostic procedure?

Amniocentesis Explanation: Amniocentesis is useful in women with elevated risk on first trimester screen or quad screen; abnormal fetal findings on sonogram; or in parents who are carriers or with a strong family history of an inherited disease. Chorionic villus sampling is an invasive diagnostic procedure that obtains tissue that can be used for fetal chromosome studies, DNA analysis and biochemical studies. Percutaneous umbilical cord blood sampling (PUBS) is used for prenatal diagnosis of hemoglobinopathies, coagulation disorders, metabolic and cytogenetic disorders, and immunodeficiencies. Biochemical analyses can be used to detect abnormal levels of AFP and abnormal biochemical products in the maternal blood and in specimens of amniotic fluid and fetal blood.

Which individual displays the precursors to acromegaly?

An adult with an excess of growth hormone due to an adenoma Explanation: When growth hormone (GH) excess occurs in adulthood or after the epiphyses of the long bones have fused, it causes a condition called acromegaly, which represents an exaggerated growth of the ends of the extremities.

The ingestion of a Shiga toxin produced by Escherichia coli O157:H7 can cause a sometimes fatal illness called hemolytic-uremic syndrome (HUS). Select the client at highest risk.

An individual who consumed undercooked hamburger during a backyard barbecue Explanation: HUS occurs primarily through the ingestion of undercooked hamburger meat or unpasteurized fruit juices contaminated with Shiga toxin. HUS is characterized by vascular endothelial damage, acute renal failure, and thrombocytopenia. There is no known connection among human fecal and urine contamination and Shiga toxin.

Which condition is an example of wound healing by secondary intention?

An infected burn of the arm Explanation: A sutured surgical incision is an example of healing by primary intention. Larger wounds (e.g., burns and large surface wounds) that have a greater loss of tissue and contamination heal by secondary intention. Healing by secondary intention is slower than healing by primary intention and results in the formation of larger amounts of scar tissue.

The nurse is evaluating the wounds of four clients. Select the client most likely to be treated with hyperbarically delivered oxygen.

An infected foot wound on a 45-year-old client with peripheral vascular disease Explanation: Hyperbaric oxygen is currently reserved for the treatment of problem wounds in which hypoxia and infection interfere with healing (e.g., an infected foot wound on a 45-year-old client with peripheral vascular disease). The other clients do not have any complications in the healing process.

Which client would be considered a good candidate for hyperbaric oxygen therapy?

An older adult with history of diabetes and intermittent claudication (poor circulation in the legs) who received a laceration while on the lawnmower Explanation: Hyperbaric oxygen therapy delivers 100% oxygen at two to three times the normal atmospheric pressure at sea level. The goal of hyperbaric oxygen therapy is to increase oxygen delivery to tissues by increasing the partial pressure of oxygen dissolved in the plasma. Hyperbaric oxygen is currently reserved for the treatment of problem wounds in which hypoxia and infection interfere with healing.

A client experiences burning muscle pain at the peak of high intensity physical training for an athletic competition. What is the best explanation for the cause of this occurrence?

Anaerobic glycolysis creates pyruvate, which converts to lactic acid. During aerobic activity, glycolysis occurs in the citric acid cycle and mitochondria oxidize electrons with an increase in ATP. However, during intense activity, the energy need may outstrip the supply of oxygen. Anaerobic glycolysis continues to supply cells with ATP when oxygen is not present. Fatty acids are oxidized to enter the citric acid cycle.

The nurse is studying numeric disorders involving autosomes. Which term refers to having an abnormal number of chromosomes?

Aneuploidy Explanation: Having an abnormal number of chromosomes is referred to as aneuploidy. When a person inherits a dominant mutant gene, but fails to express it, the trait is described as having reduced penetrance. In mitochondrial gene disorders, some cells receive a mixture of normal and mutant DNA, known as heteroplasmy. Occasionally, mitotic errors in early development give rise to two or more cell lines characterized by distinctive karyotypes, a condition referred to as mosaicism.

When discussing luteinizing hormone and follicle-stimulating hormone with students, the instructor will emphasize that these hormones are under the control of:

Anterior pituitary gland Explanation: The pituitary gland has been called the master gland because its hormones control the functions of many target glands and cells. The anterior pituitary gland or adenohypophysis contains five cell types: (1) thyrotrophs, which produce thyrotropin, also called TSH; (2) corticotrophs, which produce corticotropin, also called ACTH; (3) gonadotrophs, which produce the gonadotropins, LH, and FSH; (4) somatotrophs, which produce GH; and (5) lactotrophs, which produce prolactin.

Canada's Centre for Emergency Preparedness and Response is vigilantly keeping watch for terrorist events involving biologic agents. Of those on the list, which is of highest concern?

Anthrax Explanation: Anthrax is an ancient disease caused by the cutaneous inoculation, inhalation, or ingestion of the spores of Bacillus anthracis. Due to the ease of transmission and the speed of infection, it is of greatest concern. Parvovirus only infects canines. AIDS is not easily spread (via fluids and open wounds), and prion disease spreads through the consumption of brain material.

The cells that mediate humoral immunity do so because they are capable of producing:

Antibodies Explanation: B lymphocytes (B cells) are the only cells capable of producing antibodies; therefore, they are the cells that mediate humoral immunity. B lymphocytes (B cells) differentiate into plasma cells that produce the needed immunoglobulins (antibodies) for responding to that particular antigen. Stem cells are undifferentiated cells. T helper cells differentiate into cytotoxic and other T-cell forms.

A client has developed a tumor of the posterior pituitary gland. The client is at risk for problems with secretions of:

Antidiuretic hormone (ADH) and oxytocin Explanation: The posterior pituitary secretes ADH and oxytocin/vasopressin, while the anterior pituitary secretes the hormones listed in the other choices.

The nurse is caring for a client with an infection of Candida albicans. Which type of antimicrobial agent will the nurse plan to administer to effectively treat this disease?

Antifungal Explanation: The yeasts and molds such as organism Candida albicans are fungi that will be effectively treated with antifungal medications. Because Candida is part of the normal body flora, it usually appears as an infection only when the client's normal immune system is compromised, such as when taking broad spectrum antibiotics.

A client tells the nurse about taking vitamin E and vitamin C daily to prevent the uncontrolled formation of reactive oxygen species (ROS). Which term describes the client's rationale for taking these supplements?

Antioxidants Explanation: Antioxidants are natural and synthetic molecules that inhibit the reactions of ROS with biological structures or that prevent the uncontrolled formation of ROS. Unpaired electrons are free radicals and ROS and do not prevent these reactions. Oxidative stress occurs when the generation of ROS exceeds the ability of the body to neutralize and eliminate ROS.

A client reports general malaise and has a temperature is 103.8°F (39.9°C). What is the rationale for administering a prescribed aspirin, an antipyretic, to this client?

Antipyretics protect vulnerable organs, such as the brain, from extreme temperature elevation. Explanation: There is little research to support the belief that fever is harmful unless the temperature rises to extreme levels; it has been shown that small elevations in temperature, such as those that occur with fever, enhance immune function. Antipyretics are effective in lowering core body temperature. They alleviate discomforts of fever and protect vulnerable organs, such as the brain, from extreme elevations in body temperature.

The obstetric nurse explains to the client that when she stops breast-feeding, her breast tissue will reduce in size. The nurse understands that this regression is due to which physiologic process?

Apoptosis Explanation: Apoptosis is thought to be responsible for several normal physiologic processes, including hormone-dependent involution of tissues (e.g., the regression of breast tissue after weaning from breast-feeding). Telomeres allow the end of the DNA molecule to be replicated completely.

What concept will the nurse need to explain to a client with cancer about the purpose and action of chemotherapy?

Apoptosis Explanation: Cancer is a condition in which cells experience dysregulated apoptosis as the cells fail to age and die. Chemotherapy stimulates these cells to increase their rate of death. Pinocytosis is when a cell engulfs small solid or liquid substances from the surrounding environment. Osmosis is fluid moving across a cell membrane and is not directly related to chemotherapeutic impact on cells. Depolarization has to do with electrical impulse transmission and is not directly correlated with chemotherapy.

A dental assistant comes to the clinic with reports of a rash on her hands that feel like they are coming from the gloves being worn. The nurse observes both hands with crusted and thickened areas. What type of education does the nurse anticipate providing to the client?

Apply a topical corticosteroid and avoid further contact with latex-containing products. Explanation: Type IV hypersensitivity reactions to latex gloves are the most common form of latex allergy seen. In this form of allergy, people usually develop a contact dermatitis to one of the chemical additives rather than the latex proteins within 48 to 96 hours of exposure. The contact dermatitis often affects the dorsum of the hands and is characterized by a vesicular, pruritic rash. When glove contact is continued, the area becomes crusted and thickened. Treatment is with topical corticosteroids during the acute phase and avoidance of all latex-containing products.

A client is taking a vacation in a foreign country. The nurse teaches the client about giardiasis, a common traveler's infection. Which statement should be included in the teaching plan?

Avoid eating food and drinking beverages that might be contaminated. Explanation: Protozoa are unicellular animals with a complete complement of eukaryotic cellular machinery, including a well-defined nucleus and organelles. Most are saprophytes, but a few have adapted to the accommodations of the human environment and produce a variety of diseases, including malaria, amebic dysentery, and giardiasis. Protozoan infections can be passed directly from host to host as through sexual contact, indirectly through contaminated water or food, or by way of an arthropod vector.

Which statements are true regarding humoral immunity? Select all that apply.

B lymphocytes (B cells) are vital to the functioning of this type of immunity. Two types of responses (primary and secondary) occur. The memory response occurs on subsequent exposures to the antigen. Explanation: There are two types of adaptive immune responses, humoral and cell-mediated immunity, that function to eliminate different types of microbes. Humoral immunity is mediated by the B lymphocytes (B cells). The B cells differentiate into antibody-secreting plasma cells. The circulating antibodies then interact with and destroy the microbes that are present in the blood or mucosal surfaces. Cell-mediated, or cellular, immunity is mediated by the cytotoxic T lymphocytes (T cells) and functions in the elimination of intracellular pathogens (e.g., viruses) and in rejection of foreign tissue grafts. Two types of responses occur in the development of humoral immunity: a primary and secondary response. A primary immune response occurs when the antigen is first introduced into the body. The secondary, or memory response, occurs on second or subsequent exposures to the antigen.

A client whose mother and grandmother both have had cancer is at a higher risk for also developing the disease. Which tumor suppressor genes are associated with genetic susceptibility to breast cancer? Select all that apply.

BRCA1 BRCA2 Explanation: Two tumor suppressor genes called BRCA1 and BRCA2 have been identified in genetic susceptibility to breast and ovarian cancer.

Sputum samples from a client with pneumonia contain an infective agent that has a peptidoglycan cell wall, expresses endotoxins, replicates readily in broth and on agar, grows in clusters, has pili, and does not stain when exposed to crystal violet. This client's pneumonia has what etiology?

Bacterial Explanation: Although chlamydiae, viruses, and mycoplasmas all can cause pneumonia, only bacteria have all of these characteristics. Chlamydiae and viruses are obligate intracellular organisms and therefore would grow only in cell culture, and mycoplasmas lack the peptidoglycan cell wall typical of bacteria.

A health care researcher has identified the gene of interest in a particular genetic disorder as well as the gene's location, Xq97. Where would one find a gene named Xq97?

Band 9, region 7 of the long arm of the X chromosome Explanation: In gene names, the first letter stands for the chromosome. The second indicates the arm of the chromosome, p (short) or q (long). The first numeral indicates the band, and the second one indicates the region within that band.

Which statement about opportunistic pathogen infection in AIDS is true?

Becoming infected with an opportunistic pathogen is one requirement for the diagnosis of AIDS. Explanation: The diagnosis of AIDS is confirmed when a client's CD4+ count falls below 200 cells/μL or acquires an AIDS-defining illness (opportunistic disease). Opportunistic infections can infect healthy people, but they usually infect the immunocompromised, and they are serious. HIV attacks T cells.

A client is to have a serum thyroxine and thyroid stimulating laboratory test performed to assess the baseline status of the hypothalamic-pituitary target cell hormones. When educating the client about the laboratory tests, when would the the nurse inform him the test should be obtained?

Before 0800 Explanation: The assessment of hypothalamic-pituitary function has been made possible by many newly developed imaging and radioimmunoassay methods. Assessment of the baseline status of the hypothalamic-pituitary target cell hormones involves measuring the following laboratory specimens (ideally obtained before 0800): serum cortisol, serum prolactin, serum thyroxine and TSH, serum testosterone and estrogen and serum LH/FSH, serum GH, and plasma and urine osmolality.

When conducting a health assessment that focuses on the pain experienced by an older client diagnosed with early dementia, the nurse will pay particular attention to which of the following?

Behavioral signs of pain demonstrated by the client Explanation: The assessment of pain in older adults can range from relatively simple in a well-informed, alert, cognitively intact person with pain from a single source and no comorbidities to extraordinarily difficult in a confused person. When possible, a person's report of pain is the gold standard, but behavioral signs of pain should also be considered. This is especially true when the client's cognitive function is impaired. While the other options should be considered, the client's nonverbal behaviors should be of particular interest to the nurse.

To prevent the accumulation of hormones in our bodies, the hormones are constantly being metabolized and excreted. Where are adrenal and gonadal steroid hormones excreted?

Bile and urine Explanation: Unbound adrenal and gonadal steroid hormones are conjugated in the liver, which renders them inactive, and then excreted in the bile or urine. Adrenal and gonadal steroid hormones are not excreted in the feces, cell metabolites, or the lungs.

Which client/disease process would benefit from surgery to relieve severe, intractable pain?

Block transmission of phantom limb pain Explanation: Surgery for severe, intractable pain of peripheral or central origin has met with some success. It can be used to remove the cause or block the transmission of intractable pain from phantom limb pain, severe neuralgia, inoperable cancer of certain types, and causalgia.

The nurse and nursing student are caring for a client undergoing a severe stressor with release of epinephrine into the bloodstream. The nurse teaches the student that epinephrine will cause which effect on blood glucose levels?

Blood glucose will elevate. Explanation: Epinephrine, a catecholamine, helps to maintain blood glucose levels during periods of stress. Epinephrine causes glycogenolysis in the liver, thus causing large quantities of glucose to be released into the blood.

A client with a complete tear of the rotator cuff in the right shoulder was given the choice between surgery and stem cell transplant. The client chose to try the stem cell injection. The client asked the health care provider, "How does this work on my shoulder?" The best response would be:

Bone marrow stem cells have been shown to generate cartilage, bone, and muscle when injected directly into the injured site. Whether adult stem cells have a differentiation capacity similar to that of embryonic stem cells remains the subject of current debate and research. Thus far, bone marrow stem cells have been shown to have very broad differentiation capabilities, being able to generate not only blood cells but also fat, cartilage, bone, endothelial, and muscle cells.

A child with cystic fibrosis (CF) asks the nurse why he has this disease, but his parents are perfectly healthy. The nurse explains:

Both parents are carriers and have a recessive genotype with alleles for CF. Explanation: Both multifactorial inheritance and polygenic inheritance involve multiple genes at different loci. Multifactorial is different because it involves the effects of the environment on genes also. Unlike Mendel law, multifactorial inheritance and polygenic inheritance are unpredictable. Homozygous pairing of alleles is characteristic of recessive traits. The gene for the genetic disorder cystic fibrosis is recessive. Therefore, only persons with a genotype having two alleles for cystic fibrosis have the disease. In most cases, neither parent manifests the disease; however, both must be carriers.

Genetic screening may be indicated for individuals who have a family history of which neoplasm?

Breast cancer Explanation: The hereditary component of cancer is well identified, although it is implicated in varying degrees in different types of cancer. Breast cancer has an identified genetic component, and screening is often recommended for women with a family history of breast cancer.

A client with acromegaly comes to the clinic and informs the nurse that she is having a productive cough and a low grade fever. This is the client's fourth visit in 1 year for the same problem. What condition does the nurse understand results from this client's enlarged cartilaginous structures?

Bronchitis Explanation: The cartilaginous structures in the larynx and respiratory tract become enlarged, resulting in a deepening of the voice and tendency to develop bronchitis.

Nonshivering thermogenesis occurs in which of the following to help the newborn infant fight hypothermia?

Brown fat Explanation: Nonshivering thermogenesis occurs in the liver, brown fat tissue, and the brain and helps the newborn infant fight hypothermia.

The nurse on the pediatric unit is implementing distraction strategies for a child who is experiencing pain. Which strategies would be best for the nurse to implement? Select all that apply.

Bubbles Music Television Games Explanation: Distraction helps children of any age divert their attention away from pain and onto other activities. Common diversions include bubbles, music, television, conversation, and games.

A client is being treated for stomach cancer. The client is in considerable and constant pain, and the family is asking why. How does soft tissue cancer cause pain?

By compressing and eroding blood vessels, causing ulceration and necrosis, along with frank bleeding and sometimes hemorrhage. Explanation: The interruption of tissue integrity is responsible for the majority of pain in cancer. The physical compression is the cause of pain in bone cancer but not usually in soft tissue. There is no rearrangement of neurons that results in pain; rather, it results in loss of sensation. If the pain were to be caused by the flow of food into the stomach, it would only appear following a meal.

The nurse in the oncology unit has just admitted a client with metastatic cancer. The client asks how cancer moves from one place to another in the body. What would the nurse answer?

Cancer cells enter the body's lymph system and thereby spread to other parts of the body. Explanation: Metastasis occurs by way of the lymph channels (i.e., lymphatic spread) and the blood vessels (i.e., hematogenic spread). In many types of cancer, the first evidence of disseminated disease is the presence of tumor cells in the lymph nodes that drain the tumor area. When metastasis occurs by way of the lymphatic channels, the tumor cells lodge first in the initial lymph node that receives drainage from the tumor site. Once in this lymph node, the cells may die because of the lack of a proper environment, grow into a discernible mass, or remain dormant for unknown reasons. If they survive and grow, the cancer cells may spread from more distant lymph nodes to the thoracic duct and then gain access to the blood vasculature. Because cancer cells have the ability to shed themselves from the original tumor, they are often found floating in the body fluids around the tumor. Cancer cells are not moved from one place to another by transporter cells. Cancer cells do not form a chain to grow to the new place in the body to form a new tumor.

What will the nurse teach a client with trigeminal neuralgia about the condition?

Carbamazepine is a first-line treatment. Explanation: Trigeminal neuralgia is a condition in which clients experience brief, severe, repetitive lightninglike or throbbing pain along the distribution of one or more of the branches of the fifth cranial nerve. Clients should be taught to avoid triggers when possible, but although eating may be a trigger, it is necessary for the client to maintain nutrition. Social isolation is a possible problem arising from the condition. Time alone does not contribute significantly to improvement of the condition. Although treatment rarely provides total relief, trigeminal neuralgia can be controlled with carbamazepine and surgical release of vessels, nerve roots, or scar tissue. If other treatments are ineffective, partial destruction of the nerve branches with heat, balloon compression, or glycerol injection may be performed. Newer therapies to be considered are botulinum toxin injection and gamma radiation with stereotactic surgery.

Marfan syndrome, which is a connective tissue disorder, affects several organ systems. The most life-threatening aspects of the disorder are caused by defects in which system?

Cardiovascular Explanation: Marfan syndrome affects several organ systems, including the cardiovascular, skeletal, and ocular systems. The most life-threatening aspects of the disorder are the cardiovascular defects, which include mitral valve prolapse, progressive dilation of the aortic valve ring, and weakness of the aorta and other arteries.

A nurse is educating a parent group on childhood infectious diseases and various modes of pathogen transmission. Which topics should be included in the teaching plan? Select all that apply.

Caution parents about children playing barefoot on dirt. Avoid giving children unpasteurized milk or dairy products. Instruct children to avoid sharing grooming articles such as combs and brushes with friends. Explanation: Helminths in the dirt seek entry through the skin of the foot and into the blood circulation, where they are carried to the lungs, coughed up into the mouth, and swallowed into the GI tract, causing serious infections. C. burnetii is a Gram-negative intracellular organism that infects a variety of animals, including cattle, sheep, and goats. The organism produces a highly resistant sporelike stage that is transmitted to humans when contaminated animal tissue is aerosolized (e.g., during meat processing) or by ingestion of contaminated milk. The most prominent human ectoparasites are mites (scabies), chiggers, lice (head, body, and pubic), and fleas. Transmission of ectoparasites occurs directly by contact with immature or mature forms of the arthropod or its eggs found on the infested host or the host's clothing, bedding, or grooming articles (such as combs and brushes).

What happens when a cell is confronted with a decrease in work demands?

Cell becomes smaller. Explanation: As a cell's work demands decrease, the size of the cell decreases or atrophies. The size of the cell does not increase, but energy expenditure decreases. The cell does not die, but it adapts.

Which process associated with cellular injury is most likely to be reversible?

Cell damage resulting from accumulation of fat in the cytoplasm Explanation: Intracellular accumulation of fat leads to serious cell damage, but this is a potentially reversible effect. Ionizing radiation and damage from free radicals are more likely to be permanent, whereas apoptosis is defined as the permanent removal of injured and aged cells.

The cell surface is surrounded by a cell coat (glycocalyx.). Select the most important function of the cell coat.

Cell-to-cell recognition and adhesion The cell coat participates in cell-to-cell recognition and adhesion, and it contains tissue transplant antigens. Ribosomes serve as sites for protein synthesis in the cell; the Golgi apparatus modifies materials synthesized in the ER. DNA provides the information necessary for the synthesis of the various proteins.

A client with diabetes mellitus has sudden onset of slurred speech, incoordination, and cool, clammy skin. What will the nurse do first?

Check blood glucose Explanation: Hypoglycemia is a medical emergency. Because other conditions can also cause these symptoms, the blood glucose level should be checked first. Then glucose should be given. The client should follow this with a protein and complex carbohydrate to avoid a recurrence of hypoglycemia. Blood glucose should be tested about 15 minutes after the glucose is given to monitor progress.

A 51-year-old client has been diagnosed with stage IV breast cancer with lung metastases. The oncologist sits down with the client/family to explain treatment options. The nurse knows that which treatment option will be discussed for her cancers?

Chemotherapy Explanation: One of the advantages of chemotherapy is that, unlike surgery and radiation, it is able to treat cancer both at the primary site and at sites of metastasis. Hormone therapy is also able to exert therapeutic effects at a more systemic level, but to a lesser degree than chemotherapy.

A nurse is providing education on treatment options to the parents of a child with cancer. The nurse should include which of the following as the most widely used treatment modality for childhood cancers?

Chemotherapy Explanation: The nurse should include that the most widely used treatment modality for childhood cancers is chemotherapy.

Which child has the highest risk of experiencing a wound complication?

Child recently been diagnosed with type 2 diabetes Explanation: Children with certain comorbidities such as diabetes and malabsorption problems will be at higher risk of wound complications. Of course, secondary infections can occur after antibiotic courses, but this is not the primary cause of wound complications.

What can the nurse assume about a child's behavior when faced with the need to repeat a painful procedure?

Children act to avoid pain based on their memory of past painful events. Explanation: Children do feel pain and have been shown to reliably and accurately report pain. They also remember pain. This is evidenced in studies of children with cancer, whose distress during painful procedures increases over time without intervention, and in neonates in intensive care units, who demonstrate protective withdrawal responses to a heel stick after repeated episodes. The other options may not necessarily be true of most children.

A woman gave birth to a small infant with a malformed skull. The infant grows slowly and shows signs of substantial cognitive and intellectual deficits. The child also has facial abnormalities that become more striking as it develops. What might the nurse expect to find in the mother's pregnancy history?

Chronic alcohol use Explanation: The infant's signs and symptoms are characteristic of fetal alcohol syndrome. Folic acid deficiency is associated with neural tube defects, such as anencephaly and spina bifida. Cocaine use is associated with some of the same signs and symptoms as alcohol use but does not produce the characteristic facial abnormalities of fetal alcohol syndrome. Herpes simplex infection, although it is associated with microcephaly, hydrocephalus, defects of the eye, and hearing problems, also does not produce characteristic facial abnormalities.

A client tells the nurse that the primary care provider prescribed an antibiotic. The client was instructed to take the antibiotic three times a day for 10 days. After the 4th day, the client was feeling much better and decided to stop taking the medication. What complication could result from stopping the antibiotic early?

Chronic infection Explanation: Chronic infection can result from stopping antibiotics early because of the increasing prevalence of bacteria resistant to the effects of antibiotics. Septicemia, bacteremia, and septic shock are not the result from stopping antibiotics early.

A client reports fatigue and an inability to concentrate. The client has been unemployed for the second time in a year and is concerned about supporting the family. Select the type of stressor the client most likely is experiencing.

Chronic sustained Explanation: Chronic intermittent or chronic sustained stress occurs when a person is repeatedly exposed to a stressor. An acute time-limited stressor is one that occurs over a short time and does not reoccur.

Peripheral neuropathies occur in people with diabetes mellitus. With the loss of sensation in the lower extremities, diabetics become predisposed to what impairment?

Clawing of toes and denervation of the small muscles of the foot Explanation: Loss of feeling, touch, and position sense, which increases the risk of falling, are hallmark symptoms of peripheral neuropathy. Impairment of temperature and pain sensation increases the risk of serious burns and injuries to the feet. Denervation of the small muscles of the foot result in clawing of the toes and displacement of the submetatarsal fat pad anteriorly. These changes together with joint and connective tissue changes alter the biomechanics of the foot, increasing plantar pressure and predisposing to development of foot trauma and ulcers.

Which description does the nurse recognize fits a client in the prodromal stage of an infection?

Client is experiencing vague symptoms of fatigue and low grade fever. Explanation: Stages of infection are incubation period, prodromal stage, acute stage, convalescent stage, and the resolution stage. During incubation, there are no recognizable symptoms in the host. The prodromal stage has the initial appearance of vague, non-specific symptoms such as malaise, myalgia, and low grade fever. The acute stage is when the maximal impact on the host is evident through specific symptoms commonly from toxins and inflammation. The convalescent period is when the infection is contained and progressively eliminated from the body. The resolution is when the infection has been cleared from the body.

Which client would be diagnosed with wasting syndrome?

Client with HIV, fever, diarrhea, and significant involuntary weight loss Explanation: A client diagnosed with wasting syndrome (an AIDS-defining illness) would have chronic fever, diarrhea, and a significant involuntary weight loss (usually more than 10%) without an opportunistic infection. Pneumonia, Kaposi sarcoma, and herpes simplex and candidiasis are all opportunistic infection.

The nurse is caring for a client who choked on a peanut and then collapsed. After partially clearing the obstruction, it was noted the client experienced prolonged hypoxia. The nurse anticipates that the resulting cerebral infarction will lead to which phenomenon?

Coagulation necrosis Explanation: Tissue infarction is caused by prolonged oxygen deprivation, and the resulting large group of dead cells coagulates in the damaged area (coagulation necrosis). Apoptosis activates caspases (proteases) early in the process of controlled cell death. Phagocytosis occurs rapidly during apoptosis, so it does not elicit an inflammatory response. Protein p53 activation initiates

A client is admitted with frostbite. What will the nurse tell the client about the changes that have occurred due to cold exposure?

Cold increases blood viscosity and thrombosis. Explanation: Cold temperature exposure causes increased blood viscosity and can cause clots to form. Vasoconstriction is induced, and the flow of blood is slowed due to this. Cells do not hypertrophy in response to temperature change.

While explaining to a young child why he should be careful with a wound, the nurse reviews healing with the parent. The nurse educates the parent about how strength in the healing wound site is developed based on which substance being available?

Collagen synthesis Explanation: Collagen synthesis is important to the development of strength in the healing wound site. Plasma, exudate, and stable cells do not give strenth to the healing tissue.

The client is found to have liver disease, resulting in the removal of a lobe of the liver. Adaptation to the reduced size of the liver leads to which phenomenon in the remaining liver cells?

Compensatory hyperplasia Explanation: Compensatory hyperplasia can be stimulated in response to loss of vital tissue that is capable of regeneration, such as liver cells. Metaplasia involves replacement of one existing cell type with another fully differentiated cell type. Organ atrophy is caused by irreversible loss of cells. Physiologic hypertrophy is increased size of existing cells resulting from increased workload.

Which criterion about insulin would prompt a diagnosis of type 1 diabetes?

Complete failure of insulin secretion Explanation: In type 1 diabetes there is an absolute lack of insulin due to complete failure of the pancreas. In type 2 diabetes some insulin is produced but may not be properly used.

The nurse is preparing to administer a pregnant client's medication when it is noted that the client is prescribed tetracycline for an infection. The nurse should do which of the following?

Contact the physician Explanation: The nurse should contact the physician and request a different antibiotic as tetracylcine is a known teratogenic agent.

A hospitalized client's progress has been noted as the convalescent stage. Select the statement that best describes this stage.

Containment of infection, progressive elimination of the pathogen, repair of damaged tissue, and resolution of associated symptoms Explanation: The convalescent period is characterized by the containment of infection, progressive elimination of the pathogen, repair of damaged tissue, and resolution of associated symptoms. The resolution is the total elimination of a pathogen from the body without residual signs or symptoms of disease. The acute stage is the period during which the host experiences the maximum impact of the infectious process corresponding to rapid proliferation and dissemination of the pathogen. The prodromal stage is the initial appearance of symptoms in the host, although the clinical presentation during this time may be only a vague sense of malaise.

A physiology instructor asks the students about the purpose of the promoter region on a DNA strand. Which student response is most accurate?

Contains amino acids that the RNA polymerase recognized and binds to, thus starting the replication process Explanation: During transcription, RNA polymerase recognizes the start sequence of a gene and attaches to the DNA. It is initiated by the assembly of a transcription complex composed of RNA polymerase and other associated factors. This complex binds to the double-stranded DNA at a specific site called the promoter region. Within the promoter region is the so-called "TATA box," which contains the crucial thymine-adenine-thymine-adenine sequence that RNA polymerase recognizes and binds to, starting the replication process. Exons are RNA sequences retained on the original RNA during splicing. tRNA delivers activated amino acids to proteins in the ribosome. When several triplet codons encode the same amino acid, the genetic code is said to be redundant.

The control systems of the body act in many ways to maintain homeostasis. These control systems regulate the functions of the cell and integrate the functions of different organ systems. What else do they do?

Control life processes Explanation: The body's control systems regulate cellular function, control life processes, and integrate functions of the different organ systems. Homeostatic control systems do not feed cells when they are under stress, they do not act on invading organisms, and they do not shut down the body at death.

It has long been known that our bodies need a stable internal environment to function optimally. What serves to fulfill this need?

Control systems Explanation: A homeostatic control system consists of a collection of interconnected components that function to keep a physical or chemical parameter of the body relatively constant. Organ systems are a group of organs that function together to accomplish necessary functions in the body; for example, the cardiovascular system provides blood to all the body's components. Biochemical messengers are in the brain; they are not control systems. Neuroendocrine systems are control systems that help to regulate our response to stress. Neurovascular systems do not aid in the control of homeostasis in the body.

A client taking the typical antipsychotic agent haloperidol experiences neuroleptic malignant syndrome. What nursing intervention is highest priority after stopping the medication?

Cooling blanket Explanation: Neuroleptic malignant syndrome (NMS) is a life-threatening condition most often triggered by typical antipsychotic agents such as haloperidol and chlorpromazine. The condition usually has sudden onset of hyperthermia, muscle rigidity, change in mental status, and autonomic dysfunction seen in labile blood pressure, dyspnea, and tachycardia. The priority interventions are to stop the medication and reduce the temperature using a cooling blanket or ice packs in the axillae and groin. Subsequent interventions of IV fluids and ventilator support are directed at controlling symptoms and supporting body systems. Dopamine agonists such as bromocriptine have been used to reduce the effect of the triggering neuroleptic agent. Benzodiazepines such as diazepam can be used to reduce anxiety.

A client is diagnosed with a mild case of an autoimmune disorder and is beginning a treatment regimen. What is the mainstay of treatment with an autoimmune disorder?

Corticosteroids and immunosuppressants Explanation: Treatment of autoimmune disorders is dependent on the magnitude of the presenting manifestations and underlying mechanisms of the disease process. Because in many cases the pathophysiologic mechanisms are not always known, treatment may be purely symptomatic. Corticosteroids and immunosuppressive drugs are the mainstay of therapy directed at arresting or reversing the cellular damage caused by the autoimmune response. Plasmapheresis has been used in severe cases to remove autoreactive cells from circulation.

Which of these is an endocrine regulator of pituitary and adrenal activity and a neurotransmitter involved in autonomic nervous system activity, metabolism, and behavior?

Corticotropin-releasing factor Explanation: Corticotropin-releasing factor is a small peptide hormone found in both the hypothalamus and in extrahypothalamic structures, such as the limbic system and the brain stem. It is both an important endocrine regulator of pituitary and adrenal activity and a neurotransmitter involved in autonomic nervous system activity, metabolism, and behavior. The sympathetic nervous system manifestation of the stress reaction has been called the "fight or flight" response. This is the most rapid of the stress responses and represents the basic survival response of our primitive ancestors when confronted with the perils of the wilderness and its inhabitants. The term allostasis has been used by some investigators to describe the physiologic changes in the neuroendocrine, autonomic, and immune systems that occur in response to either real or perceived challenges to homeostasis. The hallmark of the stress response, as first described by Selye, is the endocrine-immune interaction.

After having a very stressful day in pathophysiology class, the student knows that which hormone (secreted by the adrenal cortex) will help decrease the effects of stress?

Cortisol, a glucocorticoid Explanation: Glucocorticoids, mainly cortisol, affect metabolism of all nutrients; regulate blood glucose levels; affect growth; have anti-inflammatory action; and decrease effects of stress. The other hormones do not affect stress levels.

The newborn nursery nurse is obtaining a blood sample to determine if a newborn has congenital hypothyroidism. What long-term complication is the nurse aware can occur if this test is not performed and the infant has congenital hypothyroidism?

Cretinism Explanation: Congenital hypothyroidism is a common cause of preventable intellectual disability. It affects approximately 1 in 4000 infants. The manifestations of untreated congenital hypothyroidism are referred to as cretinism. The term does not apply to the normally developing infant in whom replacement thyroid hormones therapy was instituted shortly after birth.

Select the infectious disease that may be acquired from protein particles, such as prions. Select all that apply.

Creutzfeldt-Jakob disease Chronic wasting disease Bovine spongiform encephalopathy Explanation: Prion-associated diseases have been identified, including Creutzfeldt-Jakob disease in humans, chronic wasting disease in deer and elk, scrapie in sheep, and bovine spongiform encephalopathy (BSE or mad cow disease) in cattle. The other options are not caused by prions.

What major group of intracellular proteins controls cell division? Select all that apply.

Cyclins Cyclin-dependent kinases Anaphase-promoting complexes Cell division is controlled by changes in the concentrations and activity of three major groups of intracellular proteins: cyclins, cyclin-dependent kinases (CDKs), and anaphase-promoting complexes. The citric acid cycle, sometimes called the tricarboxylic acid (TCA) or Krebs cycle, provides the final common pathway for the metabolism of nutrients. G proteins function as an on-off switch to convert external signals (first messengers) into internal signals (second messengers).

Which of the types of T cells is responsible for destroying pathogens by punching holes in their cell membrane and by secreting cytokines/lymphokines?

Cytotoxic T cells Explanation: Cytotoxic T cells destroy the pathogens. Regulatory T cells suppress immune response. Memory T cells are derived from normal T cells. The principal cells of the adaptive immune system are the T and B lymphocytes and antigen-presenting cells. T lymphocytes differentiate into helper T cells, regulatory T cells, and cytotoxic T cells, providing cell-mediated immunity. CD4+ helper T cells serve as a trigger for the immune response and are essential for the differentiation of B cells into antibody-producing plasma cells and the differentiation of T lymphocytes into CD8+ cytotoxic T cells.

Which muscle change places an individual at high risk factor for injury?

Decrease in muscle mass Explanation: A decrease in muscle mass causes older adults to be prone to falls and fractures. Increase in systolic pressure is more of a risk for the older adult than a decrease in diastolic pressure. Increased lean mass and decreased fat mass do not place the older adult at a high safety risk.

A client experiences an increase in cortisol as a result of Cushing disease. Which hormonal responses demonstrate the negative feedback mechanism?

Decreased adrenocorticotropic hormone (ACTH) Explanation: Negative feedback occurs when secretion of one hormone causes a reduction in the secretion of the hormone that stimulates production of the first hormone. In this case, ACTH manufactured by the anterior pituitary gland would normally stimulate the release of cortisol, but with the increase of cortisol produced by the secreting tumor, enough cortisol already floods the system that there should be a reduction in the ACTH level.

A nurse assessing an older adult for signs and symptoms of infection in the absence of a fever should assess for which of the following? Select all that apply.

Decreased mental status Change in fuctional capacity Fatigue Explanation: Signs and symptoms of infection in an older adult in the absence of a fever include decreases in mental status and functional capacity, fatigue, weight loss, and weakness.

A client's condition has resulted in a decrease in work demands of most cells in the body. Which change within the cell will likely result from this decrease in work?

Decreased size of organelles Explanation: Cell atrophy results in a decreased number and size of cell organelles. With cell atrophy, the cell has decreased oxygen consumption and growth of the mitochondria. The cell decreases in size. When confronted with a decrease in work demands or adverse environmental conditions, most cells are able to revert to a smaller size and to a lower and more efficient level of functioning that is compatible with survival. This decrease in cell size is called atrophy. Cellular death process is not affected.

Which risk factor presents the greatest risk for injury in an older adult who is experiencing a dysfunction in temperature regulator function?

Delayed initiation of appropriate treatment Explanation: Older adults often have a lower baseline temperature (36.4°C [97.6°F]) than younger persons, and although their temperature increases during an infection, it may fail to reach a level that is equated with significant fever. The absence of fever may delay diagnosis and initiation of antimicrobial treatment. Absence of fever does not result in misdiagnosis. While the remaining options may occur, they are not the greatest risks for injury.

A group of teenagers spent an entire day on the beach without using sunscreen. The first night, their skin was reddened and painful to touch. The second day, they awoke to find large fluid-filled blisters over several body areas. The nurse recognizes the development of blisters as which type of inflammatory response?

Delayed response Explanation: The first pattern is an immediate transient response, which occurs with minor injury. It develops rapidly after injury and is usually reversible and of short duration (15 to 30 minutes). The second pattern is an immediate sustained response. The third pattern is a delayed response, in which the increased permeability begins after a delay of 2 to 12 hours, lasts for several hours or even days, and involves venules as well as capillaries. A delayed response often accompanies injuries due to radiation, such as sunburn. The cellular stage of acute inflammation is marked by changes in the endothelial cells lining the vasculature and movement of phagocytic leukocytes into the area of injury or infection.

A clinical research study is evaluating cells that bridge both the innate and adaptive immune systems. A nurse has identified the dendritic cells (DCs) as a key component. Which statement validates this finding?

Dendritic cells (DCs) directly sense pathogens, capture foreign agents, and transport them to secondary lymphoid tissues. Once activated, they undergo a maturation process and function as antigen-presenting cells (APCs) capable of initiating adaptive immunity. Explanation: Dendritic cells (DCs) are specialized, bone marrow-derived leukocytes found in lymphoid tissue and are the bridge between the innate and adaptive immune systems. They are present primarily in an immature form that is available to directly sense pathogens, capture foreign agents, and transport them to secondary lymphoid tissues. Once activated, DCs undergo a complex maturation process in order to function as key antigen-presenting cells (APCs) capable of initiating adaptive immunity. They are responsible for the processing and presentation of foreign antigens to the lymphocytes.

A client experiences compartment syndrome after a leg injury. Surgical intervention will focus on which type of connective tissue?

Dense connective tissue Dense connective tissue such as muscle fascia forms the fibrous sheath around many organs and bone. Compartment syndrome is when the fascia fills with blood or fluid compressing the vascular and nervous connections to the extremity. Surgical release of the fascia is essential to restoring function of the extremity.

A number of fungi, such as ringworm, athlete's foot, and jock itch, are incapable of growing at a core body temperature. Hence, their infection is limited to cooler cutaneous surfaces. What are these pathogens known as?

Dermatophytes Explanation: Fungal pathogens called dermatophytes are incapable of growing at core body temperature (37°C [98.6°F]), and the infection is limited to the cooler cutaneous surfaces. Diseases caused by these organisms, including ringworm, athlete's foot, and jock itch, are collectively called superficial mycoses.

The nurse caring for a client with a large leg wound notes that the client's past medical history includes diabetes, chronic atrial fibrillation, asthma controlled by maintaninence medication, and neuropathy. Which one has the greatest potential to negatively impact the healing of the client's wound?

Diabetes Explanation: The client's diabetes has the greatest potential to negatively impact the client's wound healing as this places the client at risk for delayed wound healing and infection from hyperglycemia.

The nurse is admitting an older adult client with a chronic wound. The nurse will focus the assessment on which underlying conditions? Select all that apply.

Diabetes Immobility Vascular disease Explanation: The nurse should assess for diabetes, immobility, and vascular disease as these are more often the cause of wounds in the older adult than is aging.

An adult client presents to the emergency department after an episode of syncope. Blood pressure is low and the pulse is thready and rapid. The client frequently voids large amounts of pale, clear urine. Lung sounds are clear and skin turgor is inelastic. Which endocrine disorder is the likely cause of these manifestations?

Diabetes insipidus Explanation: Normally, antidiuretic hormone (ADH) from the posterior pituitary gland causes the renal tubule to reabsorb water and sodium. When too little ADH is secreted, the client will lose large amounts of fluid and become dehydrated. Despite fluid volume deficit, the urine output will be dilute and in large amounts. In hyperparathyroidism, the client will have an elevated blood calcium level and cardiac dysrhythmias. Fluid volume will remain stable, but the client may develop renal calculi. In myxedema, lack of thyroid hormones causes the client to become sluggish. Urinary output remains normal. Gigantism results from excess production of growth hormone in children and results in large body size. It does not influence urine output.

Which option most accurately describes the physiologic process resulting in more specialized cells with each mitotic division?

Differentiation Explanation: Cell differentiation is the process whereby a cell becomes more specialized in terms of structure and function. Cell proliferation refers to the process of increasing cell numbers by mitotic division. Stem cells are undifferentiated cells that have the capacity to generate multiple cell types. Cell death is called apoptosis. The cell cycle is an orderly sequence of events that occur as a cell duplicates its genetic contents and divides.

Sexually transmitted infections (STIs) are typically spread by which mechanisms?

Direct contact Explanation: Most STIs are spread by direct contact. Vertical transmission is possible for many sexually transmitted pathogens, but direct contact is more common. Ingestion and penetration are less likely mechanisms.

Treponema pallidum is a spirochete bacterium that causes syphilis. Which of the following is the mechanism that spreads T. pallidum from human to human?

Direct physical contact Explanation: T. pallidum is a sexually transmitted infection that is spread by direct physical contact. The Borrelia type of spirochete is spread from animal to human through lice or tick bites. Leptospira spirochetes spread from animals to humans through contact with infected animal urine. Spirochetes are anaerobic; therefore, they would not invade the host through oxygen-filled aerobic lungs.

When the nurse is performing a skin assessment on a client, a small wound is noted on the client's right leg. The wound is covered with a desiccated scab. The appropriate action by the nurse is:

Document the finding Explanation: The nurse should document the finding in the client's chart. Documentation should include wound measurements, notation of the scab and peri-wound skin characteristics. As the scab provides a protective barrier for the wound while the wound is healing, there is no need for antibiotic ointment, debriding agent, and/or surgical consult.

Which feature of a DNA molecule allows for precise replication?

Double-stranded structure Explanation: The double-stranded structure of DNA molecules allows them to replicate precisely by separation of the two strands, followed by synthesis of two new complementary strands. Similarly, the base complementary pairing allows for efficient and correct repair of damaged DNA molecules. A precise complementary pairing of purine and pyrimidine bases allows for each nucleotide in a pair to bind with the bases on opposite DNA strands by hydrogen bonds that are extremely stable under normal conditions. The spiral staircase of paired bases allows for precise pairing of the bases. Several hundred to almost one million base pairs can represent a gene, the size being proportional to the protein product it encodes.

With increasing age comes a greater chance of a woman having been exposed to damaging environmental agents such as drugs, chemicals, and radiation. These factors may act on the aging oocyte to cause which abnormality in a fetus?

Down syndrome Explanation: The risk of having a child with Down syndrome increases with maternal age—it is 1 in 1250 at 25 years of age, 1 in 400 at 35 years, and 1 in 100 at 45 years of age. The reason for the correlation between maternal age and nondisjunction is unknown, but is thought to reflect some aspect of aging of the oocyte. Although males continue to produce sperm throughout their reproductive life, females are born with all the oocytes they ever will have. These oocytes may change as a result of the aging process. With increasing age, there is a greater chance of a woman having been exposed to damaging environmental agents such as drugs, chemicals, and radiation. There is no correlation with maternal age and the other syndromes.

Maternal serum alpha-fetoprotein (AFP) is a screening test for which condition?

Down syndrome Explanation: There are several prenatal screening tests that can be done to determine the risk of having a child with Down syndrome. The most commonly used are blood tests that measure maternal serum levels of alpha-fetoprotein (AFP), human chorionic gonadotropin (hCG), unconjugated estriol, inhibin A and pregnancy-associated plasma protein A (PAPP-A). AFP is not a screening test for the other conditions.

A nurse practitioner is preparing to perform a client's Pap test and is answering the client's questions about the clinical rationale for the procedure. The nurse should describe what phenomenon?

Dysplasia of the cervical epithelium is associated with a high risk of cancer. Explanation: A Pap test involves a cytologic examination of epithelial cells checking for cell dysplasia, which is associated with cervical cancer. These cells are not prone to metastasis, calcification, or hypertrophy.

The older adult client with a history of congestive heart failure is upset following the death of her husband yesterday. The practitioner observes the client for which of the following?

Dysrhythmias Explanation: For people with limited coping abilities, either because of physical or mental health, the acute stress response may be detrimental. For people with preexisting heart disease, the overwhelming sympathetic behaviors associated with stress can lead to dysrhythmias. The other options are not stimulated by the sympathetic response.

Which statement is true concerning chromosomes?

Each human somatic cell has 23 pairs of different chromosomes Explanation: Each human somatic cell has 23 pairs of different chromosomes, one pair derived from the individual's mother and the other from the father. One of the chromosome pairs consists of the sex chromosomes. Genes are arranged linearly along each chromosome. Each chromosome contains one continuous linear DNA helix.

During a blood transfusion, a client begins to have chills, back pain, and develops a fever. The nurse determines the client is experiencing a transfusion reaction. The nurse educates the client about transfusion reactions. Which statement indicates the cause of the reaction?

Each person has two HLA haplotypes that identify human cells. Blood typing involves the identification of these haplotypes (ABO compatibility). The closer the matching of HLA types, the greater is the probability of identical antigens and the lower the chance of rejection. A reaction occurs if the immune system views the HLA types as foreign. Explanation: Human leukocyte antigens (HLAs) are the major target involved in rejection. Each person inherits one chromosome from each parent, supplying two HLA haplotypes. Blood typing involves the identification of these haplotypes. In transplantation, the closer the matching of HLA types, the greater the probability of identical antigens and the lower the chance of rejection. A reaction occurs if the blood types do not exactly match and the immune system views the HLA types as foreign.

The nurse is assessing a client for acute inflammation of a wound. For which symptom of infection does the nurse assess?

Edema Explanation: Cardinal signs of inflammation include rubor (redness), tumor (swelling or edema), calor (heat), dolor (pain) and functio laesa (loss of function). Tissue necrosis occurs with chronic inflammation.

When exercising outside on an extremely warm day, the client can feel his heart pounding very rapidly. Thinking in terms of the ability of the aorta to stretch to accommodate more blood circulating during exercise and the warm environment, this is an example of the function of which fibrous protein?

Elastin Explanation: Three types of fibers are found in the extracellular space: collagen, elastin, and reticular fibers. Elastin acts like a rubber band; it can be stretched and then returns to its original form. Elastin fibers are abundant in structures subjected to frequent stretching, such as the aorta and some ligaments. Collagen is the most common protein in the body. It is a tough, nonliving, white fiber that serves as the structural framework for skin, ligaments, tendons, and many other structures. Reticular fibers are extremely thin fibers that create a flexible network in organs subjected to changes in form or volume, such as the spleen, liver, uterus, or intestinal muscle layer.

Which type of thermometer is the best to use in determining an accurate temperature during an episode of hypothermia?

Electrical thermistor Explanation: An electrical thermistor probe is used to monitor temperatures as low as 25°C (77°F). Clinical thermometers, such as oral or rectal thermometers, are not able to register at lower levels that are associated with hypothermia. The use of a mercury thermometer is not recommended because of the hazards associated with mercury.

A nurse examines the laboratory values of a client in heart failure. Which value indicates a compensatory hormone mechanism?

Elevated atrial natriuretic hormone Explanation: In heart failure, the client experiences fluid backlog in the heart as venous blood continues to return, but cardiac output is reduced. This stretches the atria, which secrete atrial natriuretic hormone (or peptide) to stimulate vasodilation and increased renal excretion of sodium and water. This reduces the volume and the strain in the heart.

Which manifestation would the nurse observe when assessing a client diagnosed with a benign secreting tumor of the adrenal medulla?

Elevated catecholamine levels and tachycardia Explanation: The adrenal medulla secretes the neurotransmitters epinephrine (adrenalin) and norepinephrine (a precursor to epinephrine). A secreting tumor will cause the client to experience manifestations of sympathetic stimulation, such as episodic anxiety, tachycardia, and hypertension. A tumor of the adrenal medulla will not impact thyroid or aldosterone secretion.

Which form of signal transduction resulting from ligand-receptor binding has the potential to produce effects in the entire body system?

Endocrine Explanation: Cells communicate in several ways. In endocrine signaling, the cells release chemical mediators (hormones) that enter the bloodstream, then bind to cell receptors throughout the body. A ligand is a substance with a high affinity for the receptor. In autocrine signaling, a cell releases a chemical to the extracellular fluid, which changes its own activity. In paracrine signaling, a cell releases a chemical to the extracellular fluid, but enzymes rapidly metabolize the chemicals so they only act on nearby cells.

Enzymes that assist in the repair of deoxyribonucleic acid (DNA) defects by recognizing the defect, cleaving the abnormal chain, and then removing the distorted regions are known as:

Endonucleases Explanation: Several repair mechanisms exist, and each depends on specific enzymes called endonucleases that recognize local distortions of the DNA helix, cleave the abnormal chain, and remove the distorted regions. The other options are not enzymes involved in this process.

When discussing the various functions of the cellular parts, the pathophysiology professor asks a group of students, "Where are the primary sites of protein synthesis located in a cell?" Which student response is correct?

Endoplasmic reticulum Explanation: The endoplasmic reticulum (with its associated ribosomes) and Golgi apparatus represent the primary sites of protein synthesis in the cell.

The cytoplasm surrounds the nucleus of the cell and contains organelles. Select the organelles found in the cytoplasm. Select all that apply.

Endoplasmic reticulum Golgi apparatus Mitochondria Lysosomes Explanation: The cytoplasm surrounds the nucleus, and it is in the cytoplasm that the work of the cell takes place. Embedded in the cytoplasm are various membrane-enclosed organelles (e.g., endoplasmic reticulum, Golgi apparatus, mitochondria, and lysosomes) and complexes without membranes (e.g., ribosomes and proteasomes) that have important functions in cells. In addition to its organelles, the cytoplasm also contains a network of microtubules, actin microfilaments, and intermediate filaments called the cytoskeleton.

A pregnant client has just undergone cytogenetic studies to determine fetal karyotyping. She learns that her fetus has trisomy 21. Which of the following is a manifestation of this condition?

Epicanthal eye folds Explanation: A child with Down syndrome, or trisomy 21, has a flat facial profile, with a small nose and somewhat depressed nasal bridge; small folds in the inner corners of the eyes (epicanthal folds), and upward slanting of the eyes. The other responses are not seen in Down syndrome.

Severe acute respiratory syndrome (SARS), a highly transmissible respiratory infection, crossed international borders in the winter of 2002. What terms are used to describe the outbreak of SARS?

Epidemic and pandemic Explanation: Severe acute respiratory syndrome (SARS) was recognized in the Guangdong province in southern China beginning in November 2002. The illness was highly transmissible, as evidenced by the first recognized occurrence in Taiwan. Four days after returning to Taiwan from work in the Guangdong province, a businessman developed a febrile illness and was admitted to a local hospital. Within 1 month, a large nosocomial outbreak of SARS was documented to have affected approximately 3000 people in Taipei City, Taiwan. Since the SARS outbreak began in China and crossed continental borders for the first time, it was classified as not only an epidemic but also a pandemic. Regional and endemic mean the same thing, a specific area where the disease occurs. Nosocomial is an infection acquired in a health care facility.

A family consumed some undercooked hamburger at a picnic and has since developed bloody diarrhea. The nurse knows that which statement is correct regarding the infectious process?

Exotoxins that damage vascular endothelial cells cause bleeding and low platelet counts. Explanation: Exotoxins can allow organisms to produce hemorrhagic colitis, which can be fatal. It is characterized by vascular endothelial damage, acute RF, and thrombocytopenia (low platelet counts). Toxic cell walls are classified as endotoxins. The ability to survive immune responses characterizes evasive factors. Adhesion factors and invasive factors are not involved in this situation.

Select the statement that best describes the effectiveness of vaccination in the older adult population.

Experimental evidence suggests that vaccination is less successful in inducing immunization in older persons than in younger adults. Explanation: Older adults tend to be more susceptible to infections, have more evidence of autoimmune and immune complex disorders than younger persons, and have a higher incidence of cancer. Experimental evidence suggests that vaccination is less successful in inducing immunization in older persons than in younger adults. However, the effect of altered immune function on the health of older adults is clouded by the fact that age-related changes or disease may affect the immune response.

The nurse is reviewing assessment documentation of a client's wound and notes "purulent drainage." The nurse would interpret this as:

Exudate containing white blood cells, protein, and tissue debris Explanation: A purulent or suppurative exudate contains pus, which is composed of degraded white blood cells, proteins, and tissue debris. Fibrinous exudates contain large amounts of fibrinogen. Serous exudates are watery fluids low in protein. Hemorrhagic exudates occur when there is severe tissue injury that causes damage to blood vessels or when there is significant leakage of red cells

A specific type of gram-negative bacteria contains endotoxin in the bacterial cell envelope. What is the likely clinical manifestation if these bacteria become pathogenic?

Fever Explanation: A small amount of endotoxin in the circulatory system (endotoxemia) can induce clotting, bleeding, inflammation, hypotension, and fever.

The nurse is assessing a client diagnosed with varicella. The nurse is aware that nonspecific manifestations may include:

Fever Explanation: Symptoms such as fever, myalgia, headache, and lethargy are nonspecific manifestations of varicella. Liquid-filled blisters, sores with crustations, and pruritic rash are specifically related to this disease process.

When caring for a client during the proliferative phase of wound healing, the nurse teaches the client that which of these processes is taking place?

Fibroblasts secrete collagen for wound healing. Explanation: During the proliferative phase, fibroblasts synthesize and secrete collagen and other intercellular elements needed to fill the wound space. Fibroblasts also produce a family of growth factors that induce angiogenesis and endothelial cell proliferation and migration.

A client is being treated for a pressure ulcer and the care team has observed that the wound is healing. Which activity will take place during the proliferative phase?

Fibroblasts secrete the cells necessary for wound healing Explanation: The primary processes during the proliferative phase focus on the building of new tissue to fill the wound space. The key cell during this phase is the fibroblast, a connective tissue cell that synthesizes and secretes the collagen, proteoglycans, and glycoproteins needed for wound healing. White cell migration and clot formation take place during the inflammatory phase. Scar formation occurs during remodelling.

A client is seeking treatment for infertility. What decrease in hormone secretion from the anterior lobe of the pituitary gland that regulates fertility would the nurse recognize may cause this issue?

Follicle stimulating hormone (FSH) Explanation: ACTH controls the release of cortisol from the adrenal gland. TSH controls the secretion of thyroid hormone from the thyroid gland. LH regulates sex hormones. FSH regulates fertility.

A home health nurse is visiting an older adult client who is exhibiting signs of an upper respiratory infection but denies contact with an infected person. The nurse evaluates the home and recommends air conditioner duct cleaning as a precautionary measure against which pathogen?

Fungi Explanation: Fungi are free-living, eukaryotic saprophytes found in every habitat on earth. Serious fungal infections are rare and usually initiated through puncture wounds or inhalation. Air blowing into a room may be the mode of transfer of fungi spores, which have remained dormant in the duct during nonuse. Disease states or antibiotic therapy can upset the balance of normal flora and permit fungal overgrowth and set the stage for opportunistic infections.

A nurse assessing a client for Selye's conditioning risk factors for stress-related disease should include which of the following? Select all that apply.

Genetic predisposition Dietary factors Social support Age Explanation: Selye's conditioning risk factors for stress-related disease include internal factors such as genetic predisposition, age, sex, and external factors such as dietary factors, social support, and life experiences.

Type 1A diabetes is now considered an autoimmune disorder. What factors are considered necessary for type 1A diabetes to occur?

Genetic predisposition, environmental triggering event, and a T-lymphocyte-mediated hypersensitivity reaction against some beta-cell antigen Explanation: Type 1A diabetes is thought to be an autoimmune disorder resulting from a genetic predisposition (i.e., diabetogenic genes); an environmental triggering event, such as an infection; and a T-lymphocyte-mediated hypersensitivity reaction against some beta-cell antigen.

After studying genetic disorders, the pathophysiology student knows that which of the following is true?

Genotype determines phenotype. Explanation: The genetic compostion of a person is called a genotype, whereas the phenotype is the observable expression of a genotype. Diabetes mellitus is caused by a multifactorial inheritance disorder and sickle cell anemia by a single gene defect. Many diseases caused by a single-gene defect have been identified.

A nurse on the oncology floor reads a client's chart and sees that the client was treated for an adenocarcinoma. What type of tissue composes this tumor?

Glandular epithelium Explanation: The term "adenocarcinoma" denotes a malignancy of the glandular epithelium. CNS cancers usually begin with "neuro-" and there is no particular term used to denote a stem cell cancer. There are varied terms used to describe cancers of the connective tissue, depending on the tissue type.

The family of a client in the hospital with diabetes mellitus that is out of control asks the nurse to explain the client's recent weight loss while eating more than usual. How will the nurse respond?

Glucose is unused without insulin, so body fats are used for energy. Explanation: Most of these options are true statements, but they do not answer the question asked by the family. Normally, nutrients are metabolized in a number of ways. Glucose is transported into cells by insulin and then is broken down to carbon dioxide and water. When there is surplus glucose present, it is metabolized and stored as glycogen in the liver and skeletal muscles. Further surplus is converted by the liver to fatty acids and stored as triglycerides. When triglycerides are metabolized, the glycerol molecule enters the glycolytic pathway to release energy. Excess proteins are also converted to fatty acids for storage. Insulin is needed to transport glucose into cells, prevent fat breakdown, and inhibit gluconeogenesis. When diabetes is out of control there is lack of insulin. Weight loss occurs as the cells break down fats to use fatty acids for energy.

Biologic agents differ from other injurious agents in that they are able to replicate and can continue to produce their injurious effects. How do Gram-negative bacteria cause harm to the cell?

Gram-negative bacilli release endotoxins that cause cell injury and increased capillary permeability. Explanation: Gram-negative bacilli release endotoxins that cause cell injury and increased capillary permeability. Certain bacteria excrete elaborate exotoxins that interfere with cellular production of ATP. Gram-negative bacilli do not disrupt a cell's ability to replicate. Many Gram-negative bacilli cause harm to cells.

The angiogenesis process, which allows tumors to develop new blood vessels, is triggered and regulated by tumor-secreted:

Growth factors Explanation: Many tumors secrete growth factors, which trigger and regulate the angiogenesis process. Tumor cells express various cell surface attachment factors, for anchoring. Tumor cells secrete proteolytic enzymes to degrade the basement membrane and migrate into surrounding tissue. Cancer cells may produce procoagulant materials that affect clotting mechanisms.

Which hormone is produced by the anterior pituitary gland?

Growth hormone (GH) Explanation: GH is among the hormones produced and released by the anterior pituitary. Oxytocin is a posterior pituitary hormone, whereas CRH is produced by the hypothalamus. Norepinephrine and epinephrine are produced by the adrenal medulla.

The nurse assessing a client with Klinefelter syndrome would evaluate which signs as being expected findings? Select all that apply.

Gynecomastia Testicular atrophy Explanation: The nurse assessing a client with Klinefelter syndrome would evaluate the findings of gynecomastia and testicular atrophy as expected findings. However, excessive facial hair and decreased cognition would not be considered expected findings.

Which of the following would be considered an example of transmitting an infection from person-to-person through shared inanimate objects (fomites)?

HIV from a contaminated IV drug user needle Explanation: Infections can be transmitted from person-to-person through shared inanimate objects (fomites) contaminated with infected body fluids. An example of this mechanism of transmission would include the spread of the HIV and hepatitis B virus through the use of shared syringes by intravenous drug users. Infection can also be spread through a complex combination of source, portal of entry, and vector. Infection with hantavirus pulmonary syndrome is a prime example. This viral illness is transmitted from mice to humans by inhalation of dust contaminated with saliva, feces, and urine of infected rodents.

Two people experience the same stressor, yet only one is able to cope and adapt adequately. Which term do physiologists use to describe this effect, which is correlated to positive health outcomes?

Hardiness Explanation: A person with "hardiness" has a sense of purpose in life and is better able to adapt to stress. Other factors that decrease physical stress and increase the ability to adapt include a regular circadian rhythm pattern and appropriate weight for body size. Psychological dysfunction is not associated with a positive sense of purpose in life.

While teaching a preconception workshop for young adults, the nurse knows that the participants understand the educational teaching when they state which agents are teratogenic? Select all that apply.

Having a glass of wine with dinner Casual use of cocaine The use of warfarin Explanation: Even small amounts of alcohol consumed during critical periods of fetal development may be teratogenic. There is no evidence that diagnostic levels of radiation (from a chest x-ray) cause congenital abnormalities. Among drugs known to cause fetal abnormalities are cocaine and warfarin. Vitamin A and its derivatives have been targeted for concern of their teratogenic potential but not Vitamin D.

A client arrives with a large calf burn from contact with a motorcycle exhaust pipe. Inspection reveals the epidermis and dermis layers are affected with associated pain, redness, swelling, and blistering. What type of wound repair/healing will the nurse explain to this client?

Healing will occur by secondary intention. Explanation: Larger wounds, like burns, have greater loss of tissue and contamination; therefore, they heal by secondary intention. Healing by secondary intention is slower than healing by primary intention and results in the formation of larger amounts of scar tissue.

The nurse is performing an assessment for a client who has hyperthyroidism that is untreated. When obtaining vital signs, what is the expected finding?

Heart rate 110 and bounding Explanation: Cardiovascular and respiratory functions are strongly affected by thyroid function. With an increase in metabolism, there is a rise in oxygen consumption and production of metabolic end products, with an accompanying increase in vasodilation. Blood volume, cardiac output, and ventilation are all increased. Heart rate and cardiac contractility are enhanced as a means of maintaining the needed cardiac output. Blood pressure is likely to change little because the increase in vasodilation tends to offset the increase in cardiac output.

In the vascular phase of acute inflammation, initial vasoconstriction is followed rapidly by which assessment finding?

Heat and redness Explanation: The vascular phase of acute inflammation is characterized by changes in the small blood vells at the site of injury. It begins with momentary vasoconstriction followed by rapid vasodilation, which allows more blood and fluid into the area of injury, resulting in congestion, redness, and warmth. As fluid moves out of the vessels, stagnation of flow and clotting of blood occurs.

A nurse is planning care for a client coming into the emergency department via ambulance on a hot summer day with the following symptoms: temperature of 105°F (40.5°C), absence of sweating, and loss of consciousness. The nurse anticipates that the client has which condition?

Heatstroke Explanation: Symptoms of heatstroke include a body temperature greater than 104°F (40°F), absence of sweating, and central nervous system abnormalities such as delirium, convulsions, and loss of consciousness.

Which assessment supports the finding of lead toxicity?

Hemoglobin 9 g/dL (90 g/L) Explanation: Anemia is a cardinal sign of lead toxicity. Lead competes with the enzymes required for hemoglobin synthesis and with the membrane-associated enzymes that prevent hemolysis of red blood cells. The other findings are not necessarily found in lead toxicity.

The nurse is caring for a client with diabetes. Which of these findings is cause for concern, leading the nurse to initiate client education?

Hemoglobin A1C level is 8.7%. Explanation: Glycated hemoglobin test (hemoglobin A1c) should be 6% to 7% in clients with diabetes. Maintaining the desired level reduces complications of diabetes.

During the acute inflammatory response, there is a period called the transient phase where there is increased vascular permeability. What is considered the principal mediator of the immediate transient phase?

Histamine Explanation: Histamine causes dilation of arterioles and increases the permeability of venules. It acts at the level of the microcirculation by binding to histamine 1 receptors on endothelial cells and is considered the principal mediator of the immediate transient phase of increased vascular permeability in the acute inflammatory response. Arachidonic acid is a 20-carbon unsaturated fatty acid found in phospholipids of cell membranes. Release of arachidonic acid by phospholipases initiates a series of complex reactions that lead to the production of the eicosanoid family of inflammatory mediators (prostaglandins, leukotrienes, and related metabolites). Fibroblasts and cytokines are not the principal mediator of the transient phase of an acute inflammatory response.

A nurse is counseling a client on how recessive disorders are transmitted from parent to child. The nurse knows that recessive disorders occur when which allele is present?

Homozygous recessive Explanation: Recessive disorders occur when the client has homozygous (two copies) of the recessive allele present.

Which physiologic process allows hormones to exert influence upon some cells and not others?

Hormone receptors Explanation: Hormone receptors are complex molecular structures (usually proteins) that are located either on the cell surface or inside target cells. The structure of these receptors is specific to a particular hormone, which allows target cells to respond to one hormone and not to others. For example, receptors in the thyroid are specific for thyroid-stimulating hormone, and receptors on the gonads respond to the gonadotropic hormones. Positive feedback control occurs when rising levels of a hormone cause another gland to release a hormone that is stimulating to the first. Protein binding describes the ability of proteins to form bonds with other substances. Pituitary-hypothalamic feedback describes negative feedback.

When hypofunction of an endocrine organ is suspected, which type of diagnostic test can be administered to measure and assess target gland response?

Hormone stimulation Explanation: Stimulating hormone can be given to identify (determine hypofunction) if the target gland is able to increase hormone response to increased stimulation. Agglutination with enzymes is a way of measuring hormone antigen levels. The 24-hour urine sample measures hormone metabolite excretion. Radioactive hormone-antibody binding levels provide a method of measuring plasma levels.

During periods of fasting and starvation, the glucocorticoid and other corticosteroid hormones are critical for survival because of their stimulation of gluconeogenesis by the liver. When the glucocorticoid hormones remain elevated for extended periods of time, what can occur?

Hyperglycemia Explanation: In predisposed persons, the prolonged elevation of glucocorticoid hormones can lead to hyperglycemia and the development of diabetes mellitus and starvation. They stimulate gluconeogenesis by the liver, sometimes producing a 6- to 10-fold increase in hepatic glucose production. A prolonged increase in glucocorticoid hormones does not cause hepatomegaly, portal hypertension, or adrenal hyperplasia.

A health care provider is assessing a client for a potential endocrine disorder. Assessment findings identify abnormalities with emotion, pain, and body temperature. Which mechanism of endocrine control will require further laboratory/diagnostic assessment?

Hypothalamus Explanation: The hypothalamus is the coordinating center of the brain for endocrine, behavioral, and autonomic nervous system function. It is at the level of the hypothalamus that emotion, pain, body temperature, and other neural input are communicated to the endocrine system. The anterior pituitary regulates several physiologic processes, including stress, growth, reproduction, and lactation. The cerebellum is involved in motor control, and the cerebral cortex is associated with sensory, motor, and association.

Which gland acts as a signal-relaying bridge between multiple body systems and the pituitary gland?

Hypothalamus Explanation: The activity of the hypothalamus is regulated by both hormonally mediated signals (e.g., negative feedback signals) and by neuronal input from a number of sources. Neuronal signals are mediated by neurotransmitters such as acetylcholine, dopamine, norepinephrine, serotonin, gamm-aminobutyric acid (GABA), and opioids. Cytokines that are involved in immune and inflammatory responses, such as the interleukins, also are involved in the regulation of hypothalamic function. This is particularly true of the hormones involved in the hypothalamic-pituitary-adrenal axis. Thus, the hypothalamus can be viewed as a bridge by which signals from multiple systems are relayed to the pituitary gland. This cannot be said of the other options.

Which gland acts as a signal relaying bridge between multiple body systems and the pituitary gland?

Hypothalamus Explanation: The activity of the hypothalamus is regulated by both hormonally mediated signals (e.g., negative feedback signals) and by neuronal input from a number of sources. Neuronal signals are mediated by neurotransmitters such as acetylcholine, dopamine, norepinephrine, serotonin, gamma-aminobutyric acid (GABA), and opioids. Cytokines that are involved in immune and inflammatory responses, such as the interleukins, also are involved in the regulation of hypothalamic function. This is particularly true of the hormones involved in the hypothalamic-pituitary-adrenal axis. Thus, the hypothalamus can be viewed as a bridge by which signals from multiple systems are relayed to the pituitary gland. This cannot be said of the other options.

An infant whose mother had myxedema during the pregnancy has failed to meet standards for growth and is developmentally delayed. Which hormonal imbalance is this child exhibiting?

Hypothyroidism Explanation: Thyroid hormone is necessary for metabolism at all ages, as well as growth and development during childhood. Uncorrected thyroid insufficiency in childhood leads to cretinism, a condition with marked physical and intellectual disability. Myxedema is the term used for thyroid insufficiency in adults.

A client is admitted with an alteration in arterial blood gases. Cellular injury is most likely to result from which aspect of this abnormality?

Hypoxia Explanation: Lack of oxygen or hypoxia is a common cause of cell injury. Hypocapnea, alkalemia, and hypercapnea are not typical mechanisms of injury. The other common major mechanisms of cell injury are free radical formation, adenosine triphosphate depletion, and disruption of intracellular calcium homeostasis.

A 70-year-old client is being treated for a recent hemorrhagic stroke that has left the client with deficits. These deficits are most likely the result of which mechanisms of cell injury?

Hypoxia and adenosine triphosphate (ATP) depletion Explanation: Stroke is characterized by impaired cerebral circulation and consequent death of neurons from cellular hypoxia. Free radical injury, abnormal DNA synthesis, and impaired calcium homeostasis are not direct consequences of lack of blood flow to body cells.

The nurse is conducting a physical assessment of a homeless man during a night when the wind chill factor is -10°F (-23°C). When assessing the man's fingers and toes for frostbite, the nurse looks for which type of cellular injury?

Hypoxic Explanation: Exposure to cold increases blood viscosity/thickness and induces vasoconstriction by direct action on blood vessels and through reflex activity of the sympathetic nervous system. The resultant decrease in blood flow may lead to hypoxic tissue injury, depending on the degree and duration of cold exposure.

A client with common variable immunodeficiency (CVID) is admitted to the hospital for treatment. Which type of treatment will the nurse be administering to the client?

IV immunoglobulin Explanation: Treatment methods for CVID are similar to those used for other primary humoral immunodeficiencies, with IVIg being the mainstay of therapy.

In an allograft collection of stem cells, which match has the lowest risk of rejection?

Identical HLA types matching 3 of 6 HLA loci Explanation: Donors with identical HLA types, matched for at least 3 of 6 HLA loci, are associated with the least risk of graft-versus-host disease or graft rejection. While loci match is important, 6 of 6 loci are not required. A distant relative is not a guarantee of the required match.

The nurse is assisting a new mother with breastfeeding. What does the nurse understand is a primary secretory immunoglobulin (Ig) that is found in the colostrum?

IgA Explanation: IgA is primarily a secretory Ig that is found in saliva, tears, colostrum, and bronchial, gastrointestinal, prostatic, and vaginal secretions. Because it is found in secretions, its primary function is in local immunity on mucosal surfaces. IgA prevents the attachment of viruses and bacteria to epithelial cells.

The nurse is aware that the only class of immunoglobulins to cross the placenta is:

IgG Explanation: IgG is the only class of immunoglobulins to cross the placenta. Levels of maternal IgG decrease significantly during the first 3 to 6 months of life, while infant synthesis of immunoglobulins increases.

A premature neonate in the ICU suspected of having an infection has blood drawn for class specific antibodies. Which of the following confirms that the neonate has developed a congenital infection?

IgM antibodies are elevated. Explanation: Serology samples are used to identify infectious organisms by measuring the levels of antibodies. The measurement of antibody titers has another advantage in that specific antibody types such as IgM and IgG are produced by the host during different phases of an infectious process. IgM-specific antibodies generally rise and fall during the acute phase of the disease, whereas the synthesis of the IgG class of antibodies increases during the acute phase and remains elevated until or beyond resolution. Measurements of class-specific antibodies are also useful in the diagnosis of congenital infections. IgM antibodies do not cross the placenta, but certain IgG antibodies are transferred passively from mother to child during the final trimester of gestation. Consequently, an elevated level of pathogen-specific IgM antibodies in the serum of a neonate must have originated from the child and therefore indicates congenital infection. A similarly increased IgG titer in the neonate does not differentiate congenital from maternal infection.

A client received a liver transplant and is now taking immunosuppressant medication. The client has been told that a potential side effect of the therapy is the development of cancer. Select the option that best supports this information.

Immune surveillance hypothesis Explanation: The immune surveillance hypothesis suggests that the development of cancer might be associated with impairment or decline in the surveillance capacity of the immune system. Increases in cancer incidence have been observed in people with immunodeficiency diseases and in those with organ transplants who are receiving immunosuppressant drugs. The other options will not cause immunosuppression.

An adult client is scheduled for testing of a suspected growth hormone (GH)-secreting tumor. Which result from the glucose suppression test would confirm the condition?

Increased GH secretion Explanation: A suppression test is intended to determine if an organ that is oversecreting will respond to feedback to suppress that hormone. The glucose suppression test is used for an adult with acromegaly from excessive GH production. Normally, glucose would cause a decrease in GH secretion, but there is increased production of GH in response to the glucose in the test when the client has acromegaly.

The client is receiving chronic glucocorticoid therapy for chronic obstructive pulmonary disease (COPD). Which lab result would the provider expect to see?

Increased blood glucose Explanation: Cortisol, a glucocorticoid, maintains blood glucose levels (increases them) by antagonizing the effects of insulin. It also suppresses hematopoiesis, protein and collagen synthesis, and immune responses (decreases white blood count).

What will the nurse assess as the end result of activation of the renin-angiotensin-aldosterone system?

Increased blood pressure Explanation: Stimulation of the sympathetic nervous system activates the renin-angiotensin-aldosterone system, which mediates an increase in vascular tone and renal retention of sodium and water. This causes an increase in blood pressure. The other assessment findings would not be the end result of activation of this system.

A college student has just learned that her latest Papanicolaou test revealed atypical cells, a fact that has resulted in stress and an accompanying release of angiotensin II. How would the effects of this hormone be objectively manifested?

Increased blood pressure Explanation: The central nervous system (CNS) effects of angiotensin II include the release of vasopressin and a consequent increase in blood pressure. Increased respiratory rate often accompanies stress, but this is not a direct effect of angiotensin II. Oxygen saturation and blood sugar are objective measures that are unlikely to decrease.

The nurse is discussing positive feedback mechanisms. Which example best explains this mechanism?

Increased estradiol production causes increased follicle-stimulating hormone (FSH) production. Explanation: The positive feedback mechanism regulates hormones as follows: a rising level of a hormone causes another gland to release a hormone that is stimulating to the first. The other choices represent negative feedback mechanisms of control.

The nurse knows that chronic pain lacks which of the characteristic pain-related reactions?

Increased heart rate Explanation: Characteristics of chronic pain do not include autonomic responses like increased heart and respiratory rate. Loss of appetite, disturbed sleep patterns, and depression are common among people dealing with chronic pain.

If a client has a bacterial infection in the blood, the nurse will note which laboratory value that correlates with this?

Increased neutrophils Explanation: Bacterial infections produce a relatively selective increase in neutrophils, whereas parasitic and allergic response induce eosinophilia. Viral infections tend to produce a decrease in neutrophils (neutropenia) and an increase in lymphocytes. A decrease in white blood cells (leukopenia) may occur with overwhelming infection. Pyrogens are not capable of blocking viral replication or prostaglandin release.

A client has been exposed to ultraviolet (UV) radiation. Which effect from the exposure is the nurse's primary concern?

Increased risk of cancer Explanation: UV radiation contains increasingly energetic rays that are powerful enough to disrupt intracellular bonds and increase the serious risk of skin cancers. Sunburn, increased wrinkles, and decreased lubrication are less serious in nature.

A nurse is caring for a child diagnosed with DiGeorge syndrome with thymic hypoplasia. Which immune response would the nurse anticipate in this child?

Increased risk of infection Explanation: The thymus is essential to the development of the immune system because it is responsible for the production of mature, immunocompetent T lymphocytes. Mature, immunocompetent T-helper and T-cytotoxic cells leave the thymus in 2 to 3 days and enter the peripheral lymphoid tissues through the bloodstream. Without a thymus, infections would occur frequently. These T cells recognize foreign antigens and build an immune response. Therefore, with thymic hypoplasia, there is a risk for increased infection (not reduced infection or no change). Obviously, there is an antigen response.

A client is experiencing significant stress while awaiting the results of her recent lymph node biopsy. Among the hormonal contributors to this response is a release of aldosterone, resulting in which physiologic effect?

Increased sodium absorption Explanation: Mineralocorticoids such as aldosterone increase sodium absorption by the kidneys. Changes in insulin release and cardiac contractility are mediated by catecholamines, whereas cortisol potentiates the action of epinephrine.

A client diagnosed with metabolic syndrome and growth hormone (GH) deficiency will likely display which physical assessment finding?

Increased visceral fat Explanation: GH deficiency is associated with a cluster of cardiovascular risk factors including central adiposity (associated with increased visceral fat), insulin resistance, and dyslipidemia. These features also are associated with the metabolic syndrome.

A client attending a health fair asks how anthrax will be treated in the event of a bioterrorism attack. How should the nurse respond?

Individuals who are exposed will be isolated and given antibiotics. Explanation: Potential agents to be used in a bioterrorism attack include the bacillus that causes anthrax. Anthrax spreads by spores found in infected secretions, soil, or animal products. This microbe is susceptible to antibiotic treatment. Disinfecting the area will not treat the infected individual.

In describing the ideal analgesic, what factors would be included? Select all that apply.

Inexpensive Have minimal adverse effects Effective Explanation: The ideal analgesic would be effective, nonaddictive, and inexpensive. In addition, it would produce minimal adverse effects and not affect the person's level of consciousness.

A febrile, 3-week-old infant is currently undergoing a diagnostic workup to determine the cause of the fever. Which statement best conveys the rationale for this careful examination?

Infants are susceptible to serious infections because of their decreased immune function. Explanation: Fever in infants and children can be classified as low risk or high risk, depending on the probability of the infection progressing to bacteremia or meningitis and signs of toxicity. Infants between the ages of 1 to 28 days with fever should be considered to bave a bacterial infection that can cause bacteremia or meningitis. Younger children have decreased immunologic function and are more commonly infected with virulent organisms. Infants are at particularly high risk for serious bacterial infections that can cause bacteremia or meningitis. Infants are capable of thermoregulation, and fever is not necessarily indicative of a congenital disorder. Antipyretics are effective in the treatment of fever in infants.

A client with a history of cancer and recent chemotherapy was hospitalized with fever, weakness, shortness of breath and productive cough. A bronchoscopy with transbronchial biopsy showed granulomas containing the fungus Histoplasma. Based on client history and the biopsy results, a nurse can conclude that the most likely cause of this infection was due to which reason?

Inhalation of fungi and decreased host defenses permitting an opportunistic infection. Explanation: Few fungi are capable of causing diseases in humans, and most of these are incidental, self-limited infections of skin and subcutaneous tissue. Serious fungal infections are rare and usually initiated through puncture wounds or inhalation. Despite their normally harmless nature, fungi can cause life-threatening opportunistic diseases when host defense capabilities have been disabled. Disease states or antibiotic therapy can upset the balance of normal body flora, permitting fungal overgrowth and setting the stage for opportunistic infections.

Allostasis incorporates which characteristics when applied to the human body?

Interactive physiologic changes in numerous systems Explanation: Allostasis describes the interactive physiologic changes in the neuroendocrine, autonomic, and immune systems that occur in response to either real or perceived challenges to homeostasis. This concept has been used to measure the cumulative effects of stress on humans. Organ-specific responses are a part of the general adaptation syndrome response to stress. Control systems detect changes in physiologic function and regulate the physical and chemical set points.

Despite the low levels of radiation used in contemporary radiologic imaging, a radiology technician wants to minimize personal exposure to ionizing radiation. What is the primary rationale for the technician's precautions about radiation exposure?

Interferes with DNA synthesis and mitosis Explanation: Radiation has a damaging effect on DNA synthesis and mitosis, a process that is especially harmful to rapidly dividing cells. Radiation does not directly influence the action potential of cells or the accumulation of endogenous waste products. Cell changes such as hypertrophy or hyperplasia may result from radiation exposure, but such changes are secondary to interference with DNA synthesis and mitosis.

A client has been diagnosed with osteomyelitis and admitted to the hospital. The client's fever persists throughout most of the day but returns to normal at least twice a day. Which pattern of fever is this client displaying?

Intermittent Explanation: Intermittent fever patterns are very changeable, but they do return to normal at least once every 24 hours. A remitting fever pattern temperature does not return to normal and varies a few degrees in either direction. In a sustained fever pattern, the temperature remains above normal with minimal variations. A relapsing fever is one in which there is one or more episodes of fever, each as long as several days, with 1 or more days of normal temperature between episodes.

A client's temperature readings are as follows: 99.6°F (37.5°C) at 4 pm; 102°F (38.9°C) at 8 pm; and 97.9°F (36.6°C) at 12 am. The nurse's hand-off should include which note?

Intermittent fever Explanation: The nurse's hand-off report should include the presence of intermittent fever—a temperature that returns to normal at least once every 24 hours.

A client diagnosed with posttraumatic stress disorder (PTSD) tells the nurse about having "flashbacks" during waking hours or nightmares in which the past traumatic event is relived, often in vivid and frightening detail. What term for this experience will the nurse document in the client's record?

Intrusion Explanation: Intrusion involves recurrent, involuntary distressing memories or "flashbacks" during waking hours or nightmares in which the past traumatic event is relived, often in vivid and frightening detail. The client did not report avoidance, which refers to the attempt to avoid situations, including people, places, activities, and objects that arouse distressing memories; nor did the client report marked alterations in arousal and reactivity, which include inability to experience positive emotions such as love, joy, pleasure, or satisfaction.

The client tells the provider that she is afraid to sleep due to nightmares of a recent plane crash that seem real. Which state of posttraumatic stress disorder (PTSD) is the client experiencing?

Intrusion Explanation: PTSD, formerly called battle fatigue, is characterized by a constellation of symptoms that are experienced as states of intrusion, avoidance, and hyperarousal. Intrusion refers to the occurrence of "flashbacks" during waking hours or nightmares in which the past traumatic event is relived, often in frightening detail. Avoidance refers to emotional numbing. Hyperarousal refers to the presence of increased irritability, difficulty concentrating, an exaggerated startle reflex, etc.

A client has been diagnosed with cancer that was a result of dysfunctional apoptosis. The health care provider explains the process to the multidisciplinary client care team. Select the best explanation.

It allows for DNA-damaged cells to survive. Explanation: Apoptosis is considered a normal cellular response to DNA damage; loss of normal apoptotic pathways may contribute to cancer by enabling DNA-damaged cells to survive.

The nucleus is called the center of the cell because it has the ability to do which of the following?

It contains the DNA that is essential for protein synthesis to keep the cell alive. The nucleus contains the DNA that is essential to the cell because its genes contain the information necessary for the synthesis of proteins that the cell must produce to stay alive. The messenger RNA copies and carries the DNA instructions. The mitochondria transforms compounds into energy.

The rehabilitation nurse is caring for a client who is recovering from a cerebrovascular accident (CVA) with hemiplegia. The family asks the nurse if the paralysis will be permanent. Upon which of these physiologic rationales should the nurse base the response?

It is likely that paralysis is permanent, as nerve cells do not normally regenerate. Explanation: Labile cells continue to divide and replicate throughout life, replacing cells that are continually being destroyed. Stable cells are those that normally stop dividing when growth ceases; however, they are capable of regeneration when confronted with an appropriate stimulus. Permanent or fixed cells, including nerve cells, skeletal muscle cells, and cardiac muscle cells cannot undergo mitotic division. Once destroyed, they are replaced with fibrous scar tissue. Exudate into a wound area is generally short-lived; however, once damaged, the nerve cells cannot regenerate.

Which of the following is the purpose of messenger RNA (mRNA)?

It is necessary for protein synthesis. Explanation: mRNA provides the information needed for placing amino acids in their proper order for each specific type of protein.

Which statement is true concerning DNA structure and function?

It is very stable. Explanation: The DNA that contains genetic information is extremely stable. Because of its stable structure, the genetic information carried in DNA can survive the many stages of cell division and the day-to-day process of cell renewal and tissue growth. It also allows the information to survive the many processes of reduction division, the fertilization process, and the mitotic cell divisions involved in the formation of a new organism.

The nurse is caring for a client with an accumulation of 2.5 cm of darkened tissue scar over the area of a 3-mm injury. How does the nurse correctly document this finding in the medical record?

Keloid Explanation: Keloids are tumor-like masses caused by excess production of scar tissue. The tendency toward development of keloids is more common in blacks and seems to have a genetic basis.

What is a characteristic indicator that an individual is in the latent phase of HIV?

Lack of symptoms Explanation: The three phases experienced are primary infection phase, chronic asymptomatic or latency phase, and overt AIDS phase. Mononucleosis-like symptoms and high viral loads are seen in the primary infection phase, whereas the AIDS phase is characterized with opportunistic infections.

When caring for a client who has developed bacterial pneumonia, the nurse assesses for which of these abnormalities that supports presence of infection?

Leukocytosis Explanation: Bacterial infections produce a relatively selective increase in neutrophils (neutrophilia), a type of white blood cell, whereas parasitic and allergic responses induce eosinophilia. Viral infections tend to produce a decrease in neutrophils (neutropenia) and an increase in lymphocytes (lymphocytosis). Thrombocytes or platelets assist in blood clotting. Neutropenia refers to a decrease in neutrophils, a type of WBCs, during times of immunosuppression or immunocompromise.

The living part of the cell found both inside and outside of the nucleus is called protoplasm. In addition to proteins, carbohydrates, electrolytes, and water, what other substance is found in protoplasm?

Lipids Protoplasm is composed of water, proteins, lipids, carbohydrates, and electrolytes. Lipids make up 2% to 3% of the protoplasm. Lipids are the main parts of cell membranes surrounding the outside and inside of cells. Lysosomes contain membrane-enclosed sacs filled with hydrolytic enzymes. Acid hydrolases are structures that require an acid environment. Peroxisomes contain the enzymes needed for breaking down very long-chain fatty acids.

While lecturing about what happens to undigested materials that remain in the cytoplasm, the instructor will emphasize which materials as residual bodies? Select all that apply.

Lipofuscin granules in the heart muscle cells of an older adult client Tattoo pigments from a tattoo received during a teenage prank Explanation: Undigested materials may remain in the cytoplasm as residual bodies or be extruded from the cell. In some long-lived cells, such as neurons and heart muscle cells, large quantities of residual bodies accumulate as lipofuscin granules or age pigments. Other indigestible pigments, such as inhaled carbon particles and tattoo pigments, also accumulate and may persist in residual bodies for decades.

While teaching a science class, the instructor mentions that both autocrine and paracrine hormonal actions occur without entering the bloodstream. A student aks, "What cells do paracrine actions affect?" Which response is correct?

Local Explanation: Paracrine actions are hormonal interactions with local cells other than those that produce the hormone; autocrine actions are with self-cells (cells from which they were produced). Both autocrine and paracrine hormonal actions affect target cells. Neither paracrine nor autocrine actions affect cell storage.

A nursing student who is studying cancer cells identifies which of the following to be the best definition of anaplasia?

Loss of cell differentiation in cancerous tissue Explanation: The term "anaplasia" is the loss of cell differentiation in cancerous tissue. Rapid proliferation of cancer cells refers to a rapid increase in number of cells, while decreased proliferation is a decrease in the growth or decrease in the number of cells. An increase in cell differentiation refers to the process by which a less specialized cell becomes a more specialized cell type. This is not what happens during anaplasia.

The cardinal signs of inflammation include swelling, pain, redness, and heat. What is the fifth cardinal sign of inflammation?

Loss of function Explanation: These signs are rubor (redness), tumor (swelling), calor (heat), and dolor (pain). In the second century AD, the Greek physician Galen added a fifth cardinal sign, functio laesa (loss of function). Altered level of consciousness is not a cardinal sign of inflammation. Sepsis and fever are systemic signs of infection.

In some cancers, the presenting factor is an effusion, or fluid, in the pleural, pericardial, or peritoneal spaces. Research has found that almost 50% of undiagnosed effusions in people not known to have cancer turn out to be malignant. Which cancers are often found because of effusions?

Lung and ovarian cancers Explanation: Lung cancers, breast cancers, and lymphomas account for about 75% of malignant pleural effusions. Reports of abdominal discomfort, swelling, and a feeling of heaviness and an increase in abdominal girth, which reflect the presence of peritoneal effusions or ascites, are the most common presenting symptoms in ovarian cancer, occurring in up to 65% of women with the disease.

Select the organelle that is responsible for the breakdown of excess and worn-out cell parts as well as foreign substances that are taken into the cell.

Lysosomes Explanation: Lysosomes break down excess and worn-out cell parts as well as foreign substances that are taken into the cell. Peroxisomes contain enzymes that are used in oxidative reactions. Proteasomes are responsible for proteolysis of malformed and misfolded proteins. Ribosomes serve as sites for protein synthesis in the cell.

A client with hereditary angioneurotic edema (HAE) is experiencing an attack. Which priority intervention should the nurse be prepared to administer?

Maintaining a patent airway Explanation: Laryngeal edema is a life-threatening manifestation that can lead to complete airway obstruction and death without intervention. Although vomiting may occur due to the swelling of the structures of the gastrointestinal mucosa, the priority intervention would be maintaining a patent airway.

A nursing instructor sees the need for further instruction when one student makes which statement concerning cancer?

Malignant neoplasms tend to grow slowly. Explanation: Malignant neoplasms, which invade and destroy nearby tissue and spread to other parts of the body, tend to grow rapidly and spread widely and have the potential to cause death.

A client who has just recently been diagnosed with cancer asks the nurse what tumor markers are. Which answer would be the nurse's best response?

Markers are antigens expressed on the surface of tumor cells. Explanation: Tumor markers are antigens expressed on the surface of tumor cells or substances released from normal cells in response to the presence of a tumor.

Which maternal factor contributes to an infant having macrosomia, hypoglycemia, and hyperbilirubinemia?

Maternal diabetes Explanation: A mother with diabetes is more likely to have a large body size (macrosomia), especially if the diabetes is uncontrolled. Other fetal abnormalities include hypoglycemia, hypocalcemia, polycythemia, and hyperbilirubinemia. The other factors contribute primarily to the infant being small for gestational age (SGA). The other disorders do not necessarily lead to a high birth weight.

An obstetrics nurse is counseling an expectant mother. The mother is concerned about letting people hold her baby once the baby is born, fearing that the infant will get sick. What should the nurse explain to the mother?

Maternal immunoglobulins cross the placenta and protect the newborn early in life. Explanation: Passive immunity is immunity transferred from another source. An infant receives passive immunity naturally from the transfer of antibodies from its mother in utero and through breast milk. Maternal IgG crosses the placenta and protects the newborn during the first few months of life. Innate immunity are not effective yet, and protection is unrelated to the amount of lymphoid tissue.

Which characteristic of genetic disorders involves a single-gene trait?

Mendelian patterns of inheritance Explanation: Single-gene disorders are characterized by patterns of transmission that follow the Mendelian patterns of inheritance. Multifactorial inheritance involves more than one gene mutation, rearrangement of groups of genes, and uneven numbers of some chromosomes in each cell.

Which of the following statements is true concerning ribonucleic acid (RNA)?

Messenger RNA is the template for protein synthesis. Explanation: Messenger RNA is the template for protein synthesis. Ribosomal RNA is the physical structure in the cytoplasm where protein synthesis takes place. Transfer RNA is a clover-shaped molecule. RNA is synthesized in the nucleus and then moved into the cytoplasm.

A couple is scheduled to obtain familial lineage studies. The nurse is providing education. To which aspect of the DNA analysis should the woman pay particular attention regarding inherited disorders?

Mitochondrial DNA Explanation: Mitochondrial DNA is inherited maternally (i.e., from the mother) and provides a basis for familial lineage studies. Extrachromosomal DNA is used in the synthesis of mitochondrial RNAs and proteins used in oxidative metabolism. G-protein-linked receptors mediate cellular responses for numerous types of first messengers, including proteins, small peptides, amino acids, and fatty acid derivatives such as the prostaglandins.

A nurse confers with the attending physician about a client with encephalopathy who has been attributed to mitochondrial gene mutation. Why do mutations of the mitochondria affect the brain?

Mitochondrial mutations affect the ability of tissues to synthesize ATP, and tissues with high demands malfunction without enough ATP. Explanation: Tissues that have a high demand for the energy provided by ATP malfunction very quickly when ATP cannot be produced fast enough. This includes the brain, the muscular system, the auditory system, and the retina. Mitochondria cannot be silenced by the cell. Mitochondrial proteins and mutations can affect the cell where they reside but not the entire organ.

A client has developed a large wound as a result of trauma sustained during an auto accident. The cells will undergo which process to regenerate tissue?

Mitosis Explanation: Mitosis is the cell cycle process in which nongerm cells are replicated, providing ways for the body to replace cells that have a limited life span—skin and blood cells; increase tissue mass during periods of growth; and repair tissue, such as in wound healing. Cytogenetics is the study of the structure and numeric characteristics of the cell's chromosomes. Meiosis is limited to replicating germ cells and results in the formation of gametes or reproductive cells. Cloning is a general term for the research activity that creates a copy of same biological entity.

Which treatment regimen is most likely to result in stable blood glucose levels for a client with a diagnosis of type 1 diabetes?

Monitor blood glucose levels throughout the day and administer exogenous insulin replacement as needed Explanation: Because of the loss of insulin response, all people with immune-mediated type 1 diabetes require exogenous insulin replacement to reverse the catabolic state, control blood glucose levels, and prevent ketosis.

Systemic lupus erythematosus is best characterized by which principle?

Most clients have antinuclear antibodies present in their blood. Explanation: Clients diagnosed with systemic lupus erythematosus will most commonly have antinuclear antibodies in their blood. Lupus is primarily a disease of women and it is an autoimmune reaction.

Cancer prognosis has improved most dramatically when treatment plans include which of the following?

Multiple types of treatment Explanation: Treatment plans that use more than one type of therapy, often in combination, are providing cures for a number of cancers that a few decades ago had a poor prognosis, and in turn are increasing the life expectancy. The remaining options can be components of a multi-pronged treatment plan.

Which disorder does the nurse know is considered to be an autoimmune disease?

Myasthenia gravis Explanation: Autoimmune disorders are those in which the immune system attacks body tissues. Some autoimmune disorders attack specific body systems while others are systemic. Systemic autoimmune disorders include rheumatoid arthritis, scleroderma, and systemic lupus erythematosus. Other autoimmune diseases include idiopathic thrombocytic purpura, Hashimoto thyroiditis, Type 1 diabetes mellitus, myasthenia gravis, primary biliary cirrhosis, temporal arteritis, Crohn's disease, and ulcerative colitis. Rhabdomyolysis is a disorder of muscle breakdown that typically follows injury or medications such as statins. Diverticulitis is inflammation of outpouchings of the colon. Osteoarthritis is joint destruction from wear and tear.

An overweight, 14-year-old boy feels tired all the time. He sleeps 12 to 14 hours a day and has a voracious appetite but no energy to burn off the calories. He has been diagnosed with hypothyroidism brought about by the accumulation of a nonpitting mucosus type of edema. For which life-threatening condition should his care team be prepared?

Myxedema coma Explanation: Myxedema implies the presence of a nonpitting mucous type of edema caused by an accumulation of a hydrophilic mucopolysaccharide substance in the connective tissues throughout the body. The hypothyroid state may be mild, with only a few signs and symptoms, or it may progress to a life-threatening condition called myxedema coma. Pheochromocytoma is a tumor of the adrenal gland. Thyroid storm is related to hyperthyroidism. Paraneoplastic syndrome is a separate condition.

What is the most common mechanism of hormone control?

Negative feedback Explanation: With negative feedback, the most common mechanism of hormone control, some feature of hormone action directly or indirectly inhibits further hormone secretion so that the hormone level returns to an ideal level or set point.

When thyroxine (T4) in the thyroid is low, it triggers the pituitary to increase thyroid stimulating hormone (TSH), which then increases T4 secretion. This is an example of which of the following?

Negative feedback mechanism Explanation: In a negative feedback mechanism, when the monitored function or value decreases below the set point of the system, the feedback mechanism causes the function or value to increase. In a positive feedback mechanism, the initiating stimulus produces more of the same. Neuroendocrine control systems that influence behavior are called the stress response. The general adaptation syndrome is a manifestation of the body's attempt to adapt to stimuli.

What feedback system provides stability to the stress response?

Negative feedback mechanism Explanation: Most control systems in the body operate by negative feedback mechanism. This provides more stability than a positive feedback mechanism.

Which is an abnormal mass of tissue in which the growth exceeds and is uncoordinated with that of the normal tissues?

Neoplasm Explanation: An abnormal mass of tissue in which the growth exceeds and is uncoordinated with that of the normal tissues is called a neoplasm. A transformation occurs when a cell has become cancerous. A mutation is an alteration in the DNA that may or may not result in a transformation. An insertion is a type of mutation.

The tissue that is damaged in Huntington chorea is which of the following?

Nerve tissue Explanation: Huntington chorea is a neurodegenerative disorder. It does not involve the other types of tissue.

The manifestations of the stress response are strongly influenced by which system?

Nervous Explanation: The manifestations of the stress response are strongly influenced by both the nervous and endocrine systems.

The nurse is teaching a class of pregnant woman about nutrition, emphasizing their need for folic acid. The nurse's rationale for this relates to prevention of which birth defect?

Neural tube defects Explanation: Folic acid deficiency has been implicated in the development of neural tube defects (anencephaly, spina bifida, encephalocele). Studies have shown a significant decrease in neural tube defects when folic acid was taken long term by women of reproductive age.

Although stress exposure initiates integrated responses by multiple systems, the functional changes are first manifested in which body system?

Neuroendocrine Explanation: Exposure to stress activates an immediate response by the neuroendocrine system that plays a role in most of the responses to stress and attempts to adapt. The coordinated physiologic system response to stress is meant to increase the probability of survival, but also designed to be an acute response. Therefore, the hormones that are released during the response do not persist long enough to cause damage to vital tissues. Subsequently, all other body systems become part of the response.

An adolescent presents at the clinic with reports of pedunculated lesions projecting from the skin on the trunk area. The nurse knows that this is a sign of:

Neurofibromatosis type 1 Explanation: In more than 90% of persons with neurofibromatosis type 1, cutaneous and subcutaneous neurofibromas develop in late childhood or adolescence. The cutaneous neurofibromas, which vary in number from a few to many hundreds, manifest as soft, pedunculated lesions that project from the skin. Marfan syndrome affects several organ systems, including the ocular system (eyes), the cardiovascular system (heart and blood vessels), and the skeletal system (bones and joints). Down syndrome is a congenital condition characterized by varying degrees of intellectual disability and multiple defects. Klinefelter syndrome is a condition that occurs in men who have an extra X chromosome in most of their cells. The syndrome can affect different stages of physical, language, and social development. The most common symptom is infertility.

A nurse caring for a client with a genetic disorder of mitochondrial DNA assesses for abnormalities in which body system?

Neuromuscular Explanation: Mitochondrial DNA disorders, although rare, affect the tissues of the neuromuscular system.

A school nurse is teaching a class on immunity. Which statement contains an accurate explanation about cellular defenses?

Neutrophils engulf invading organisms where lysosomes break them down. Explanation: Phagocytosis is when a cell, such as macrophages or neutrophils, engulfs an invading organism or damaged cells. Once inside the cell, the encapsulated particle is broken down by lysosomal enzymes.

What will conduct injurious stimuli to alert the body to potential damage?

Nociceptors Explanation: Nociceptors are sensitive to painful and noxious stimuli and alert the system to injury. Thermoreceptors will perceive heat, proprioceptors will perceive body position, and odorant receptors will perceive the sensation of smell.

The client experienced a traumatic event in the past that has caused chronic nightmares in which the event is relived. The client may have increased levels of which of the following?

Norepinephrine Explanation: People with chronic posttraumatic stress disorder (PTSD) have been shown to have increased levels of norepinephrine and increased activity of alpha-adrenergic receptors. The other options are not increased in PTSD. Reference:

The locus caeruleus (LC) is an area in the brain stem that produces which hormone?

Norepinephrine (NE) Explanation: Central to the neural component of the neuroendocrine response to stress is an area of the brain stem called the locus caeruleus (LC). The LC is densely populated with neurons that produce norepinephrine. Cortisol, a glucorticoid, is produced in the adrenal cortex. Aldosterone, a mineralocorticoid, is produced in the adrenal cortex. Antidiuretic hormone is produced in the hypothalamus and posterior pituitary.

A client is experiencing stress as a nurse prepares to insert a peripheral intravenous catheter into his forearm. The client's locus caeruleus (LC) is consequently producing which hormone?

Norepinephrine (NE) Explanation: The LC is densely populated with neurons that produce norepinephrine (NE); it does not produce CRF, ADH, or ACTH.

A student arrives at the health clinic anxious and afraid. The student found an enlarged lymph node in the groin area that is extremely painful to touch and "knows" it is cancer. What information should the health care provider relay to this student about lymphadenitis?

Not all swollen lymph nodes are due to cancer. It could be caused by an infection in the genital region. Explanation: Localized acute and chronic inflammation may lead to a reaction in the lymph nodes that drain the affected area. This response represents a nonspecific response to mediators released from the injured tissue or an immunologic response to a specific antigen. Painful, palpable nodes are commonly associated in inflammatory processes, whereas nonpainful lymph nodes are more characteristic of neoplasms. If the student has experienced an increase in sexual activity, he or she should be evaluated for a sexually transmitted or other genital infection.

A two-day postoperative client's temperature was 98.5°F (36.9°C) at 3:00 pm. At 6:00 pm, the unlicensed assistant (UAP) notifies the nurse that the client's temperature is 102°F (38.9°C). Which action should the nurse take?

Notify the physican. Explanation: The nurse should contact the physician, as the increase in the client's temperature is outside of the normal range and/or the normal diurnal variation in temperature.

What are the three main parts of a typical cell? Select all that apply.

Nucleus Cell membrane Cytoplasm Explanation: When seen under a light microscope, three major components of the cell become evident: the nucleus, the cytoplasm, and the cell membrane. Cell walls and vacuoles are found in plants and bacteria.

The nurse is caring for a client who has AIDS and suffers from wasting syndrome. What is the priority nursing consideration for this client?

Nutrition Explanation: The wasting syndrome is an AIDS-defining illness and is common in people with HIV infection or AIDS. Wasting syndrome is characterized by involuntary weight loss of at least 10% of baseline. Treatment for wasting includes nutritional interventions.

Following a knee replacement, a client develops an infection at the site. After months of unsuccessful anti-infective therapy, the surgeon removed the implant. What explanation can the nurse provide when the client asks why the anti-infective therapy did not work?

Organisms on a colonized implant produce a self-protecting biofilm. Explanation: If infected, artificial heart valves, joint replacements, and other implanted medical devices can become colonized. The infecting organism produces a matrix of cells and capsular material that prevents the infection from being cleared from the body. At that point, the implant must be removed to clear the infection.

Although both eukaryotes and prokaryotes are capable of causing infectious diseases in humans, eukaryotes are unique because they have a/an:

Organized nucleus Explanation: Eukaryotic cells have an organized nucleus. Many prokaryotes contain extra chromosomal pieces of circular DNA (plasmids). Prokaryotic cells have a flexible lipid cytoplasmic membrane. Both types of cells vary in shape and size.

What happens as a cell's workload declines? Select all that apply.

Oxygen consumption decreases. Protein synthesis decreases. Cell size decreases. Explanation: As the cell's workload declines, oxygen consumption decreases, protein synthesis decreases, and cell size decreases. Insulin levels do not increase, and energy expenditure would decrease.

Which tract in the spinal cord conducts the diffuse, dull, aching sensations that are associated with chronic and visceral pain?

Paleospinothalamic tract Explanation: The paleospinothalamic tract is a slower-conducting, multisynaptic tract concerned with the diffuse, dull, aching, and unpleasant sensations that commonly are associated with chronic and visceral pain.

There has been an incidence of influenza in both Europe and in the United States. What is the term that best describes this incidence?

Pandemic Explanation: A pandemic incidence refers to the spread of disease beyond continental boundaries, while an epidemic describes an abrupt and unexpected increase in the incidence of disease over endemic rates.

A nursing instructor is teaching a group of students about the action of hormones. The instructor determined that teaching was effective when the students recognize the local action of hormones as:

Paracrine Explanation: When hormones act locally on cells other than those that produced the hormone, the action is called paracrine. Hormones can also exert an autocrine action on the cells in which they were produced. Pancreatic and hormonal are not actions.

Which statement is true concerning human cell division?

Parent cell divides and each daughter cell recives chromosomes identical to the parent. Explanation: Cell division, or mitosis, is the process during which a parent cell divides and each daughter cell receives chromosomes identical to the parent. The series of mitotic divisions is called cleavage. Gene activity is needed for cell division and cell replication.

A woman experiences a viral infection while pregnant. Which type of immunity does an infant have at birth against this infection?

Passive Explanation: Passive immunity is when the antibodies against an antigen are transferred directly to the host, such as when a fetus gains from the mother's immune system. Active immunity is acquired through exposure to antigens, and having the B and T lymphocytes develop a response. Tolerance is the ability of the immune system to react to foreign substances but not the body cells. Adaptive immunity is when the immune system responds to antigens and is moderated by B and T lymphocytes.

A big difference in the treatment of childhood cancer as opposed to adult cancer is that chemotherapy is the most widely used treatment therapy for childhood cancer. What is the reason for this?

Pediatric tumors are more responsive to chemotherapy than adult cancers. Explanation: Chemotherapy is more widely used in the treatment of children with cancer than in adults because children better tolerate the acute adverse effects, and in general, pediatric tumors are more responsive to chemotherapy than adult cancers. Children are very adaptable and tolerate more forms of cancer treatment than adults do. Children do complain about the nausea and vomiting chemotherapy can cause, just like adults do, and they do not like losing their hair, just like adults.

A 33-year-old client who is a long-term intravenous user of heroin has been recently diagnosed with hepatitis C. Which portal of entry most likely led to the client's infection?

Penetration Explanation: Direct inoculation from intravenous drug use, and the accompanying disruption in the integrity of the body's surface barrier, is an example of entry by penetration. Vertical transmission occurs from mother to child, and neither direct contact nor ingestion is a likely source of this client's infection.

A nurse is completing a genetic map of a client's family. The client has brown eyes and his wife has blue eyes. Both the son and the daughter have brown eyes. Which of the following is the same in both children?

Phenotype Explanation: Both the son and the daughter have the same phenotype, which is defined as the recognizable traits associated with a specific genotype.

Mechanical forces that produce tissue trauma would be classified as which type of cellular injury?

Physical agent Explanation: Cell injury can be caused by a number of agents, including physical agents, chemicals, biologic agents, and nutritional factors. Among the physical agents that generate cell injury are mechanical forces that produce tissue trauma, extremes of temperature, electricity, radiation, and nutritional disorders. Chemical agents can cause cell injury through several mechanisms: they can block enzymatic pathways, cause coagulation of tissues, or disrupt the osmotic or ionic balance of the cell. Biologic agents differ from other injurious agents in that they are able to replicate and continue to produce injury. Among the nutritional factors that contribute to cell injury are excesses and deficiencies of nutrients, vitamins, and minerals.

A client who lives with many psychosocial and physiologic stressors is experiencing the exhaustion stage of stress. The client is likely to develop what consequence of this stage?

Physiologic damage Explanation: The exhaustion stage of stress causes physiologic damage. This can culminate in death, but this is not guaranteed. Emotional balance will not occur and heightened awareness accompanies the alarm stage.

Which structure controls the functions of the greatest number of target glands and cells?

Pituitary gland Explanation: The pituitary gland has been called the master gland because its hormones control the functions of many target glands and cells. It supersedes the importance of the thyroid, adrenal cortex, or pancreas in this regulatory role.

The cell membrane is also called what?

Plasma membrane The cell membrane is often called the plasma membrane. The nuclear membrane is another type of membrane within the cell. The cell membrane provides receptors for hormones and other biologically active substances; it is not a receptor membrane. A main structural component of the membrane is its lipid bilayer. It is not a bilayer membrane.

A client with hyperthyroidism arrives in the emergency department with a fever of 104ºF (40°C), tachycardia, chest pain, and crackles in bilateral lung bases. What treatment options does the nurse anticipate being prescribed in order to remove the thyroid hormone? Select all that apply.

Plasmapheresis Dialysis Hemoperfusion Explanation: The client is exhibiting signs of excess thyroid hormone, or thyroid storm. Thyroid hormones can be removed by plasmapheresis, dialysis, or hemoperfusion absorption. Aspirin increases the level of free thyroid hormone and should not be used during thyroid storm. Levothyroxine will increase the level of thyroid hormone.

What is a common opportunistic infection in AIDS clients?

Pneumocystis jiroveci pneumonia (PCP) Explanation: PCP is a very common opportunistic infection in individuals with AIDS. It is a common bacteria found in households and does not routinely effect someone with a healthy immune system. Chancroid and syphilis are infections that do not occur commonly in the population in general, including those with AIDS. Myalgia is not an infection, but it is muscle pain, a common symptom of an infection.

Select the most common symptoms of diabetes. Select all that apply.

Polydipsia Polyuria Polyphagia Explanation: The most commonly identified signs and symptoms of diabetes are often referred to as the three polys: (1) polyuria (i.e., excessive urination), (2) polydipsia (i.e., excessive thirst), and (3) polyphagia (i.e., excessive hunger). Polyhydramnios is a medical condition describing an excess of amniotic fluid in the amniotic sac. Polycythemia is a condition of increased red blood cells.

Most human traits are determined by multiple pairs of genes, many with alternate codes, accounting for some dissimilar forms that occur with certain genetic disorders. What type of inheritance involves multiple genes at different loci, with each gene exerting a small additive effect in determining a trait?

Polygenic inheritance Explanation: Polygenic inheritance involves multiple genes at different loci, with each gene exerting a small additive effect in determining a trait. Multifactorial inheritance is similar to polygenic inheritance in that multiple alleles at different loci affect the outcome; the difference is that multifactorial inheritance includes environmental effects on the genes. Monofactorial inheritance is nonexistent as is collaborative inheritance.

Select the statement that best describes polygenic inheritance.

Polygenic traits involve multiple genes. Explanation: Polygenic inheritance involves multiple genes at different loci, with each gene exerting a small additive effect in determining a trait. Most human traits are determined by multiple pairs of genes, many with alternate codes, accounting for some dissimilar forms that occur with certain genetic disorders. Polygenic traits are predictable, but with less reliability than single-gene traits. Multifactorial inheritance is similar to polygenic inheritance in that multiple genes at different loci affect the outcome; however, environmental effects on the genes also affect the outcome.

A client receives steroids for several months to treat an inflammatory condition. Which action by the primary health care provider indicates an understanding of the negative feedback mechanism when the client no longer needs the medication?

Prescribing a tapering dose of the medication over weeks Explanation: Negative feedback occurs when secretion of one hormone causes a reduction in the secretion of the hormone that stimulates production of the first hormone. In this case, adrenocorticotropic hormone (ACTH) manufactured by the anterior pituitary would normally stimulate release of cortisol, but with the increase of cortisol produced by the secreting tumor, enough cortisol already floods the system that there should be a reduction in the ACTH level. Sudden withdrawal of the medication would leave the client without glucocorticoids and risk an Addisonian crisis. Gradual withdrawal of the medication allows the pituitary to measure the drop in cortisol levels and begin secreting ACTH.

When testing nociceptive stimuli to elicit a withdrawal reflex in the body, what stimuli are commonly used?

Pressure from a sharp object Explanation: Stimuli used to elicit a withdrawal reflex include pressure from a sharp object, strong electric current to the skin, or application of heat or cold of approximately 10°C above or below normal skin temperature.

A continuing education nurse in a long-term care facility is discussing wound healing in older adult clients. Because older adult clients are more likely to have comorbidities like problems with mobility, diabetes, or vascular problems, the nurse should assess the clients for which problems? Select all that apply.

Pressure wounds on buttocks Impaired healing related to diabetes Ischemic ulcer formation in feet Explanation: Older adults are more vulnerable to chronic wounds—chiefly pressure, diabetic, and ischemic ulcers—than younger persons, and these wounds heal more slowly. However, these wounds are more likely due to other disorders (e.g., immobility, diabetes mellitus, or vascular disease) rather than aging.

Which best describes the number of active disease cases at any given time?

Prevalence Explanation: Disease prevalence indicates the number of active cases at any given time. The term incidence is used to describe the number of new cases of an infectious disease that occurs within a defined population over an established period of time. A disease is considered endemic in a particular geographic region if the incidence and prevalence are expected and relatively stable. An epidemic describes an abrupt and unexpected increase in the incidence of disease over endemic rates.

As part of maintaining homeostasis, why are hormones, secreted by endocrine cells, continuously inactivated?

Prevent accumulation Explanation: Continuous inactivation of secreted hormones is necessary to prevent accumulation that could disrupt the feedback mechanism. Increased secretion stimulates production of more receptor sites. Metabolic waste absorption is not a function of the endocrine system.

The nurse caring for a postoperative client documents that the surgical incision is healing by:

Primary intention Explanation: The nurse would document the surgical wound as healing by primary intention as there is no tissue loss. Wounds healing from secondary intention are larger and have a greater loss of tissue and contamination. Wounds do not heal by tertiary intention or systemic intention.

In the usual course (stages) after a pathogen has entered the host body, the stage when the host initially develops the appearance of signs/symptoms like a mild fever and body aches is:

Prodromal Explanation: The prodromal stage follows inoculation (the initial stage) and is identified by the initial onset of symptoms in the host. Tissue inflammation and damage is evident during the acute (3rd) stage. Pathogen elimination and containment are characteristics of the convalescent (4th) period, which follows the acute stage.

A client has a mild headache and fatigue. He also states he has some aches and pains. Which stage of fever does the nurse determine the client is experiencing?

Prodrome Explanation: During the first or prodromal period there are nonspecific complaints such as mild headache and fatigue, general malaise, and fleeting aches and pains.

There are two forms of endoplasmic reticulum (ER) found in a cell. They are the rough and the smooth ER. What does the rough ER do in a cell?

Produces proteins Explanation: Rough ER is studded with ribosomes attached to specific binding sites on the membrane. Proteins produced by the rough ER are usually destined for incorporation into cell membranes and lysosomal enzymes or for exportation from the cell. The rough ER segregates (rather than combines) these proteins from other components of the cytoplasm and modifies their structure for a specific function. Rough ER does not transport anything through the cell membrane. Rough ER is studded with ribosomes; it does not destroy them.

Which statement best describes the major difference between prokaryotes and eukaryotes?

Prokaryotes lack an organized nucleus. Explanation: Bacteria are autonomously replicating unicellular organisms known as prokaryotes because they lack an organized nucleus. Compared with nucleated eukaryotic cells, the bacterial cell is small and structurally primitive. Similar to eukaryotic cells but unlike viruses, bacteria contain both DNA and RNA, although their genome is considerably smaller than eukaryotes and typically is encoded on a single chromosome.

A client with a history of brain tumors that resulted in partial removal of the pituitary gland years ago expresses concern to the health care provider about whether she will be able to breast-feed her infant. Which physiologic function of the pituitary gland facilitates breast milk production?

Prolactin Explanation: The anterior pituitary gland or adenohypophysis contains five cell types: (1) thyrotrophs, which produce thyrotropin, also called TSH; (2) corticotrophs, which produce corticotropin, also called ACTH; (3) gonadotrophs, which produce the gonadotropins, LH and FSH; (4) somatotrophs, which produce GH; and (5) lactotrophs, which produce prolactin that is involved with breast growth and milk production.

A client sustained an injury 3 days ago. The nurse is assessing the status of the wound and anticipates the wound to be in which phase of healing?

Proliferative Explanation: The proliferative phase begins within 2 to 3 days of an injury and focuses on building new tissue to fill the wound. The inflammatory phase begins at the time of the injury occurrence. The remodeling phase occurs approximately 3 weeks after the injury. There is not a collagen phase.

Which sign of congenital hypothyroidism in a neonate is one of the first signs the nurse would pick up on?

Prolonged period of physiologic jaundice Explanation: With congenital lack of the thyroid gland, the infant usually appears normal and functions normally at birth because of hormones supplied in utero by the mother. Prolongation of physiologic jaundice, caused by delayed maturation of the hepatic system for conjugating bilirubin, may be the first sign. There may be respiratory difficulties and a hoarse cry, feeding difficulties, and an enlarged abdomen. This condition will not interfere with meconium passage; will not result in elevated ICP resulting in full, tight fontanels; and will not result in a palpable mass in the neck.

What is the priority intervention for a nurse caring for a client with wasting syndrome from HIV?

Promote nutritional intake Explanation: Wasting syndrome is defined as an AIDS-related illness with the manifestations of involuntary weight loss of at least 10% of baseline, diarrhea or chronic weakness and fever, but no other infections or neoplasms. It is treated with nutritional interventions such as oral supplements or enteral or parenteral nutrition.

Hyperbaric treatment for wound healing is used for wounds that have problems in healing due to hypoxia or infection. It works by raising the partial pressure of oxygen in plasma. How does hyperbaric oxygen treatment enhance wound healing?

Promotion of angiogenesis Explanation: An increase in tissue oxygen tension by hyperbaric oxygen enhances wound healing by a number of mechanisms, including the increased killing of bacteria by neutrophils, impaired growth of anaerobic bacteria, and promotion of angiogenesis and fibroblast activity. Eosinophil activity is not affected by hyperbaric treatment of wounds.

Several hormones, including growth hormone (GH) and thyrotropin-releasing hormone (TRH), are bound to and carried by which substance?

Proteins Explanation: Some hormones, such as steroids and thyroid hormone, are bound to protein carriers for transportation to the target cell destination. Cholesterol is a precursor for steroid hormone. Prohormones have an extra amino acid and convert to hormones in the Golgi complex.

When trying to explain hypothyroidism to a newly diagnosed client, the nurse stresses the fact that the thyroid hormone is transported in blood by specific:

Proteins Explanation: Some hormones, such as steroids and thyroid hormone, are bound to protein carriers for transportation to the target cell destination. The extent of carrier binding influences the rate at which hormones leave the blood and enter the cells. Cholesterol is a precursor for steroid hormone. Prohormones have an extra amino acid and are converted to hormones in the Golgi complex.

Which agent is the cause of malaria?

Protozoan parasite Explanation: Malaria is caused by a microscopic, unicellular protozoa, common in water supplies of developing countries.

Select the most accurate statement regarding measurements of urinary hormone.

Provide a better measure of hormone levels during a designated period. Explanation: Measurements of urinary hormone or hormone metabolite excretion often are done on a 24-hour urine sample and provide a better measure of hormone levels during that period than hormones measured in an isolated blood sample. The advantages are relative ease of obtaining urine samples and blood sampling is not required. The disadvantages are that timed urine collections often are difficult to obtain and urine samples may be accidentally discarded or inaccurately preserved; drugs or disease states that alter hormone metabolism may interfere with the test results.

Which of the following is the purpose of attempting to develop a HapMap (haplotype map)?

Providing a link between genetic variations and common diseases Explanation: An international effort has been organized to develop a HapMap of these variations to provide a link between genetic variations and common complex diseases such as cancer, heart disease, diabetes, and some forms of mental disease.

A nurse is monitoring the temperature of a client with hypothermia during the rewarming process. Which location for the temperature probe will provide the most accurate results?

Pulmonary artery Explanation: Temperatures in different parts of the body vary according to how close the probe is to the body core. Skin is the coolest. Rectal temperatures are considered the most accurate for general use. However, when a client's temperature is changing rapidly as in someone being rewarmed, or treated for acute hyperthermia, core body temperature is best measured by a probe near the heart and thoracic organs.

A client in the intensive care unit has been having rapidly changing core body temperatures that requires close monitoring. The client is intubated and has a pulmonary artery catheter and a Foley catheter. The nurse should obtain and/or record the temperature from which of the following?

Pulmonary artery catheter Explanation: The pulmonary artery catheter is the preferred measurment when body temperatures are rapidly changing and need to be monitored closely on a critically ill person in an intensive care unit.

Select the option that best describes the nitrogenous group that has two nitrogen rings and includes adenine and guanine.

Purine base Explanation: The purine bases, adenine and guanine, have two nitrogen rings. Thymine and cytosine are pyrimidine bases.

The nurse screening for diabetes mellitus at a health fair obtains these results. Which client should be referred to a primary health care provider for further evaluation?

Random blood glucose 195 mg/dL (10.82 mmol/L) Explanation: Laboratory values that are considered normal are hemoglobin A1C less than 6.5 percent, fasting plasma glucose of (FPG) less than 100 mg/dL or less than 140 mg/dL 2 hours after an oral glucose tolerance test (GTT). Urine should be free of ketones. A hemoglobin A1C value that is greater than or equal to 6.5 percent, a fasting blood glucose greater than 126mg/dL, or a blood glucose level greater than 200 mg/dL 2 hours after a glucose tolerance test (GTT) indicate diabetes mellitus. Values between these levels are considered to place clients at increased risk for diabetes mellitus and require further evaluation. A random blood glucose level is expected to correlate with the two-hour GTT results and should be below 200 mg/dL.

Which manifestation would a nurse expect when assessing a child with insufficient growth hormone (GH) secretion?

Rank below 10% on the growth chart Explanation: GH stimulates growth of bone and muscle and promotes protein synthesis and fat metabolism and decreased carbohydrate metabolism. A child lacking sufficient GH would demonstrate lack of growth but not the mental delays seen in childhood hypothyroidism.

The nurse is explaining to a client the adverse side effects of cancer therapies. Which target of both chemotherapy and radiation treatment accounts for the adverse as well as therapeutic effects of these treatments?

Rapidly proliferating cells Explanation: Chemotherapy and radiation treatment both preferentially affect rapidly proliferating cells, which includes some normal body cells, such as epithelial and hair-follicle cells, as well as cancer cells. These effects of chemotherapy and radiation therapy do not result from the fact that they affect hormones, blood vessels, or cell receptors.

The Golgi complex, or Golgi bodies, consists of stacks of thin, flattened vesicles or sacs within the cell. These Golgi bodies are found near the nucleus and function in association with the endoplasmic reticulum (ER). What is one purpose of the Golgi complex?

Receive proteins and other substances from the cell surface by a retrograde transport mechanism Recent data suggest that the Golgi apparatus has yet another function: it can receive proteins and other substances from the cell surface by a retrograde transport mechanism. Golgi bodies do not produce bile. They produce secretory, not excretory, granules, and they produce large carbohydrate molecules rather than small ones.

Homozygotes are what people are called in whom the two alleles of a given pair are the same (AA or aa). Heterozygotes are what people are called who have different alleles (Aa) at a gene locus. What kind of trait is expressed only in homozygous pairing?

Recessive trait Explanation: A recessive trait is one that is expressed only when two homozygous people have a child. A dominant trait is one expressed in either a homozygous or a heterozygous pairing. A single-gene trait and a penetrant trait do not exist. However, single-gene inheritance does exist.

What is the initial step in the process of phagocytosis to degrade any bacteria and cellular debris?

Recognition and adherence Explanation: Phagocytosis is initiated by the recognition and binding of particles by specific receptors on the surface of phagocytic cells. This binding is essential for trapping the agent, which then triggers engulfment and activates the killing potential of the cell. Therefore, recognition and adherence occur first, followed by engulfment and then intracellular killing. Margination is the accumulation of leukocytes (not antigens) along the endothelial surface.

There are two criteria that have to be met in order for a diagnosis of an infectious disease to occur. What are these two criteria?

Recovery of probable pathogen and documentation of signs and symptoms compatible with an infectious process. Explanation: The diagnosis of an infectious disease requires two criteria: (a) the recovery of a probable pathogen or evidence of its presence from the infected sites of a diseased host, and (b) accurate documentation of clinical signs and symptoms compatible with an infectious process. Culture and sensitivity are the growing of microorganisms outside the body and the testing to see what kills it. Identifying a microorganism by microscopic appearance and Gram stain reaction are not the criteria for diagnosis. Serology, an indirect means of identifying infectious agents by measuring serum antibodies in the diseased host, and the quantification of those antibodies (an antibody titer) are not criteria for diagnosis.

The nurse needs to assess a 1-year-old child for fever. Which approach will produce the most accurate reading?

Rectal Explanation: Measurement of core body temperature is important when evaluating fever. The rectal route is considered the most accurate. In adults and older children, the oral route is lower, but still accurate; however, in young children the oral route may be unreliable. Forehead thermometers can predict trends, but are not as accurate as other routes. The axillary route requires up to 10 minutes for the temperature to register appropriately.

A nurse discussing screening options with a client at risk for colon cancer should include education about which screening methods? Select all that apply.

Rectal examination Fecal occult blood test Colonoscopy Explanation: The nurse should include rectal examinination, a fecal occult blood test, and colonoscopy as screening options for colon cancer. Transrectal ultrasonography and PSA testing are screening tests for prostate cancer.

Parents of a 4-year-old child discovered the child has been chewing and swallowing imported toy figurines that have tested positive for lead. Which blood test should the care team prioritize?

Red blood cell (RBC) levels Explanation: Anemia is a cardinal sign of lead toxicity. Consequently, assessment of the quantity and morphology of RBCs is paramount in cases of suspected lead toxicity. WBC and liver studies are not central to the care of this client. Lead is indeed nephrotoxic, and urea and creatinine levels are relevant to assessment, but the priority blood test is assessment of the RBCs.

The nurse would identify the presence of granulation tissue at a wound site by which characteristic?

Red, moist tissue Explanation: Granulation tissue is a glistening red, moist connective tissue that fills the injured area. The other options reflect the presence of scarring, infection, or necrosis.

Hormones are chemical messengers that provide which function in the body?

Regulate body functions Explanation: Hormones regulate and integrate body functions. Hormones act on specific target cells, but they cause a variety of effects on tissues. Hormones do not transport other substances; hormones are transported and present in body fluids at all times.

A client has been brought to the emergency department with a large, gaping wound from a farming accident. The client is critically ill and has required blood products and surgery to clean and close the wound. The nurse, mentoring a student nurse, explains the inflammatory phase, beginning by discussing the roles that macrophages play in wound healing. What are those roles? Select all that apply.

Release growth factors that stimulate epithelial cell growth Facilitate the body's ability to grow new vessels in the injured area Remove debris from the wound

A client who had an implantable cardioverter-defibrillator (ICD) returns the next week with a fever, chills, and elevated WBC. The physician suspects the wound is infected. If this wound does not respond to antibiotic therapy, the nurse can anticipate the client will undergo:

Removal of device Explanation: Wound infections are of special concern in persons with implantation of foreign bodies such as orthopedic devices (e.g., pins, stabilization devices), cardiac pacemakers, and shunt catheters. These infections are difficult to treat and may require removal of the device.

What instruction will the nurse provide to a client with a new diagnosis of anaphylactic reaction to peanuts?

Routinely wear medical alert identification. Explanation: Clients with anaphylaxis have type I hypersensitivity reactions that can be triggered by the presence of minute amounts of the antigen. They should always wear or carry medical alert identification. With history of anaphylaxis, they should also carry epinephrine syringes for emergency administration. Waiting until the client stops breathing would put the person at risk of sudden death. Hypersensitivity reactions do not decrease over time, but may become more severe.

Which microorganism can cross the placenta and enter the fetal circulation causing the most frequent fetal anomalies?

Rubella Explanation: TORCH stands for toxoplasmosis, other, rubella (German measles), cytomegalovirus, and herpes. This acronym can be used to remember the agents most frequently implicated in fetal anomalies. Strept thorat, common colds, and the flu can cause concern for the mother but rarely result in fetal anomalies.

The parents of an infant boy ask the nurse why their son was born with a cleft lip and palate. The nurse responds that cleft lip and palate are defects that are caused by many factors. The defect may also be caused by teratogens. Which teratogens can cause cleft lip and palate?

Rubella Explanation: This defect may be caused by the teratogen rubella and is often encountered in children with chromosomal abnormalities.

The mother of an infant born with profound intellectual disability and hearing loss tells the nurse that she had a viral infection in the first trimester of pregnancy. The nurse identifies which congenital infection as the cause of the fetal defects?

Rubella Explanation: When an infectious disease is transmitted from mother to child during gestation or birth, it is classified as a congenital infection. The most frequently observed congenital infections include toxoplasmosis (caused by the parasite Toxoplasma gondii), syphilis, rubella, cytomegalovirus infection, and HSV infections (the TORCH infections); varicella-zoster (chickenpox); parvovirus B19; group B streptococci (Streptococcus agalactiae); and HIV. The severity of congenital defects associated with these infections depends greatly on the gestational age of the fetus when transmission occurs, but most of these agents can cause profound intellectual disability and neurosensory deficits, including blindness and hearing loss.

A nurse is assessing a client for the classic signs of acute inflammation. The nurse would assess the client for:

Rubor, swelling, and pain Explanation: The classic signs of inflammation are rubor (redness), tumor (swelling), calor (heat), and dolor (pain). The remaining options are more characteristic of symptomatology resulting from circulatory dysfunction.

The nurse knows that the client with which complication of diabetes has the greatest risk for the development of foot ulcers?

Sensory neuropathy Explanation: Sensory neuropathy is a major risk factor for foot ulcers due to the fact that people have impaired pain sensation and can be unaware of foot injuries and infections. Autonomic neuropathy, microvascular disease, or diabetic ketoacidosis, while not affecting risk for foot injuries, suggest that the client's diabetes is inadequately controlled.

The nurse is caring for a client diagnosed with systemic inflammatory response syndrome. Which illness is likely responsible for this diagnosis?

Sepsis Explanation: In severe bacterial infections (sepsis), large quantities of microorganisms in the blood result in the production and release of enormous quantities of inflammatory cytokines and development of what is referred to as systemic inflammatory response syndrome. Systemic inflammatory response syndrome results in generalized vasodilation, increased vascular permeability, intravascular fluid loss, myocardial depression, and fatal circulatory shock.

A client is being evaluated for atopic dermatitis possibly caused by a latex-related allergic reaction. The nurse will review which lab results to determine if an allergy is present?

Serum IgE Explanation: IgE binds to mast cells and basophils and is involved in parasitic infections, as well as allergic and hypersensitivity reactions. Serum IgE causes the symptoms of allergic reactions and is elevated in type 1 hypersensitivity disorders.

Select the screening procedure performed on newborns to decrease the risk of intellectual disability and compromised neurological development.

Serum phenylalanine test Explanation: Newborn infants are routinely screened within 12 hours of birth for abnormal levels of serum phenylalanine to detect high levels that could cause irreversible intellectual disability. The other procedures that may be performed after birth would not be used as a screening tool for intellectual disability.

A client with diabetes carries insulin with him at all times. At 11:35, he obtains a blood glucose reading of 218 mg/dL (12.1 mmol/L) and self-administers a dose of insulin in anticipation of eating lunch at noon. What type of insulin did he most likely inject?

Short acting Explanation: Short-acting (regular) insulin is used to facilitate metabolism of the food that is being eaten. Premixed insulin is possible, but is not used as commonly. Intermediate- and long-acting insulin would have too distant an onset.

A parent arrives in the endocrinology clinic with her 8-year-old son, concerned about his rapid development and tall stature. What significant assessment finding does the nurse recognize is important to report to the physician related to the development of precocious puberty?

Significant genital enlargement Explanation: Diagnosis of precocious puberty is based on physical findings of early thelarche, adrenarche, and menarche. The most common sign in boys is early genital enlargement. Radiologic findings may indicate advanced bone age. People with precocious puberty are unusually tall for their age as children but short as adults because of the early closure of the epiphyses.

Which type of epithelium is found in the lining of blood vessels, lymph nodes, and alveoli of the lungs?

Simple squamous epithelium Explanation: Simple squamous epithelium is adapted for filtration, which works well on the blood vessels, lymph nodes, and alveoli of the lungs. Stratified squamous keratinized epithelium makes up the epidermis of the skin. A pseudostratified ciliated columnar epithelium with goblet cells forms the lining of most of the upper respiratory tract. Transitional epithelium is well adapted for the lining of organs that are constantly changing their volume, such as the urinary bladder.

What statement is true concerning the use of ATP [adenosine triphosphate] by skeletal muscles?

Skeletal muscles use ATP at a rapid rate during exercise. Explanation: Skeletal muscles use minimal amounts of ATP when relaxed, and use ATP at a rapid rate when exercising.

A nurse assessing for factors that impact a client's ability to adapt to stress includes assessing for which of the following? Select all that apply.

Sleep-wake cycle Hardiness Nutrition Physiologic reserve

A client with severe hypothyroidism is presently experiencing hypothermia. What nursing intervention is a priority in the care of this client?

Slow rewarming of the client to prevent vasodilation and vascular collapse Explanation: If hypothermia is present, active rewarming of the body is contraindicated because it may induce vasodilation and vascular collapse. Prevention is preferable to treatment and entails special attention to high risk populations, such as women with a history of Hashimoto thyroiditis.

Full localization, discrimination of intensity, and interpretation of somatosensory stimuli requires processing by which region of the brain?

Somatosensory cortex Explanation: The full localization, discrimination of the intensity, and interpretation of the meaning of the stimuli require processing by the somatosensory cortex. The anterolateral pathway gives off numerous branches that travel to the reticular formation of the brain stem; the branches provide the basis for increased wakefulness or awareness after strong somatosensory stimulation and for the generalized startle reaction that occurs with sudden and intense stimuli. They also stimulate autonomic nervous system. Ruffini end-organs are found in the skin and deeper structures and detect tissue vibration, heavy and continuous touch, and pressure. Stretch-sensitive receptors in the skin (i.e., Ruffini endings, Pacinian corpuscles, and Merkel cells) also signal postural information. In the thalamus, sensory information is roughly localized and perceived in a crude sense.

International travel has contributed to increased prevalence and incidence of nonindigenous diseases by increasing which of the following?

Sources of infection Explanation: Travel results in more sources of infection, but it does not necessarily influence portals of entry, virulence, and course of diseases.

Select the option that best describes the type of tissue that is capable of regeneration when appropriately stimulated.

Stable Explanation: Stable tissues contain cells that normally stop dividing when growth ceases. However, these cells are capable of undergoing regeneration when confronted with an appropriate stimulus and are thus capable of reconstituting the tissue of origin. Continuously dividing tissues or labile tissues are those in which the cells continue to divide and replicate throughout life, replacing cells that are continually being destroyed. The cells in permanent tissues do not proliferate. The cells in these tissues are considered to be terminally differentiated and do not undergo mitotic division in postnatal life.

A client has received an injection containing thyrotropin-releasing hormone (TRH) and is now being assessed for serum levels of thyroid-stimulating hormone (TSH). Which type of diagnostic testing is this client undergoing?

Stimulation testing Explanation: Introduction of TRH tests the pituitary gland's ability to produce TSH, and is an example of a stimulation test. Suppression testing examines a gland's response to a stimulus that would normally result in decreased hormone production. RIA and autoantibody testing are examples of direct and indirect measurement of serum levels of a hormone.

The physician suspects a client may be experiencing hypofunction of an endocrine organ. Select the most appropriate test to determine organ function.

Stimulation tests Explanation: Stimulation tests are used when hypofunction of an endocrine organ is suspected. Suppression tests are used when hyperfunction of an endocrine organ is suspected. Genetic testing is used for DNA analysis, and imaging may be used as a follow-up after the diagnosis.

A client is suspected of having a parasitic infection with roundworms. For what laboratory testing will the nurse prepare the client to most rapidly identify this infection?

Stool exam for ova Explanation: Cultures of microorganisms are commonly used to grow bacteria, fungi, and yeasts. Some protozoa and helminths can be cultured; however, the fastest and most common method for identifying parasitic worms is visible identification of the organisms, cysts, or ova from a client specimen. Cell cultures are grown on a specific type of media used for Chlamydiaceae, Rickettsiaceae, and human viruses. Antibody titers are useful for indirectly diagnosing diseases that cannot be cultured, such as hepatitis B. Gene sequencing is commonly used for identifying viruses such as HIV and hepatitis C.

A young, healthy adult helps lift a sofa to move it 25 feet. The nurse knows that which source of energy will the client most likely use in this process?

Stored ATP Explanation: Stored ATP, creatine phosphate, and muscle glycogen are among the first energy sources utilized during aerobic exercise. Amino acids, fatty acids, and glucose are utilized after prolonged periods of exercise.

The student is examining a tissue sample with a microscope. The student decides that it is squamous keratinized tissue. Which type of epithelial tissue is the student viewing?

Stratified epithelium Explanation: Stratified, squamous keratinized epithelium makes up the epidermis of the skin. Simple epithelium is found in the lining of blood vessels, lymph nodes, and alveoli of the lungs. Glandular epithelium is located in exocrine and endocrine glands. Neuroepithelium is located in in olfactory mucosa, retina, and the tongue.

A client with cirrhosis has just received a liver transplant. Tissue rejection can be best prevented by what means?

Suppression of CD8+ cytotoxic T cells Explanation: T-cell-mediated graft rejection is called cellular rejection and is induced by two mechanisms: destruction of graft cells by CD8+ cytotoxic T cells and delayed hypersensitivity reactions triggered by CD4+ helper T cells. Neutrophil, complement, and macrophages are nonspecific inflammatory cells.

A client with a long-standing diagnosis of Crohn disease has developed a perianal abscess. Which treatment will this client most likely require?

Surgical draining Explanation: Although antibiotics are likely to form a component of this client's treatment, abscesses most often require surgical draining. A pressure dressing or the use of antivirals is likely unnecessary.

A nurse has just learned that her child has a life-threatening complement disorder known as hereditary angioneurotic edema (HAE). Due to deficiency in C1-INH, the nurse needs to be prepared for which possible life-threatening clinical manifestation?

Swelling of the airway Explanation: HAE is a rare, life-threatening complement disorder that results from deficiency of C1-inhibitor (HAE-C1-INH). It is an inherited autosomal dominant trait that causes mutation in the 11th chromosome. Deficiencies in C1-INH result in uncontrolled release of vasoactive substances that promote vascular permeability. The net result is development of tissue swelling in the subcutaneous tissues of the extremities, face/torso, or upper airway and GI tract. Laryngeal edema is a life-threatening manifestation that can lead to complete airway obstruction and death without interventions.

The stress response involves the activation of which physiological systems? Select all that apply.

Sympathetic nervous system (SNS) HPA axis Immune system Explanation: The stress response involves the activation of several physiologic systems (sympathetic nervous system, the HPA axis, and the immune system) that work in a coordinated fashion to protect the body against damage from the intense demand made on it. The cardiovascular and respiratory systems are indirectly activated by the sympathetic nervous system.

The clinical picture, or presentation of a disease in the body, is called:

Symptomatology of the disease Explanation: The term symptomatology refers to the collection of signs and symptoms expressed by the host during the disease course. This is also known as the clinical picture or disease presentation. The virulence of the disease is its power to produce the disease. The source of the disease is the place where it came from. The diagnosis of the disease is the naming of the disease process in the body.

Ultimately, the inner mitochondrial membrane transports proteins for what purpose?

Synthesis of adenosine triphosphate (ATP) The outer mitochondrial membrane contains a large number of transmembrane porins, through which water-soluble molecules may pass. The inner membrane contains the respiratory chain enzymes and transport proteins needed for the synthesis of ATP. The mitochondrial membrane does not play a role in cell repair or growth.

A client has developed the facial appearance that is characteristic of myxedema, along with an enlarged tongue, bradycardia, and voice changes. Which treatment modality is most likely to benefit this client?

Synthetic preparations of T3 or T4 Explanation: Myxedema and the client's other signs are associated with hypothyroidism, which necessitates thyroid hormone replacement. Beta-adrenergic blocking drugs and antithyroid drugs are indicated in the treatment of hyperthyroidism, whereas treatments relevant to adrenal cortical function are not relevant to hypothyroidism.

Antinuclear antibodies are characteristic of:

Systemic lupus erythematosus Explanation: The presence of antinuclear antibodies is most commonly assoicated with systemic lupus erythematosus.

Which statement explains how T lymphocytes and B lymphocytes differ?

T lymphocytes mature in the thymus gland; B lymphocytes mature in the bone marrow. Explanation: T lymphocytes mature in the thymus gland and provide cell-mediated immunity. T lymphocytes activate other lymphocytes and phagocytes. B lymphocytes mature in the bone marrow and are responsible for forming the antibodies that provide humoral immunity. Both are amplified by cytokines.

A 42-year-old female client with breast cancer has a tumor that is minimal in size and extension, has minimal regional lymph nodes involved, and has no distant metastasis. Using the TNM system, the tumor is staged as:

T1, N1, and M0 Explanation: This client has T1—minimal size and extension of the tumor; N1—minimal regional lymph node involvement; and M0—no distant metastasis.

Hyperthyroidism that is inadequately treated can cause a life-threatening condition known as a thyroid storm. What are the manifestations of a thyroid storm? Select all that apply.

Tachycardia Delirium Very high fever Explanation: Thyroid storm is manifested by a very high fever, extreme cardiovascular effects (e.g., tachycardia, congestive failure, and angina), and severe CNS effects (e.g., agitation, restlessness, and delirium). The mortality rate is high. Very low fever and bradycardia are not manifestations of a thyroid storm.

A Jewish couple that is trying to conceive ask the nurse if they should receive genetic counseling. The nurse is aware that an autosomal recessive disorder that causes a deficiency in hexosaminidase A may be present. What disorder should the couple be tested for?

Tay-Sachs disease In Tay-Sachs disease, an autosomal recessive disorder, hexosaminidase A, which is the lysosomal enzyme needed for degrading the GM2 ganglioside found in nerve cell membranes, is absent. Ganglioside accumulates in many tissues, such as the heart, liver, and spleen. Its accumulation in the nervous system and retina of the eye causes the most damage.

The health care provider informs the client that she is at risk for developing an autosomal recessive disorder. For which disorder may the client be at risk?

Tay-Sachs disease Explanation: Tay-Sachs disease is the only one of these diseases that is an autosomal recessive disorder. All the others are autosomal dominant disorders.

A client's primary care provider has recommended biofeedback in an effort to address chronic stress and reduce the potential for complications. What will be the goal of this intervention?

Teaching the client to consciously control their own physiologic (body) functioning Explanation: Biofeedback is a technique in which an individual learns to control physiologic functioning. It does not use exercise alone, and it aims to resolve, not simply accommodate, stress. Various types of stimulation are used, but not all are tactile.

The psychologist is counseling a client who is overwhelmed by losing his job and the recent death of a parent. Which strategy would be effective for stress reduction?

Teaching the client to systematically contract and relax major muscle groups Explanation: Progressive muscle relaxation is one method of relieving musculoskeletal tension and decreasing sympathetic system activity (part of the stress response). It consists of systematic contraction and relaxation of the major muscle groups. Radio and television music is inappropriate for music therapy because of the inability to control selection of pieces that are played. Deficiencies or excesses of any nutrients can alter a person's health status and impair his ability to adapt. Social isolation also increases stress.

A client is experiencing anorexia, myalgia, arthralgia, headache, and fatigue. The nurse should assess for:

Temperature Explanation: Common clinical manifestations of fever include anorexia, myalgia, arthralgia, headaches, and fatigue; thus, the nurse should assess the client's temperature.

A nurse conducting a community education program on hyperthermia determines that the participants understand the information when they identify that the heat index is which of the following?

Temperature sensed when temperature and humidity are combined Explanation: The nurse determines that the participants understand the information when they identify that the heat index is the temperature sensed by the body when humidity and temperature are combined.

The nurse learns that different types of headaches respond to different therapies. Which headache is most responsive to nonpharmacologic therapy?

Tension Explanation: Tension-type headaches often are more responsive to nonpharmacologic techniques, such as biofeedback, massage, acupuncture, relaxation, imagery, and physical therapy, than other types of headache. For people with poor posture, a combination of range-of-motion exercises, relaxation, and posture improvement may be helpful. The other options are usually best treated with medications that focus on the cause of the pain.

What should the nurse administer to a client who was bitten by a stray cat?

Tetanus immunization Explanation: Tetanus immunization is recommended for an animal bite, especially for a stray cat. Oral steroids and one dose of antibiotics would not be beneficial to prevent an infection. The varicella vaccine prevents chicken pox.

Which responsibility of the extracellular matrix (ECM) is most accurate?

The ECM provides the scaffolding for tissue renewal. Explanation: The ECM provides the scaffolding for tissue renewal. Although the cells in many tissues are capable of regeneration, injury does not always result in restoration of normal structure unless the ECM is intact. Some of the proteases, such as the collagenases, are highly specific, cleaving particular proteins at a small number of sites. This allows for the structural integrity of the ECM to be retained while healing occurs.

DNA fingerprinting is based in part on recombinant DNA technology and in part on those techniques originally used in medical genetics to detect slight variations in the genomes of different individuals. These techniques are used in forensic pathology to compare specimens from the suspect with those of the forensic specimen. What is being compared when DNA fingerprinting is used in forensic pathology?

The banding pattern Explanation: Banding patterns are analyzed to see if they match. Four bases—guanine, adenine, cytosine, and thymine (uracil is substituted for thymine in RNA)—make up the alphabet of the genetic code. A sequence of three of these bases forms the fundamental triplet code used in transmitting the genetic information needed for protein synthesis. The small variation in gene sequence (termed a haplotype) is thought to account for the individual differences in physical traits, behaviors, and disease susceptibility. Chromosomes contain all the genetic content of the genome.

The nurse is teaching a client with diabetes about the signs and symptoms of hypoglycemia. The client asks, "Why will I get headache, disturbed behavior, coma, and seizures if it's my pancreas that's impaired?" Which response is the best explanation?

The brain relies on blood glucose as its main energy source. Explanation: Because the brain relies on blood glucose as its main energy source, hypoglycemia produces behaviors related to altered cerebral function. Headache, difficulty in problem solving, disturbed or altered behavior, coma, and seizures may occur. Hyperglycemia causes ketone breakdown. Hepatic glycogenolysis is stimulated by epinephrine, resulting in the raising of the level of blood glucose. However, that process is generally initiated by the fight-or-flight response, as opposed to the physiologic drop in blood glucose levels that stimulates glucagon secretion. Somogyi phenomenon is also known as rebound hyperglycemia. The Somogyi phenomenon describes a rebound high blood glucose level in response to low blood glucose.

An infant has a rare autosomal recessive disorder, ataxia-telangiectasia, that has recently been diagnosed. Which developmental milestone assists in diagnosing this disorder?

The child is beginning to walk. Explanation: Ataxia is the predominant neurodegenerative feature, which usually goes undiagnosed until the child beings to walk. Talking, dietary intake, and tooth development do not correlate with this diagnosis.

A client experiencing severe neck pain and fever comes to the emergency room and is diagnosed with bacterial meningitis. Knowing that the client's immune system is fighting the infection, the infectious disease nurse correctly identifies which pathway as the activation of the complement system?

The classical Explanation: The classical pathway is initiated by an antigen-antibody complex (either IgG or IgM mediated), which causes a specific reactive site on the antibody to be "uncovered" so that it can bind directly to the C1 molecule in the complement system. Once C1 is activated, a "cascade" of sequential reactions is set in motion. Initially, a small amount of enzyme is produced, but with activation of successive complement proteins successively increasing, concentrations of proteolytic enzymes are produced.

A client's history and physical indicate that the client is a carrier of sickle cell anemia. The nurse anticipates which of the following?

The client can transmit the disorder but does not exhibit symptoms. Explanation: Clients that are carriers of a disorder are able to transmit the disorder but do not exhibit signs and symptoms of that disorder.

A client has recently been diagnosed with chronic back pain that requires the daily use of hydromorphone, an opioid analgesic. For the first few weeks, the client achieved relief with 4 mg every 6 hours. However, the client now requires 6 mg doses to achieve the same effect. How should the nurse best interpret this phenomenon?

The client is developing opioid tolerance, which is expected Explanation: The client is developing a tolerance to the opioid, which is not unexpected and which is not synonymous with addiction. There may be a need to include nonpharmacologic interventions or NSAIDs, but these do not need to replace the opioid.

A client who has been exercising for a few weeks begins to sweat as soon as he exercises. What will the nurse determine from this assessment?

The client is reacting normally. Explanation: Exercise training helps the body adapt and the rate of sweat production increases to help regulate temperature. For instance, exercise can increase metabolic head production 10-fold. Thermoregulatory responses such as sweating simultaneously increase heat loss, thus keeping body temperature from rising dangerously high. The trained person begins to sweat sooner, often within 1 to 2 minutes of the start of exercise. The client is reacting normally.

The nurse is providing discharge instructions for a client with Graves disease who has ophthalmopathy. What should the nurse be sure to include in the instructions to decrease exacerbation of this clinical manifestation?

The client should be strongly urged not to smoke. Explanation: Ophthalmopathy can also be aggravated by smoking, which should be strongly discouraged. It is not necessary for the client to avoid contact with others. Alcohol is not contraindicated but should be limited when taking any medication regimen. The client should not adjust the doses of medications without first consulting the physician.

A teenager has been exposed to a person infected with chicken pox. After 2 weeks, the client has not contracted the virus. How is this possible? Select all that apply.

The client was vaccinated for chickenpox. The client was previously exposed to chickenpox. Explanation: Active immunity is acquired when the host mounts an immune response to an antigen either through the process of vaccination or from environmental exposure. It is called active immunity because it requires the host's own immune system to develop an immunologic response, including the development of memory. Passive immunity transferred from mother to fetus only lasts a few weeks to months following birth.

A pregnant client is just completing her first trimester and has opted for alpha-fetoprotein (AFP) and human chorionic gonadotropin (hCG) testing. The nurse should identify what implication from this client's decision?

The client wishes to know about her fetus' risk of having a trisomy. Explanation: AFP and hCG testing are most often performed to determine the risks of a trisomy. Multifactorial disorders are not revealed by this form of testing and the nurse should not presume that the client will choose to have an abortion if the results are positive. The use of AFP and hCG testing does not preclude the use of amniocentesis.

A client has presented to the emergency department after he twisted his ankle while playing soccer. Which assessment findings are cardinal signs that the client is experiencing inflammation? Select all that apply.

The client's ankle is visibly red The ankle appears to be swollen The ankle is warmer than the unaffected ankle The client is experiencing pain Explanation: The cardinal signs of inflammation are rubor (redness), tumor (swelling), calor (heat), and dolor (pain). Bleeding is not among the cardinal signs.

A client with a diagnosis of lung cancer has developed bone metastases resulting in severe and protracted pain. Which assessment components should the nurse prioritize when assessing the client's pain?

The client's subjective report of the character and severity of pain Explanation: Although objective signs of pain may or may not be evident, the priority component of any pain assessment is the client's self-report. Self-report is usually regarded as the most reliable estimate of pain.

The treatment prescribed for an autoimmune disorder is primarily dependent upon what?

The current manifestations of the disease and the mechanisms that cause the disease process Explanation: Treatment of autoimmune disorders is dependent upon the magnitude of the presenting manifestations and underlying mechanisms of the disease process. Since in many cases the pathophysiologic mechanisms are not always known, treatment may be purely symptomatic. While corticosteroids are often use, they are not always the first line of treatment. The client's age and medical history can be factors in treatment choices but are not the primary consideration.

Which statement is true concerning gene expression?

The degree to which a gene is active is called gene expression. Explanation: The degree to which a gene or particular group of genes are active is called gene expression. A phenomenon termed "induction" is an important process by which gene expression is increased. Gene repression is a process by which a regulatory gene acts to reduce gene expression. Whenever product levels decrease, gene transcription is increased.

What information should be included in the teaching plan of care for the parents of a child diagnosed with Tay-Sachs disease?

The disorder involves accumulation of abnormal lipids. Explanation: In Tay-Sachs disease, a genetic disorder, abnormal lipids accumulate in the brain and other tissues, causing motor and mental deterioration beginning at approximately 6 months of age, followed by death at 2 to 5 years of age.

The nurse is caring for a client with decreased serum protein levels secondary to liver failure. When administering medications that are highly protein bound, the nurse anticipates the resulting drug level will respond in which of these ways?

The drug level will be elevated as lack of protein allows more free drug to circulate. Explanation: Peptide hormones and protein hormones usually circulate unbound in the blood. Specific carrier proteins synthesized in the liver carry steroid hormones and thyroid hormone, for example. Drugs that compete with a hormone for binding with transport carrier molecules increase hormone action by increasing the availability of the active unbound hormone.

The oncologist is reviewing data to determine if a client diagnosed with a malignant tumor is at risk for developing metastasis. Select the option that places the client at greatest risk.

The establishment of a dedicated vascular system Explanation: Once in the distant site, the process of metastatic tumor development depends on the establishment of blood vessels and specific growth factors that promote proliferation of the tumor cells. Early detection and the aggressiveness of the cancer cells are factors in the speed of growth once the tumor has been established. General health has little impact on tumor development.

Which statement best identifies the functions of genes during an adaptive cellular response?

The expression of the differentiation genes is altered, but the operating gene remains unaffected. Explanation: In many adaptive cellular responses, the expression of the differentiation genes is altered, but that of the operating genes is unaffected.

A person's genotype can best be described as which of the following?

The genetic makeup of an individual Explanation: The genetic makeup of an individual is known as a genotype. Phenotype is the observable expression of a genotype in terms of morphologic, biochemical, or molecular traits. If the trait is expressed in the heterozygote (one member of the gene pair codes for the trait), it is said to be dominant. If it is expressed only in the homozygote (both members of the gene pair code for the trait), it is recessive.

A client undergoing an evaluation of hormone levels asks, "What regulates the hormone levels?" Which response by the nurse would be considered most accurate?

The hypothalamic-pituitary-target cell system Explanation: The levels of many of the hormones are regulated by feedback mechanisms that involve the hypothalamic-pituitary-target cell system. Positive feedback control refers to rising levels of a hormone that causes another gland to release a hormone that is stimulating to the first. The hypophyseal portal system connects the supraoptic and paraventricular nuclei of the hypothalamus with the posterior pituitary gland. Exogenous forms of hormones (given as drug preparations) can influence the normal feedback control of hormone production and release.

A child is born with dwarfism to normal-sized parents. The physician is explaining how growth hormone (GH) plays a central role in the increase in stature that characterizes childhood and adolescence. What is the first step in the growth hormone chain of events?

The hypothalamus secretes GHRH. Explanation: Like other pituitary functions, hypothalamic stimulation precedes hormone release. In the case of GH, stimulation is the result of GHRH by the hypothalamus. GH is then released by the pituitary gland, stimulating the liver to release IGFs, which ultimately causes the epiphyseal plates of long bones to grow.

The nurse is discussing the administration of GH with a parent of a child who has short stature caused by growth hormone (GH) deficiency. What should the nurse include when educating the parents about administration?

The medication will be given daily during the period of active growth and can be continued into adulthood. Explanation: GH is administered by daily subcutaneous injection during the period of active growth, and can be continued into adulthood.

A newborn has been lethargic, is not nursing well, and is basically looking ill. Following lab tests, it has been found that the newborn has IgM present in his blood. How should the nurse interpret this finding?

The presence of IgM suggests the infant has a current infection. Explanation: IgM is the first immunoglobulin to appear in response to antigen and is the first antibody type made by a newborn. This is diagnostically useful because the presence of IgM suggests a current infection in the infant by a specific pathogen. IgE is involved in inflammation, allergic responses, and combating parasitic infections. It binds to mast cells and basophils. The binding of antigen to mast cell- or basophil-bound IgE triggers these cells to release histamine and other mediators important in inflammation and allergies. IgA is found in saliva and tears, and is a primary defense against infections in mucosal tissues.

A student is feeling inside her backpack to find her mobile phone. There are a number of other items in the backpack. Which component of somatosensory conduction is most likely to provide the detailed sensory information that will help her distinguish her phone from other items?

The primary dorsal root ganglion neuron, dorsal column neuron, and the thalamic neuron Explanation: The discriminative pathway, which is associated with the fine touch of item discrimination, involves just three neurons: the primary dorsal root ganglion neuron, the dorsal column neuron, and the thalamic neuron. Slow-conduction and anterolateral pathways that involve the reticular activating system are associated with pain, thermal sensation, and indiscriminate touch.

A nurse in a genetic clinic reads that a client's karyotype includes a broken "p." The nurse interprets this to mean which of the following?

The short arm of the X chromosome is broken. Explanation: The "p" refers to the short arm of the X chromosome, thus the nurse's interpretation should be that the short arm of the X chromosome is broken.

A client with long-standing type 2 diabetes is surprised to see high blood sugar readings while recovering from an emergency surgery. Which factor may have contributed to the client's inordinately elevated blood glucose levels?

The stress of the event caused the release of adrenal cortical hormones. Explanation: Elevation of glucocorticoid levels (i.e., cortisol), such as during stressful events, can lead to derangements in glucose metabolism. Tissue trauma does not cause gluconeogenesis, and illness does not inhibit the action of glucagon. The dawn phenomenon is not a likely cause of the client's disruption in blood sugar levels.

Which of the following is the definition of cytogenetics?

The study of the structure and numeric characteristics of the cells' chromosomes Explanation: The study of cytogenetics is the study of the structure and numeric characteristics of the cells' chromosomes. The other statements are not true.

Like DNA, RNA is a long string of nucleotides encased in a large molecule. However, there are three aspects of its structure that makes it different from DNA. What are these aspects? Select all that apply.

The sugar in each nucleotide of RNA is ribose. RNA is a single-stranded molecule. RNA's thymine base is replaced by uracil. Explanation: RNA is a single-stranded rather than a double-stranded molecule. Second, the sugar in each nucleotide of RNA is ribose instead of deoxyribose. Third, the pyrimidine base thymine in DNA is replaced by uracil in RNA. All cells are supposed to have 23 pairs of chromosomes.

Each skeletal muscle is a discrete organ made up of hundreds or thousands of muscle fibers. Although muscle fibers predominate, substantial amounts of connective tissue, blood vessels, and nerve fibers are also present. What happens during muscle contraction?

The thick myosin and thin actin filaments slide over each other, causing shortening of the muscle fiber. Explanation: Thin and thick filaments are the two types of muscle fibers that are responsible for muscle contraction. The thin filaments are composed primarily of actin, and the thick filaments are composed of myosin. During muscle contraction, the thick myosin and thin actin filaments slide over each other, causing shortening of the muscle fiber, although the length of the individual thick and thin filaments remains unchanged. When activated by ATP, the cross-bridges swivel in a fixed arc, much like the oars of a boat, as they become attached to the actin filament. During contraction, each cross-bridge undergoes its own cycle of movement, forming a bridge attachment and releasing it, and moving to another site where the same sequence of movement occurs. This pulls the thin and thick filaments past each other. The calcium-calmodulin complex is in smooth muscle. It binds to and activates the myosin-containing thick filaments, which interact with actin.

What determines the number of mitochondria in a given cell type?

The type of activity the cell performs. The number of mitochondria in a given cell type is largely determined by the type of activity the cell performs and how much energy is needed to undertake the activity. The nutrition the cell receives, the gender of the organism, and the number of similar cells in the organism do not have any influence on the number of mitochondria present in any individual cell type.

Following a biopsy, a client has been diagnosed as having a benign neoplastic tumor. Which characteristic most likely applies to his tumor?

The well-differentiated, neoplastic cells are clustered in a single mass. Explanation: Benign tumors are composed of well-differentiated cells that resemble the cells of the tissues of origin and are characterized by a slow, progressive rate of growth that may come to a standstill or regress. They tend to exist in a single mass. Malignant tumors tend to be poorly differentiated, grow rapidly, secrete hormones or cytokines, and have the potential to break loose.

The nurse is caring for a client with an infected wound that is left to heal by secondary intention. Which observation does the nurse expect to make during assessment of the wound area?

The wound is healing slowly with epithelial and scar tissues present. Explanation: Wounds healing by secondary intention undergo wound contraction resulting in a scar considerably smaller than the original wound. Cosmetically, this may be desirable because it reduces the size of the visible defect. Keloids result from abnormal wound healing, resulting in tumor-like masses caused by excess production of scar tissue. Sutures are present in wounds healing by primary intention.

Which explanation identifies correctly how the G protein-linked receptors are similar?

They have a ligand-binding extracellular receptor component, which causes changes that activate the G protein on the cytoplasmic side of the cell membrane. Explanation: Although there are differences among the G protein-linked receptors, all share a number of features. They all have a ligand-binding extracellular receptor component, which recognizes a specific ligand or first messenger. Upon ligand binding, they all undergo conformational changes that activate the G protein found on the cytoplasmic side of the cell membrane. Instead of having a cytosolic domain that associates with a G protein, enzyme-linked receptors have cytosolic domain either that has intrinsic enzyme activity or that associates directly with an enzyme. The binding of the hormone to a special transmembrane receptor results in activation of the enzyme adenylyl cyclase at the intracellular portion of the receptor. This enzyme then catalyzes the formation of the second messenger cAMP, which has multiple effects on cell function. Insulin, for example, acts by binding to an enzyme-linked receptor. Ion channel-linked receptors are involved in the rapid synaptic signaling between electrically excitable cells.

A client with otitis media is ordered to receive amoxicillin. The client questions the nurse on how the drug works. Which statement describes how penicillin works?

This drug causes interference with a specific step in bacterial cell wall synthesis. Explanation: The four basic mechanisms of the antibiotic action are interference with a specific step in bacterial cell wall synthesis (e.g., penicillins, cephalosporins, glycopeptides, monobactams, carbapenems); inhibition of bacterial protein synthesis (e.g., aminoglycosides, macrolides, ketolides, tetracyclines, chloramphenicol, oxazolidinones, streptogramins, and rifampin); interruption of nucleic acid synthesis (e.g., fluoroquinolones, nalidixic acid); and interference with normal metabolism (e.g., sulfonamides, trimethoprim).

The nurse is caring for a client who has experienced hypovolemic shock secondary to penetrating multiple trauma. When caring for the client postoperatively, which of these factors does the nurse recognize places the client at risk for poor wound healing?

Tissue hypoxia Explanation: Impaired wound healing may occur due to poor blood flow (e.g., swelling) or preexisting health problems. In situations of trauma, a decrease in blood volume may cause a reduction in blood flow to injured tissues. Hyperbaric oxygen is a treatment that delivers 96% to 100% oxygen at greater than twice the normal atmospheric pressure at sea level to improve wound healing. Keloids are masses produced from excess scar tissue; scar tissue formation is the final phase of wound healing.

An oncologic client is scheduled to begin antiangiogenesis therapy. What is the goal of this type of treatment?

To limit the size of the tumor by limiting its ability to recruit blood vessels Explanation: Antiangiogenesis therapy is designed to inhibit the growth of new blood vessels, thus limiting the availability of blood to the growing tumor, resulting in decreased oxygen delivery and decreased ATP synthesis. There are no genetic effects on the tumor promoters or genetics.

A mature scar will likely be pale in color due to which rationale?

Transformation from a highly vascular granulation tissue into an avascular scar. Explanation: As the scar matures, vascular degeneration eventually transforms the highly vascular granulation tissue into a pale, largely avascular scar. Lack of malanocytes or elastic tissue is not responsbile for the pale skin. Use of sunscreen is always a good idea but not the primary reason for new scars to be pale in color.

A client comes to the clinic for evaluation of a sharp, intermittent, severe, stabbing facial pain that she describes as "like an electric shock." The pain occurs on only one side of her face. It seems to be triggered when she chews, brushes her teeth, or sometimes when she merely touches her face. Often it is accompanied by involuntary grimacing. What diagnosis is most likely?

Trigeminal neuralgia Explanation: Her symptoms are characteristic of trigeminal neuralgia, caused by damage to cranial nerve V, which carries impulses of touch, pain, pressure, and temperature to the brain from the face and jaw. Postherpetic neuralgia is a complication of shingles. Migraine headache symptoms feel as if they occur in one or more regions of the head, not the face. Complex regional pain syndrome affects either one arm or leg.

Which pain disorder is a manifestation of a disruption of cranial nerve function?

Trigeminal neuralgia Explanation: Trigeminal neuralgia is manifested by facial tics or spasms and characterized by paroxysmal attacks of stabbing pain that usually are limited to the unilateral sensory distribution of one or more branches of the trigeminal cranial nerve (CN V). Cranial nerve involvement is not implicated in postherpetic neuralgia, phantom limb pain, or TMJ syndrome.

The provider is doing genetic counseling with a client. He explains that which condition affects only females?

Turner syndrome Explanation: Turner syndrome describes an absence of all (45, X/0) or part of the X chromosome and, as such, is seen only in females. Klinefelter syndrome is a condition of testicular dysgenesis accompanied by the presence of one or more extra X chromosomes in excess of the normal XY complement. Most males with Kleinfelter syndrome have one extra X chromosome (47, XXY). Down syndrome, trisomy 21, can be seen in either sex. Fragile X syndrome, as with other X-linked disorders, affects boys more often than girls.

The type of hypersensitivity reaction that is mediated by IgG or IgM antibodies directed against target antigens on specific host cell surfaces or tissues is also known as which type of hypersensitivity reaction?

Type II Explanation: A type I hypersensitivity reaction is dependent on IgE-mediated activation of mast cells and basophils and the subsequent release of chemical mediators of the inflammatory response. Type II hypersensitivity reactions are mediated by IgG or IgM antibodies directed against target antigens on specific host cell surfaces or tissues and result in complement mediated phagocytosis and cellular injury. Type III hypersensitivity is caused by the formation of antigen-antibody immune complexes in the bloodstream, which are subsequently deposited in vascular epithelium or extravascular tissues and which activate the complement system and induce a massive inflammatory response. Type IV hypersensitivity involves tissue damage in which cell-mediated immune responses with sensitized T lymphocytes cause cell and tissue injury.

The nurse is assessing a client with diabetes and notes an area on the client's right foot as inflamed, necrotic, and eroded. The client states he accidentally slammed his foot in a door 2 weeks ago. The nurse would document this finding as a(n):

Ulceration Explanation: Ulceration refers to a site of inflammation where an epithelial surface (e.g., skin or gastrointestinal epithelium) has become necrotic and eroded, often with associated subepithelial inflammation. Ulceration may occur as the result of traumatic injury to the epithelial surface (e.g., peptic ulcer) or because of vascular compromise (e.g., foot ulcers associated with diabetes).The other options do not present these manifestations

A nurse is teaching a class on health promotion and includes information about the risk from ultraviolet radiation. Which concepts should be included in this class? Select all that apply.

Ultraviolet radiation increases the risk of skin cancer. Ultraviolet radiation damages DNA. Explanation: Ultraviolet radiation increases the risk of skin cancer and damages DNA. It is a concern year-round. Eighty percent of the sun's UV rays are able to pass through the clouds and fog. They block most UVB rays but won't block out UVA rays, which penetrate deep into the skin.

The nurse is assessing a newborn. What characteristic finding makes the nurse suspect the newborn has trisomy 21?

Upward slanting of eyes Explanation: Newborns with Down syndrome have short stubby fingers and a single palmar crease (simian crease). The other distinctive features include upward slanting eyes, protruding tongue, and low-set little ears.

Select the base used in transmitting genetic information needed for protein synthesis that replaces thymine in a ribonucleic acid (RNA) structure.

Uracil Explanation: A sequence of three of these bases forms the fundamental triplet code for 1 of the 20 amino acids used in protein synthesis in humans. This triplet code is called a codon. The molecular link between the DNA code of genes and the amino acid sequence of proteins is ribonucleic acid (RNA), a macromolecule similar in structure to DNA, except that uracil (U) replaces thymine (T) as one of the four bases.

Which of the following is an accurate method of assessing pain in children ages 3 to 8 years?

Using a faces scale Explanation: Children between ages 3 and 8 years can accurately point to a scale of faces that ranges from very tearful to very happy to indicate their pain level. Children older than 8 are accurately able to report pain on a numeric scale, or using words ranging from "none" to "the most I have ever had." Physiologic responses to pain, such as tachycardia and guarding, are not very accurate because they are nonspecific and may not occur with chronic pain.

An HIV-positive mother passes the virus to her infant during delivery. This type of transmission is known as:

Vertical transmission Explanation: The transmission of STIs is not limited to sexual contact. Vertical transmission of these agents, from the mother to child, can occur across the placenta or during birth when the mucous membranes of the child come in contact with infected vaginal secretions of the mother. The other options are not modes of transmission.

The nurse is caring for a client with common variable immunodeficiency (CVID) disorder. Which clinical manifestation of the disorder is common with this client?

Viral pneumonia Explanation: Clinical manifestations of CVID can begin at any time of life and most commonly include recurrent bacterial and viral infections of the respiratory tract.

A client with a history of migraine headaches tells the physician that he or she usually experiences an aura before the onset of the headache. The client is most likely experiencing:

Visual disturbances Explanation: Migraine aura is associated with visual symptoms, including flickering lights, spots, or loss of vision; sensory symptoms, including feeling of pins or needles, or numbness; and speech disturbances or other neurologic symptoms.

Which characteristic differentiates a migraine with aura from a migraine without aura?

Visual symptoms such as flickering lights precede the headache Explanation: An aura is visual (flickering lights, spots, or loss of vision), sensory (feeling of pins and needles, or numbness), and/or speech disturbance that precedes a migraine. Nonpharmacologic treatments may be used with varying success in both types of migraine, and nausea and vomiting may precede or accompany each. Changes in mood and affect are not central to an aura.

While educating a group of individuals about to undergo knee surgery, the nurse stresses the importance of eating a well-balanced diet, especially high in vitamins. Which vitamin promotes collagen synthesis to facilitate the wound to the knee to heal properly?

Vitamin C Explanation: Vitamin C is needed for collagen synthesis. In vitamin C deficiency, improper sequencing of amino acids occurs, proper linking of amino acids does not take place, the by-products of collagen synthesis are not removed from the cell, new wounds do not heal properly, and old wounds may pull apart. Vitamin A functions in stimulating and supporting epithelialization, capillary formation, and collagen synthesis. Vitamin K plays an indirect role in wound healing by preventing bleeding disorders that contribute to hematoma formation and subsequent infection.

When caring for a postoperative client, in order to promote wound healing, which of these nutrients does the nurse encourage the client to consume?

Vitamin C Explanation: Vitamins are essential cofactors for the daily functions of the body; vitamins A and C play an essential role in the healing process. Vitamin C is needed for collagen synthesis. Vitamin K plays an indirect role in wound healing by preventing bleeding disorders that contribute to hematoma formation and subsequent infection. Proteins, fats, carbohydrates and microminerals such as zinc are required for wound healing as well.

While examining a client, the practitioner notices large, flat, cutaneous pigmentations. They are a uniform light brown with sharply demarcated edges. He uses a Wood lamp to better detect the lighter spots. The practitioner knows that the pigmentations may be a component of which disorder?

Von Recklinghausen disease Explanation: A second major component of type 1 neurofibromatosis, or von Recklinghausen disease, is the presence of large, flat cutaneous pigmentations known as café au lait spots. A Wood lamp, which uses ultraviolet light, can be used to detect lighter spots. The other options are chromosomal disorders and do not exhibit these pigmentations.

Global infectious diseases are now being recognized. These diseases, once known as endemic to one part of the world, are now being found in other parts of the world because of international travel and a global marketplace. Which disease is considered a global infectious disease?

West Nile virus Explanation: Aided by a global market and the ease of international travel, the past 5 years have witnessed the importation or emergence of a host of novel infectious diseases. During the late summer and early fall of 1999, West Nile virus (WNV) was identified as the cause of an epidemic involving 56 clients in the New York City area. This outbreak, which led to seven deaths (primarily in the elderly), marked the first time that WNV had been recognized in the Western hemisphere since its discovery in Uganda nearly 60 years earlier. Coxsackie diseases, caused by the coxsackie virus; respiratory syncytial disease, better known as RSV; and hand, foot, and mouth disease are not considered global diseases.

A 20-year-old male presents at the clinic complaining of severe fatigue, night sweats, and fever. While taking the client's history, he reports having multiple sexual partners and unprotected sex. HIV/AIDS is suspected. What diagnostic test would be ordered to confirm the diagnosis?

Western blot assay Explanation: The HIV antibody test procedure consists of screening with an enzyme immunoassay (EIA), also known as enzyme-linked immunosorbent assay (ELISA), followed by a confirmatory test, the Western blot assay, which is performed if the EIA is positive. The complete metabolic panel cannot diagnose HIV/AIDS. The diagnostic test for HIV/AIDS is the ELISA, not the ALEA. The confirmatory test for HIV/AIDS is not the Eastern blot test.

Hypertrophy may occur as the result of normal physiologic or abnormal pathologic conditions. The increase in muscle mass associated with exercise is an example of physiologic hypertrophy. Pathologic hypertrophy occurs as the result of disease conditions and may be adaptive or compensatory. Examples of adaptive hypertrophy are the thickening of the urinary bladder from long-continued obstruction of urinary outflow and the myocardial hypertrophy that results from valvular heart disease or hypertension. What is compensatory hypertrophy?

When one kidney is removed, the remaining kidney enlarges to compensate for the loss. Explanation: Compensatory hypertrophy is the enlargement of a remaining organ or tissue after a portion has been surgically removed or rendered inactive. The body does not enlarge its major organs during times of malnutrition. Gene expression, not actin expression, stimulates the body to increase the muscle mass of the heart. Hypertrophy is not a progressive decrease in the size of anything; it is an increase in the size of cells.

The obstetrical nurse is caring for a client who has been treated for gestational diabetes. When teaching the client about the causes of gestational diabetes, the nurse should include which risk factor in the teaching?

Woman with a family history of diabetes Explanation: Gestational diabetes occurs most commonly in black, Hispanic/Latino and Native American/First Nation women. It most frequently affects women with a family history of diabetes; a history of stillbirth or spontaneous abortion; women who previously gave birth to a newborn with fetal anomaly or had a previous large-for-gestational-age infant; those who are obese; those of advanced maternal age; or those who have had five or more pregnancies.

The nurse is caring for an obese client who has had abdominal surgery. The medical record states the wound has developed a dehiscence. Which finding does the nurse anticipate observing when changing the dressing?

Wound edges are 1.5 inches apart. Explanation: Mechanical factors such as increased local pressure or torsion can cause wounds to pull apart, or dehisce.

A client comes into the emergency room with an animal bite to the hand. Which intervention should the nurse anticipate?

Wound irrigation Explanation: The nurse should anticipate wound irrigation as animal bites have a high risk for infection. Intravenous fluids and hyperbaric oxygen are not indicated. Bite wounds of the hand are not usually sutures to avoid loss of function.

A nurse is teaching a pregnant client about teratogenic drugs. Which drug category will the nurse emphasize for the client to avoid during pregnancy?

X Explanation: This system classifies all drugs approved after 1983 into five pregnancy risk categories: A, B, C, D, and X. Drugs in category A are the least dangerous, and categories B, C, and D are increasingly more dangerous. Those in category X are contraindicated during pregnancy because of proven teratogenicity.

A nurse is reading a report that refers to a gene on band 2, region 2 of the short arm of the X chromosome. The nurse understands that the location of the gene would typically be documented as which of the following?

Xp22 Explanation: Reference to a gene located on band 2, region 2 of the short arm of the X chromosome would typically be documented as Xp22. The X represents the X chromosome, the "p" refers to the short arm of the X. The first "2" represents the band and the second "2" represents the region of the short arm where the gene is located.

Select the option that best describes a single-celled organism that reproduces by a budding process.

Yeast Explanation: Yeasts are single-celled organisms, about the size of a red blood cell, and reproduce through budding. Molds produce long, hollow-branching filaments called hyphae. Bacteria are much smaller than a red blood cell. Viruses are much smaller than a red blood cell.

A mother is diagnosed with a bacterial infection and is worried that her newborn infant will also contract the infection. Which statement should the nurse include in the teaching plan for the client?

Your newborn has maternal IgG antibodies that were transferred through the placenta before birth, providing some protection from infection. Explanation: Passive immunity is immunity transferred from another source. The most common form of passive immunity is that conferred from mother to fetus. During fetal development, maternal IgG antibodies are transferred to the fetus via the placenta. After birth, the neonate also receives IgG antibodies from the mother in breast milk or colostrum. Therefore, infants are provided with some degree of protection from infection for approximately 3 to 6 months, giving their own immune systems time to mature.

A client recently diagnosed with leukocyte adhesion deficiency (LAD) asks, "Why am I always sick with an infection?" Which response by the nurse explains this rare autosomal recessive disorder?

Your white blood cells are not able to leave the blood vessels and move into the area of infection." Explanation: Leukocyte adhesion deficiency (LAD) is a rare autosomal recessive disorder characterized by immunodeficiency, resulting in recurrent infections. A WBC differential will reveal extremely elevated levels of neutrophils (on the order of 6-10 times normal) because they are unable to leave the blood vessels. Certain integrins play an important role in allowing white blood cells to pass through the vessel wall, a process called transmigration.

A client has been diagnosed with cancer and will begin aggressive treatment. The client's course of treatment includes drug therapies that affect cyclins and cyclin-dependent kinases (CDKs). This drug will achieve a therapeutic effect by:

affecting the progression of cells through the process of cell division. Explanation: The cyclins are a family of proteins that control the entry and progression of cells through the cell cycle. Manipulation of cyclins, CDKs, and CDK inhibitors serves as the basis for development of newer forms of drug therapy that can be used in cancer treatment. These types of drugs would not cause cell atrophy and they do not have a direct effect on the role of stem cells.

Which emergency department client is most likely demonstrating clinical manifestations of acute stress? A client with:

an acute heightened sense of alertness to surroundings and personnel Explanation: Acute stress is time limited, and reactions are those of the autonomic nervous system—the fight-or-flight response. Acute stress is short term and does not recur. Centrally, there is facilitation of neural pathways mediating arousal, alertness, vigilance, cognition, and focused attention, as well as appropriate aggression. Acute stress situations are controlled by a negative feedback system. When chronic stress continues, components of the system become abnormally active or fatigued and fail. The other distractors may relate to a physical problem and not "acute stress."

A nurse caring for a client who has undergone an antithyroid peroxidase (anti-TPO) antibody test with elevated results should be prepared to educate the client about which disease process?

ashimoto thyroiditis Explanation: Diagnosis of Hashimoto thyroiditis is usually made by detecting elevated levels of anti-thyroid peroxidase antibodies (TPOAb) in the serum. Gigantism is diagnosed as an excessive secretion of growth hormone. There may be high levels of insulin-like growth factor 1 (IGF-1), which also stimulates excessive skeletal growth. States of hyperparathyroidism are associated with increased levels of 1,25-(OH)2D3. Parathyroid hormone (PTH) is an important regulator of calcium and phosphate levels in the blood. Diabetes is diagnosed following fasting plasma glucose test level of 126 mg/dL (6.99 mmol/L) or greater. Glycated hemoglobin testing (Hemoglobin A1C) is a test that measures the quantity of a subtype of hemoglobin that has been glucated, meaning glucose molecules have become bound to the hemoglobin molecule. In conditions of hyperglycemia, the A1C level is increased.

When confronted with a decrease in work demands or adverse environmental conditions, most cells are able to revert to a smaller size and a lower, more efficient level of functioning that is compatible with survival. This decrease in cell size is known as:

atrophy. This decrease in cell size is called atrophy. Hypertrophy, metaplasia, and hyperplasia are all cellular adaptations that result in greater number or size. Dysplasia is a loss of cellular organization.

A client is diagnosed with an adenoma. The nursing student identifies this as a:

benign tumor of glandular epithelial tissue. Explanation: An adenoma is a benign tumor of glandular epithelial tissue. An osteoma is a benign tumor of bone tissue.

The nurse in an infectious-disease clinic will primarily treat injuries to tissues and cells caused by:

biologic agents. Explanation: Injury from biologic agents derives from bacterial and viral infections. The injuries differ from the other forms because they are able to replicate and can continue to produce their injurious effects. The etiology of infections does not normally include oncogenic agents, chemical agents, or calcifications.

An oncology client is about to begin chemotherapy. During the education, the nurse mentions that continuously dividing cells will be most affected by the chemotherapy. The client asks, "What are continuously dividing cells?" The nurse responds, "These are cells that continue to divide and replicate like:

cells on the surface of your skin." cells in your mouth." cells lining your GI tract." Explanation: Continuously dividing or labile tissues are those in which the cells continue to divide and replicate throughout life, replacing cells that are continually being destroyed. They include the surface epithelial cells of the skin, oral cavity, vagina, and cervix; the columnar epithelium of the gastrointestinal tract, uterus, and fallopian tubes; the transitional epithelium of the urinary tract; and bone marrow cells. Neurons lose their ability to proliferate once development of the nervous system is complete. Heart muscle cells were believed to not regenerate; however, recent findings suggest that about half of your heart's muscle cells are normally replaced over a lifetime. However, it is a slow process and these cells are not continuously dividing.

The student is reviewing the aging process. One group of theories of aging involves the shortening of telomeres until a critical minimal length is attained and then senescence ensues. These theories are known as:

cellular theories. Explanation: There are a number of cellular theories of senescence that are under investigation, including those that focus on telomere shortening.

The nurse is explaining congenital defects to a newly pregnant client. She explains that one of the most common birth defects is:

cleft lip with or without cleft palate. Explanation: Cleft lip with or without cleft palate is one of the most common birth defects. Other congenital defects that are thought to arise through multifactorial inheritance are clubfoot, congenital dislocation of the hip, congenital heart disease, pyloric stenosis, and urinary tract malformation.

A hostage situation has occurred at a high school campus. The school superintendent and faculty are concerned about the students experiencing posttraumatic stress disorder (PTSD). The most important intervention would be:

debriefing with a crisis team about the event as soon as possible. Explanation: Debriefing or talking about the traumatic event with a crisis team at the time it happened is an effective therapeutic tool. Strong family relations are important, but this would not be the time to work on strengthening them. Not all people involved in a traumatic event should use pharmacologic treatments, and never advise a client to "get over" an event.

Which explanation most accurately describes the characteristics of saprophytes? They:

derive energy from decaying organic matter. Explanation: Saprophytes are free-living organisms obtaining their growth from dead or decaying organic material in the environment. They are not necessarily spore producing, and they are not typical components of human microflora. Because most are bacterial or fungal, they contain both RNA and DNA.

Unlike other teratogens, alcohol exposure during pregnancy (fetal alcohol spectrum disorders) can have what harmful effect on the fetus?

developmental abnormalities throughout the prenatal and postnatal period Explanation: Unlike other teratogenic exposures that cause abnormalities during a short period of time during fetal development (usually in the early weeks), fetal alcohol spectrum disorders (FASD) damage the chromosomes of the developing fetus as long as alcohol continues to circulate through the fetal bloodstream. FASD is diagnosed by the presence of three findings that do not include liver abnormalities and are not life threatening. Facial features characteristic of FASD are most noticeable during childhood. The facial features of FASD may not be as apparent in older people. Each defect can vary in severity, probably reflecting the timing of alcohol consumption in terms of the period of fetal development, amount of alcohol consumed, and hereditary and other environmental influences.

A public health nurse notes an increase in regional throat cancer cases. Upon epidemiological studies, many of the throat cancer clients also had oral exposure to human papillomavirus (HPV). This exposure to HPV would be considered by:

direct contact with infected secretions. Explanation: Most STIs are spread by direct contact. In addition to causing infectious diseases, certain viruses also have the ability to transform normal host cells into malignant cells during the replication cycle. This group of viruses is referred to as oncogenic and includes certain retroviruses and DNA viruses, such as the Epstein-Barr virus, hepatitis B virus, and human papillomavirus. Vertical transmission is possible for many sexually transmitted pathogens, but direct contact is more common. Ingestion and penetration are less likely mechanisms.

A young man has been diagnosed with Huntington disease following several months of progressive muscle weakness. The man has two young children: one boy and one girl. The nurse should teach the man that:

each child has 50% chance of developing Huntington disease. Explanation: In a single-gene autosomal dominant disease such as Huntington, the affected parent has a 50% chance of transmitting the disorder to each offspring. Sex-linked differences do not exist.

A client presents to the emergency department following a major traffic accident. Though outwardly there are no apparent physical injuries found, the client is experiencing chest pain and heightened alertness, which the health care worker attributes to the first stage of general adaptation syndrome (GAS). The health care worker concludes the client is experiencing manifestations related to the release of:

epinephrine. Explanation: The general adaptation syndromes has three stages—the first is alarm (fight or flight); second is resistance (fight); and the third is exhaustion. The alarm stage is characterized by a generalized stimulation of the sympathetic nervous system (SNS) and the HPA, resulting in release of catecholamines and cortisol. Increased insulin release or TSH release is not part of the GAS.

Select the option that best identifies how adult cancers differ in origin from childhood cancers. Adult cancers originate from:

epithelial cells. Explanation: Adult cancers are typically of epithelial cell origin, while childhood cancers involve the hematopoietic system, nervous system, soft tissues, bone, and kidneys. Childhood cancers have a more primitive (embryonic) appearance.

The surgeon has documented that a client is developing "proud flesh" at the postoperative wound site. The nurse recognizes this as:

excessive granulation tissue. Explanation: Excessive granulation tissue, sometimes referred to as proud flesh, may form and extend above the edges of the wound, preventing re-epithelialization from taking place. Surgical removal or chemical cauterization of the defect allows healing to proceed. The other options are not viable for the question.

In a genetic disorder called xeroderma pigmentosum, an enzyme needed to repair sunlight-induced DNA damage is lacking. This autosomal recessive disorder is characterized by:

extreme photosensitivity and a greatly increased risk of skin cancer in skin that has been exposed to the sun. Explanation: In a genetic disorder called xeroderma pigmentosum, an enzyme needed to repair sunlight-induced DNA damage is lacking. This autosomal recessive disorder is characterized by extreme photosensitivity and a 2000-fold increased risk of skin cancer in sun-exposed skin. Exposure to sun causes the skin to toughen and become leathery feeling, but not in patches of pink pigmented skin. Vitiligo is a benign acquired skin disease of unknown cause, consisting of irregular patches of various sizes totally lacking in pigment and often having hyperpigmented borders. It can appear in the skin of any race and is not scaly. Photosensitivity is a sign of xeroderma pigmentosum, but this disease increases, not decreases, the person's risk of skin cancer.

A child is brought to the emergency department after falling and cutting the leg on a piece of fencing. While explaining the process of normal tissue repair, the nurse explains that certain cells, like fibroblasts, proliferate at times like this with the production of:

growth factors to help the cells respond and expand in numbers to repair the injury. Explanation: Several cell types proliferate during tissue repair, including remnants of injured parenchymal tissue cells, vascular endothelial cells, and fibroblasts. The proliferation of these cell types is driven by proteins called growth factors. The production of growth factors and the ability of these cells to respond and expand in sufficient numbers are important determinants of the repair process. Glucocorticoids do not help foster tissue repair. Blood does form clots to stop the bleeding at the site of injury; however, it does not play a role in new tissue repair.

Select the assessment data that place a client most at risk for the development of an opportunistic infection. A client who:

has a compromised immune system. is currently receiving chemotherapy. just completed 6 weeks of radiation therapy. has a diagnosis of malnutrition. Explanation: All microorganisms, even saprophytes and members of the normal flora, can be opportunistic pathogens capable of producing an infectious disease when the health and immunity of the host have been severely weakened by illness, malnutrition, or medical therapy.

The nurse is caring for a client who tried to become pregnant but suffered a spontaneous abortion (miscarriage) during her first trimester. What is the most likely etiology of the client's abortion?

he fetus had a chromosomal disorder. Explanation: Chromosomal abnormalities are thought to cause around half of first-term abortions. For most women, this possibility is a much more likely cause of abortion than radiation or toxins. X-linked disorders are not thought to be a major cause of spontaneous abortion.

The nurse explains to the hypertensive client that the increased workload required to pump blood against an elevated arterial pressure results in a progressive increase in left ventricular muscle mass. This is an example of:

hypertrophy. Explanation: In pressure overload, as occurs with hypertension, the muscle cells hypertrophy to increase the amount of functioning tissue.

A client with COPD controlled with long-term corticosteroids has developed an infection following bowel surgery. The nurse anticipated this complication since steroids:

impair the phagocytic property of leukocytes. Explanation: The therapeutic administration of corticosteroid drugs decreases the inflammatory process and may delay the healing process. These hormones decrease capillary permeability during the early stages of inflammation, impair the phagocytic property of the leukocytes, and inhibit fibroblast proliferation and function. Steroid side effects usually cause hyperglycemia, muscle wasting, and hypertension.

Our body's response to psychological perceived threats is not regulated to the same degree as our body's response to physiologic perceived threats. The psychological responses may be:

inappropriate and sustained. Explanation: The response to physiologic disturbances that threaten the integrity of the internal environment is specific to the threat; the body usually does not raise the body temperature when an increase in heart rate is needed. In contrast, the response to psychological disturbances is not regulated with the same degree of specificity and feedback control; instead, the effect may be inappropriate and sustained. No systems in the body are regulated by a positive feedback system. In cardiovascular physiology, the baroreflex or baroreceptor reflex is one of the body's homeostatic mechanisms for maintaining blood pressure. It has nothing to do with the body's response to a psychological threat.

A client is experiencing the early stages of an inflammatory process and develops leukocytosis. The nurse recognizes this as a/an:

increase in circulating neutrophils. Explanation: Leukocytosis, or the increase in white blood cells, is a frequent sign of an inflammatory response, especially those caused by bacterial infection. Leukocytosis occurs due to an increase in circulating neutrophils and eosinophils. Leukocytosis does not occur because of increased cell production, and blood supply is typically increased as part of the inflammatory process.

A fomite is a/an:

infection transfer agent. Explanation: Inanimate objects that carry an infectious agent are known as fomites.

Smooth muscle contractions are typically characterized as:

involuntary Smooth muscle is often called involuntary muscle because it contracts without the person willing it to contract. Voluntary, motor, and somatic contractions are all under conscious control.

Which pregnant woman likely faces the greatest risk of developing gestational diabetes? A client who:

is morbidly obese (defined as greater than 100 pounds over ideal weight). Explanation: Obesity is among the risk factors for gestational diabetes mellitus (GDM). Obstetric complications, multiple pregnancies, high triglycerides, and hypertension are not specific risk factors for GDM.

While the nurse is performing a skin assessment on a dark-skinned client, the nurse notes that the client has a healed wound on the leg but that the wound has an excess of scar tissue. The nurse documents this as:

keloid. Explanation: The nurse doucments the existence of excess of scar tissue on a healed wound as a keloid. These are more common in black clients. Proud flesh is an excess of granulation tissue in a healing wound. Remodeling is the third phase of wound healing. Epithelialization is the migtation, proliferation, and differentiation of epeithilial cells on the wound edges and occurs during the proliferative phase of wound healing.

Which health care provider is the highest priority for immediate assistance in the first 24 hours following the birth of a newborn with a cleft lip?

lactation consultant Explanation: Newborns with a cleft lip typically have difficulty with feeding, and the assistance of a lactation consultant may be of help in establishing feeding patterns. Oxygenation is not a typical problem, while activities of daily living and assistive devices are not relevant considerations. Although social work is often of assistance when a child is born with a congenital condition, a cleft lip has fewer implications than most other inherited disorders.

An infant was born with facial nerve paralysis that occurred with delivery. As the infant ages, it becomes apparent that the facial muscles affected by the nerve damage are not moving. Seeking surgical repair, the family asks why the damage to the child's face is not being repaired by the body. The health care provider states that neurons (connected to the facial muscles) are highly specialized cells that:

lose their ability to proliferate once development of the nervous system is complete. Explanation: Neurons, which are highly specialized cells, lose their ability to proliferate once development of the nervous system is complete. In constant renewing cell populations, like the GI tract, the more specialized cells are unable to divide and rely on parent cells of the same lineage. Growth factors aide in tissue repair, not repair of neurons.

A client's tissue biopsy reveals the presence of cells that appear to have abnormalities in shape and size. Carcinoma is suspected because these cells possess the characteristics of:

loss of differentiation. Explanation: The deviation from normal patterns of differentiation is a characteristic of cancer cells. Angiogenesis is also associated with malignancy, but this does not directly account for abnormalities in size and shape. Cohesiveness, adhesion, and cell density-dependent inhibition are characteristics of normal cells.

A nurse is caring for an infant born with a cleft lip and palate. The priority of care would address:

malnutrition. Explanation: The immediate problem in infants with cleft lip and palate is feeding because of their inability to suck to breast-feed or drink from a bottle. The others will be of concern at a later time and are not life threatening.

A middle-aged female client has been diagnosed with a thyroid condition. The nurse educates the client about the prescription and needed follow-up lab work, which will help regulate the dosage. The client asks, "Why do I not return to the clinic for weeks, since I am starting the medication tomorrow morning?" The nurse bases the answer on the knowledge that thyroid hormones:

may take days for the full effect to occur, based on the mechanism of action. Explanation: Hormones produce their effects through interaction with high-affinity receptors, which in turn are linked to one or more effector systems within the cell. These mechanisms involve many of the cell's metabolic activities, ranging from ion transport at the cell surface to stimulation of nuclear transcription of complex molecules. The rate at which hormones react depends on their mechanism of action. Thyroid hormone, which controls cell metabolism and synthesis of intracellular signaling molecules, requires days for its full effect to occur. None of the other distractors are accurate reasons to have the client return to the clinic weeks after starting the medication.

A client in the second trimester of pregnancy arrives for the first prenatal visit and admits that she and her partner traveled to South America where they went on a rain forest excursion when she was 4 weeks' pregnant. Which congenital mosquito-borne abnormality may be seen on ultrasound if the fetus is affected?

microcephaly Explanation: The Zika virus is a flavivirus transmitted by mosquitoes. Investigation by the World Health Organization revealed that it was assocated with neonates being born with microcephaly. More work is needed to uderstand the impact of Zika on transmission cycles and disease burden in both human and nonhuman populations. Spina bifida is a birth defect that occurs when the spine and spinal cord do not form properly. It falls under the broader category of neural tube defects. Syndactyly is one of the most common hereditary limb malformations depicting the fusion of certain fingers and/or toes. Gastroschisis is a birth defect of the abdominal wall. The neonate's intestines are found outside of the neonate's body, exiting through a hole beside the umbilical cord.

A client with diabetes who is diagnosed with a gangrenous right heel ulcer presents with a wound that has no line of demarcation, is spreading rapidly, and has a foul odor. The health care worker recognizes these manifestations as:

moist gangrene. Explanation: In moist gangrene, there is no line of demarcation between the normal and diseased tissues, and the spread of tissue damage is rapid. Moist gangrene often results in a foul odor caused by bacterial action at the site. Dry gangrene has a clear demarcation and spreads slowly. Gas and internal gangrene do not apply to this client's symptoms.

The bacteria that line the gut of a human help maintain normal gut health and provide essential nutrients. This type of relationship is:

mutualistic. Explanation: The term mutualism is applied to an interaction in which the microorganism and the host both derive benefits from the interaction. Commensalism is a relationship where the organism receives benefit at no harm to the host. A parasite is an organism that derives benefits from its biologic relationship with another organism with the potential of causing harm.

While reviewing the phases of wound healing, the students note that the first cells to arrive after the injury are the:

neutrophils. Explanation: The cellular phase of inflammation follows and is evidenced by the migration of phagocytic white blood cells that digest and remove invading organisms, fibrin, extracellular debris, and other foreign matter. The neutrophils arrive within minutes and are usually gone by day 3 or 4. They ingest bacteria and cellular debris. Within 24 to 48 hours, macrophages, which are larger phagocytic cells, enter the wound area and remain for an extended period. These cells, arising from blood monocytes, are essential to the healing process. The final component of the proliferative phase is epithelialization, which is the migration, proliferation, and differentiation of the epithelial cells at the wound edges to form a new surface layer that is similar to that destroyed by the injury.

A client had a positive Pap smear. The surgeon diagnosed "cancer in situ of the cervix." The client asks, "What does this mean?" From the following statements, which is most appropriate in response to this question? The tumor has:

not crossed the basement membrane, so it can be surgically removed with little chance of growing back. Explanation: Cancer in situ is a localized preinvasive lesion. As an example, with breast ductal carcinoma in situ, the cells have not crossed the basement membrane. Depending on its location, an in situ lesion usually can be removed surgically or treated so that the chances of recurrence are small. For example, cancer in situ of the cervix is essentially 100% curable.

The nurse is studying genetic disorders. She learns that an allele is:

one of two members of a gene pair, for a particular trait, in a chromosome pair. Explanation: The two members of a gene pair, one inherited from the mother and the other from the father, are called alleles. The presence of more than two chromosomes to a set is called polysomy. The physical trait associated with a particular gene is the phenotype.

The radiologist is reviewing potential types of radiation therapy for a client. Select the type of radiation that directly breaks down chemical bonds in a cell.

onizing radiation Explanation: Ionizing radiation affects cells by causing ionization of molecules and atoms in the cell by directly hitting the target molecules of the cell and/or by producing free radicals that interact with critical cell components. Non-ionizing radiation exerts its effects on the cell by causing a vibration and rotation of atoms and molecules. Sunlight is a form of ultraviolet radiation that induces skin damage by reactive oxygen species and by damage to the melanin-producing processes in the skin.

The physical therapist is evaluating a male client who has increased his muscle mass by exercising. The therapist understands this physiologic process is known as:

physiologic hypertrophy. Explanation: Hypertropy represents an increase in cell size and with it an increase in the amount of functioning tissue mass. The increase in muscle mass associated with exercise is an example of physiologic hypertrophy. Hyperplasia refers to an increase in the number of cells in an organ or tissue. There are two types of physiologic hyperplasia: hormonal and compensatory. Metaplasia represents a reversible change in which one adult cell type is replaced by another adult cell type. Dysplasia is characterized by deranged cell growth of a specific tissue that results in cells that vary in size, shape and organization.

Teratogenic substances cause abnormalities during embryonic and fetal development. These substances have been divided into three classes. These classes are called:

radiation; drugs and chemical substances; and infectious agents. Explanation: Teratogenic agents have been divided into three groups: radiation; drugs and chemical substances; and infectious agents. The period of organogenesis, the third trimester, and the second trimester are not teratogenic substances. They are time periods during the pregnancy. Teratogenic substances are not classified as outside, inside, or internal. Although drugs and chemical substances are a class of teratogenic agents, smoking is included in that class as a teratogenic agent. It is not a class unto itself. Bacteria and viruses are considered infectious agents and are therefore teratogenic agents.

The genetic counselor is explaining structural changes in chromosomes that result from breakage and rearrangement of chromosome parts. The pattern that occurs when there are simultaneous breaks in two chromosomes, from different pairs, with exchange of chromosome parts is:

ranslocation Explanation: Translocation occurs when there are simultaneous breaks in two chromosomes, from different pairs, with exchange of chromosome parts. Isochromosome formation occurs when the centromere, or central portion, of the chromosome separates horizontally instead of vertically. Ring formation results when deletion is followed by uniting of the chromatids to form a ring. Inversion requires two breaks on a single chromosome with inversion to the opposite side of the centromere or with the fragment inverting but remaining on the same arm.

After many years of cigarette smoking, a client is admitted to have a "mass" removed from the lung. When explaining the surgery and recovery, the physician notes that the client is likely to have a good amount of fibrosis develop at the surgical area. After the physician leaves the room, the client asks the nurse what was meant by "fibrosis" in the lung. The nurse bases the response on the fact that tissue repair can:

result in replacement tissue in the form of connective (fibrous) tissue, which leads to scar formation or fibrosis of the lung. Explanation: Tissue repair can take the form of replacement by connective (fibrous) tissue, which leads to scar formation or fibrosis in organs such as the liver or lung. Regeneration of tissue results with injured cells being replaced with cells of the same type. Although age does play a role in tissue healing, it is not the only factor that can result in scar formation.

While sponging a client who has a high temperature, the nurse observes the client begins to shiver. At this point, the priority nursing intervention would be to:

stop sponging the client and retake a set of vital signs. Explanation: Modification of the environment ensures that the environmental temperature facilitates heat transfer away from the body. Sponge baths with cool water or an alcohol solution can be used to increase evaporative heat losses. More profound cooling can be accomplished through the use of a cooling blanket or mattress, which facilitates the conduction of heat from the body into the coolant solution that circulates through the mattress. Care must be taken so that cooling methods do not produce vasoconstriction and shivering that decrease heat loss and increase heat production.

A client's low serum T4 level has led to a diagnosis of hypothyroidism. When planning this client's care, the nurse should:

teach the client about the safe and effective use of synthetic thyroid hormones. Explanation: Hypothyroidism is treated by replacement therapy with synthetic preparations of T3 or T4. Graves disease is associated with hyperthyroidism, not hypothyroidism. Surgery is not a usual treatment modality.

Following a severe automobile accident, a client is scheduled to have surgery to either repair or remove his spleen, pancreas, and stomach. The client wants the organs repaired and not removed if at all possible. However, the nursing staff understands that extensive regeneration in parenchymal organs can only occur if:

the residual tissue is structurally and functionally intact. Explanation: Regeneration can occur in parenchymal organs with stable cell populations but, with the exception of the liver, is usually a limited process. It should be pointed out that extensive regeneration can occur only if the residual tissue is structurally and functionally intact. If the tissue is damaged by infection or inflammation, regeneration is incomplete and accomplished by replacement with scar tissue.

A breast cancer client has just learned that her tumor clinical stage is T3, N2, M0. After the physician leaves, the client asks the nurse to explain this to her again. The nurse will use which statement in his or her answer? Your:

tumor is large and at least two lymph nodes are positive for cancer cells. Explanation: Tumor staging groups clients according to the extent and spread of the disease, using the TNM (tumor, node, and metastasis) system. In the TNM system, T1, T2, T3, and T4 describe tumor size, N0, N1, N2, and N3, lymph node involvement; and M0 or M1, the absence or presence of metastasis.

A client with many nonspecific complaints has been ordered a positron emission tomography (PET) scanning for evaluation of:

tumors located on the endocrine glands. Explanation: Positron emission tomography (PET) scanning is being used more widely for evaluation of endocrine tumors. PET scans do not calculate the pancreas response to insulin. A DEXA is used for diagnosis and monitoring of osteoporosis (bone density). Isotopic imaging includes radioactive scanning of the thyroid (using radioiodine) and parathyroids.

Mendelian, or single-gene, patterns of inheritance include autosomal dominant and recessive traits that are transmitted from parents to their offspring in a predictable manner. A recessive trait occurs when:

two copies (homozygous) of the recessive allele are present. Explanation: A recessive trait is one expressed only when two copies (homozygous) of the recessive allele are present. Dominant traits are expressed with either homozygous or heterozygous pairing of the alleles.

A client cuts herself with a sharp knife while cooking dinner. The client describes how the wound started bleeding and had a red appearance almost immediately. The nurse knows that in the vascular stage of acute inflammation, the vessels:

vasodilate, causing the area to become congested and resulting in the red color and warmth. Explanation: Vasodilation allows more blood and fluid into the area of injury, resulting in congestion, redness, and warmth. Vasodilation is quickly followed by increased permeability of the microvasculature. The loss of fluid results in an increased concentration of blood constituents (red blood cells, leukocytes, platelets, and clotting factors), stagnation of flow, and clotting of blood at the site of injury. This aids in limiting the spread of infectious microorganisms. The loss of plasma proteins increases fluid movement from the vascular compartment into the tissue space and produces the swelling, pain, and impaired function that are the cardinal signs of acute inflammation.


Conjuntos de estudio relacionados

HESI Case Studies- Benign Prostatic Hyperplasia

View Set

5 - MRW - Electrical Energy Fundamentals

View Set

Digital Information Technology Semester Exam Review

View Set

LUOA Earth Science: Module 3: What's Going on Inside the Earth?

View Set

Chapter 04: Patient and Caregiver Teaching

View Set

intro to observation and screening

View Set

N 3270 Ch. 6 Values, Ethics, and Advocacy (PrepU)

View Set

Chapter 32: Skin Integrity and Wound Care

View Set